› medical_mx › images › ... · 5 samantha hendren; ellen mckeown; arden m . morris; sandra l...

96
Medical Trends 2015 Edition Colorectal Cancer Current Clinical Issues & Future Trends Early and Locally Advanced Colorectal Disease Oncology Self-Study Series 2 JOURNAL of CLINICAL ONCOLOGY Includes content from the Journal of Clinical Oncology Editors-in-chief: Pere Gascón, MD, PhD Josep Tabernero, MD, PhD ASCO-CRC-2015-V2-INT.indb 1 4/8/2015 4:41:37 PM Copyright by MT

Upload: others

Post on 28-May-2020

1 views

Category:

Documents


0 download

TRANSCRIPT

Page 1: › medical_mx › images › ... · 5 Samantha Hendren; Ellen McKeown; Arden M . Morris; Sandra L . Wong; Mary Oerline; Lyndia Poe; Darrell A . Campbell Jr, and Nancy J . Birkmeyer

Medical Trends

2015 Edition

Colorectal CancerCurrent Clinical Issues & Future TrendsEarly and Locally Advanced Colorectal Disease

OncologySelf-Study Series

2JOURNAL ofCLINICAL ONCOLOGY

Includes content from the Journal of Clinical Oncology

Editors-in-chief:■ Pere Gascón, MD, PhD■ Josep Tabernero, MD, PhD

ASCO-CRC-2015-V2-INT.indb 1 4/8/2015 4:41:37 PM

Copyright by MT

Page 2: › medical_mx › images › ... · 5 Samantha Hendren; Ellen McKeown; Arden M . Morris; Sandra L . Wong; Mary Oerline; Lyndia Poe; Darrell A . Campbell Jr, and Nancy J . Birkmeyer

The ideas and opinions expressed in this publication do not necessarily reflect those of the American Society of Clinical Oncology. Products mentioned in this publication should not be construed as an endorsement of the product or the manufacturer’s claims. Readers are encouraged to contact the manufacturer with any questions about the features or limitaions of the products mentioned. The American Society of Clinical Oncology assumes no responsibility for any injury and/or damage to persons or property arising out of or related to any use of the material contained in these abstracts and articles. The reader is advised to check the appropriate medical literature and the product information currently provided by the manufacturer of each drug to be administered to verify the dosage, the method and duration of administration, or contraindications. It is the responsability of the treating physician or other health care professional, relying on independent experience and knowledge of the patient, to determine drug dosages and the best treatment for the patient. An effort has been made to check generic and trade names, and to verify drug doses. The ultimate responsibility, however, lies with the prescribing physician. Please convey any errors to the editors.

ASCO Oncology Self-Study Series, Colorectal Cancer: Current Clinical Issues & Future TrendsEarly and Locally Advanced Colorectal Disease

© 2015 American Society of Clinical Oncology

This program has been designed and developed by Medical Trends SL under permission and supervision of the American Society of Clinical Oncology.

Edited by Medical Trends, SLTravessera de les Corts, 55, 1.ª planta08028 Barcelona – Spain

Editors-in-chief: Pere Gascón, MD, PhD – Director, Division Medical Oncology – Scientific Coordinator, Department Hematology-Oncology, ICMHO – Clinic University Hospital – University of Barcelona

Josep Tabernero, MD, PhD – Medical Director of Cancer Institute Baselga (IOB). Director of the Gastrointestinal Cancer Unit (IOB). Head of Oncology, University Hospital Vall d’Hebron in Barcelona

All rights reserved. No part of this publication may be reproduced, stored in a retrieval system, or transmitted, in any form, or by any other means, electronic, mechanical, photocopying, recording, or otherwise, without the prior written permission of the copyright owner.

ASCO-CRC-2015-V2-INT.indb 2 4/8/2015 4:41:37 PM

Copyright by MT

Page 3: › medical_mx › images › ... · 5 Samantha Hendren; Ellen McKeown; Arden M . Morris; Sandra L . Wong; Mary Oerline; Lyndia Poe; Darrell A . Campbell Jr, and Nancy J . Birkmeyer

III

■■ Contents

LEARNING OBJECTIVES . . . . . . . . . . . . . . . . . . . . . . . . . . . . . . . . . . . . . . . . . . . . . . . . . . . 1Knowledge and skills acquired upon completion of this volume

SYLLABUS . . . . . . . . . . . . . . . . . . . . . . . . . . . . . . . . . . . . . . . . . . . . . . . . . . . . . . . . . . . . . 3Main body of information

■■ Samantha Hendren; Ellen McKeown; Arden M. Morris; Sandra L. Wong; Mary Oerline; et al.

Implementation of a Hospital-Based Quality Assessment Program for Rectal Cancer . . . . . . . . . . . . . . . . . . . . . . . . . . . . . . . . . . . . . . . . . . . . . . . . . . . . . . . . 5(J Oncol Pract 2014;10(3):e120–e129 .)

■■ Nadine J. McCleary; Efrat Dotan, and Ilene Browner

Refining the Chemotherapy Approach for Older Patients With Colon Cancer . . . . . . 19(J Oncol Pract 2014;32(24):2570–2580 .)

■■ Anthony C. Wong; Shannon Stock; Deborah Schrag; Katherine L. Kahn; Talya Salz; et al.

Physicians’ Beliefs About the Benefits and Risks of Adjuvant Therapies for Stage II and Stage III Colorectal Cancer . . . . . . . . . . . . . . . . . . . . . . . . . . . . . . . . . . . . . . . . . . . . 35(J Oncol Pract 2014;10(5):e360–e367 .)

INFLUENTIAL PAPERS . . . . . . . . . . . . . . . . . . . . . . . . . . . . . . . . . . . . . . . . . . . . . . . . . . . . 47Selected abstracts from highly relevant publications

HOT TOPICS . . . . . . . . . . . . . . . . . . . . . . . . . . . . . . . . . . . . . . . . . . . . . . . . . . . . . . . . . . . . 57A comprehensive selection of discussions and insights covering the hottest aspects related to colorectal cancer

■■ Helen M. Parsons; James W. Begun; Karen M. Kuntz; Todd M. Tuttle; Patricia M. McGovern; et al.

Lymph Node Evaluation for Colon Cancer in an Era of Quality Guidelines: Who Improves? . . . . . . . . . . . . . . . . . . . . . . . . . . . . . . . . . . . . . . . . . . . . . . . . . . . . . . . . . 59(J Clin Pract 2014;9(4):e164–e168 .)

■■ Talya Salz; Shrujal S. Baxi; Victoria S. Blinder; Elena B. Elkin; Margaret M. Kemeny; et al.

Colorectal Cancer Survivors’ Needs and Preferences for Survivorship Information . . . 67(J Oncol Pract 2014;10(4):e277–e282 .)

ASCO-CRC-2015-V2-INT.indb 3 4/8/2015 4:41:37 PM

Copyright by MT

Page 4: › medical_mx › images › ... · 5 Samantha Hendren; Ellen McKeown; Arden M . Morris; Sandra L . Wong; Mary Oerline; Lyndia Poe; Darrell A . Campbell Jr, and Nancy J . Birkmeyer

IV

Contents

LANDMARK CLINICAL TRIALS . . . . . . . . . . . . . . . . . . . . . . . . . . . . . . . . . . . . . . . . . . . . . 77Trial results that have highly influenced scientific knowledge and practice

Effect of flexible sigmoidoscopy screening on colorectal cancer incidence and mortality: a randomized clinical trial . . . . . . . . . . . . . . . . . . . . . . . . . . . . . . . . . 79

Preoperative magnetic resonance imaging assessment of circumferential resection margin predicts disease-free survival and local recurrence: 5-year follow-up results of the MERCURY study . . . . . . . . . . . . . . . . . . . . . . . . . . . . . . . . . . . . . . . . . . . 79

Tumor regression grading after preoperative chemoradiotherapy for locally advanced rectal carcinoma revisited: updated results of the CAO/ARO/AIO-94 trial . . . . . . . . . . 80

Neoadjuvant chemotherapy without routine use of radiation therapy for patients with locally advanced rectal cancer: a pilot trial . . . . . . . . . . . . . . . . . . . . . . . . . . . . 81

SELF-ASSESSMENT . . . . . . . . . . . . . . . . . . . . . . . . . . . . . . . . . . . . . . . . . . . . . . . . . . . . . . 83Questions for Self-Examination and detection of possible areas of interest in this volume

Questions . . . . . . . . . . . . . . . . . . . . . . . . . . . . . . . . . . . . . . . . . . . . . . . . . . . . . . . . . . . . . . 85

ASCO-CRC-2015-V2-INT.indb 4 4/8/2015 4:41:37 PM

Copyright by MT

Page 5: › medical_mx › images › ... · 5 Samantha Hendren; Ellen McKeown; Arden M . Morris; Sandra L . Wong; Mary Oerline; Lyndia Poe; Darrell A . Campbell Jr, and Nancy J . Birkmeyer

1

■■ Learning Objectives

After completion of this volume, the reader will be able to:

1. Identify the best approach to using chemotherapy in older patients

with colon cáncer

2. Assess physicians’ current perceptions of the benefits and risks

of adjuvant therapies for stage II and III colorectal cancer

3. Describe the effect of 3- to 5-years of scheduled carcincoembryonic

antigen and computed tomography follow-up to detect recurrence

in colorectal cancer

4. Evaluate the quality-guidelines data on the advantages and

limitations of lymph-node resection in colon cancer

ASCO-CRC-2015-V2-INT.indb 1 4/8/2015 4:41:37 PM

Copyright by MT

Page 6: › medical_mx › images › ... · 5 Samantha Hendren; Ellen McKeown; Arden M . Morris; Sandra L . Wong; Mary Oerline; Lyndia Poe; Darrell A . Campbell Jr, and Nancy J . Birkmeyer

ASCO-CRC-2015-V2-INT.indb 2 4/8/2015 4:41:37 PM

Copyright by MT

Page 7: › medical_mx › images › ... · 5 Samantha Hendren; Ellen McKeown; Arden M . Morris; Sandra L . Wong; Mary Oerline; Lyndia Poe; Darrell A . Campbell Jr, and Nancy J . Birkmeyer

■■ Syllabus

ASCO-CRC-2015-V2-INT.indb 3 4/8/2015 4:41:37 PM

Copyright by MT

Page 8: › medical_mx › images › ... · 5 Samantha Hendren; Ellen McKeown; Arden M . Morris; Sandra L . Wong; Mary Oerline; Lyndia Poe; Darrell A . Campbell Jr, and Nancy J . Birkmeyer

ASCO-CRC-2015-V2-INT.indb 4 4/8/2015 4:41:37 PM

Copyright by MT

Page 9: › medical_mx › images › ... · 5 Samantha Hendren; Ellen McKeown; Arden M . Morris; Sandra L . Wong; Mary Oerline; Lyndia Poe; Darrell A . Campbell Jr, and Nancy J . Birkmeyer

5

■■ Samantha Hendren; Ellen McKeown; Arden M . Morris; Sandra L . Wong; Mary Oerline; Lyndia Poe; Darrell A . Campbell Jr, and Nancy J . Birkmeyer

Implementation of a Hospital-Based Quality Assessment Program for Rectal Cancer(J Oncol Pract 2014;10(3):e120–e129.)

Purpose: Quality improvement programs in Europe have had a markedly beneficial effect on the processes and outcomes of rectal cancer care . The quality of rectal cancer care in the United States is not as well understood, and scalable quality improvement programs have not been developed . The purpose of this article is to describe the implementation of a hospital-based quality assessment program for rectal cancer, targeting both community and academic hospitals .Methods: We recruited 10 hospitals from a surgical quality improvement organization . Nurse reviewers were trained to abstract rectal cancer data from hospital medical records, and abstracts were assessed for accuracy . We conducted two surveys to assess the training program and limitations of the data abstraction . We validated data completeness and accuracy by comparing hospital medical record and tumor registry data .Results: Nine of 10 hospitals successfully performed abstractions with ≥ 90% accuracy . Experienced nurse review-ers were challenged by the technical details in operative and pathology reports . Although most variables had less than 10% missing data, outpatient testing information was lacking from some hospitals’ inpatient records . This implementation project yielded a final quality assessment program consisting of 20 medical records variables and 11 tumor registry variables .Conclusion: An innovative program linking tumor registry data to quality-improvement data for rectal cancer quality assessment was successfully implemented in 10 hospitals . This data platform and training program can serve as a template for other organizations that are interested in assessing and improving the quality of rectal cancer care .

■■ Introduction

Rectal cancer (RC) care has been the subject of intensive quality measurement and improvement programs in Europe.1 In European trials, surgery, pathology, and adjuvant therapy have been standardized, which has improved outcomes.2–5 For rectal cancer, the quality of surgical technique is particularly relevant. For example, a surgical quality improvement program in Norway resulted in local recurrence rates drop-ping from 12% to 6% and 4-year survival increasing from 60% to 73%.6 Population-based registries in at least eight countries

have also facilitated quality assessment over time.7 RC registries permit comparison of processes and outcomes, as well as iden-tification of areas for quality improvement.

By contrast, the quality of care for RC is poorly understood and more variable in the United States. Although medical aspects of RC care have been targeted for process improvement by organizations such as ASCO (via its Quality Oncology Practice Initi-ative) and the National Quality Forum, these programs are generally based on outpatient oncology practices and do not include the hospital-based, surgical phase of care.

Key points

•Rectal cancer (RC) care has been the subject of intensive quality meas-urement and improvement programs in Europe.

•RC registries permit comparison of processes and outcomes, as well as identification of areas for quality improvement.

•RC is poorly understood and more variable in the United States.

ASCO-CRC-2015-V2-INT.indb 5 4/8/2015 4:41:37 PM

Copyright by MT

Page 10: › medical_mx › images › ... · 5 Samantha Hendren; Ellen McKeown; Arden M . Morris; Sandra L . Wong; Mary Oerline; Lyndia Poe; Darrell A . Campbell Jr, and Nancy J . Birkmeyer

Syllabus

6

However, hospital-based registries do exist for quality improvement in surgery, such as the nationwide American College of Surgeons’ National Surgical Quality Improvement Program (ACS-NSQIP).8–10 Unfortunately, the ACS-NSQIP database does not contain any cancer-specific vari-ables for rectal cancer. Although studies have been performed that link ACS-NSQIP data to national tumor registry data, these studies still lack critical clinical information, such as tumor location, clinical staging tests, CEA level, surgical approach, type of anastomosis, and whether a total mesorec-tal excision was performed.11–14

Regional surgical quality assessment organ-izations also exist, such as the Michigan Surgical Quality Collaborative (MSQC).15–18 Similar to ACS-NSQIP, the MSQC collects patient, surgical, and 30-day outcomes data for surgery cases. These outcomes data are then fed back to the hospitals as risk-adjusted hospital comparisons. The organization holds regular meetings of surgeons and data abstractors to share best practices and motivate process improve-ment. Regional collaborative organizations like MSQC have not generally focused on cancer surgery quality, but they could potentially be adapted for this purpose.

In this context, the purpose of our study was to implement a hospital-based, quality assessment program for RC in the setting of the MSQC organization. The MSQC tar-gets both community and academic medi-cal centers, unlike other programs based in large, tertiary cancer centers. The goals of this report are (1) to demonstrate that surgical data abstractors can accurately abstract cancer-specific data from hospital medical records, (2) to describe an innova-tive linkage of chart abstraction data with tumor registry data at the hospital level, (3) to share lessons learned about chal-lenges of collecting data from hospital records, and (4) to present a final variable list that other regional or national groups might find helpful in efforts to measure RC quality of care.

■■ Methods

■■ SettingThe MSQC is an organization of 52 hos-pitals with the coordinating center at the University of Michigan, Ann Arbor. MSQC is financially supported by Blue Cross/ Blue Shield of Michigan, a not-for-profit private health insurance company, and its mission is to promote surgical safety and quality.15–18

■■ Selection of variablesSurgeons with an interest in colorectal can-cer quality of care designed and modified a set of key variables representing RC clinical features, processes of care, and outcomes. National and international publications and guidelines were reviewed, including the National Quality Forum (NQF), the National Comprehensive Cancer Network (NCCN) Clinical Practice Guidelines, the Ameri-can Joint Committee on Cancer (AJCC) 2010 manual, QOPI, and the College of American Pathologists’ (CAP) manual.

Ultimately, the variables were designed to allow for the following quality measures to be measured over time: lymph node procurement (an NQF and ASCO qual-ity measure); performance of mesorectal excision (NCCN guidelines recommended process); grading of mesorectal exci-sion (included in CAP guidelines); use of neoadjuvant chemoradiotherapy for clinical stage II/III cases (NCCN guidelines recommended process); margin positivity rate (strongly associated with local recur-rence rate);3 ostomy nurse care; sphincter preservation rate among eligible patients (outcome); anastomotic leak rate (out-come); recurrence rate (outcome); and mortality (outcome).

Initial variables consisted of 44 variables to be abstracted from the medical record, plus 22 variables to be obtained from the hospital’s tumor registrar. Tumor registry variables were modeled after the Facility Oncology Registry Data Standards (FORDS) manual (published by the American College

Key points

•Hospital-based registries do exist for quality improvement in surgery, such as the nationwide American College of Surgeons’ National Surgical Quality Improvement Program (ACS-NSQIP).

•Regional surgical quality assessment organizations also exist, such as the Michigan Surgical Quality Collaborative (MSQC). Similar to ACS-NSQIP, the MSQC collects patient, sur-gical, and 30-day out comes data for surgery cases.

•Regional collaborative organizations like MSQC have not generally focused on cancer surgery quality, but they could potentially be adapted for this purpose.

•The purpose of our study was to implement a hospital-based, quality assessment program for RC in the setting of the MSQC organization.

•The MSQC targets both community and academic medical centers, unlike other programs based in large, tertiary cancer centers.

•MSQC is financially supported by Blue Cross/ Blue Shield of Michigan, a not-for-profit private health insurance company, and its mission is to promote surgical safety and quality.

•Surgeons with an interest in colorectal cancer quality of care designed and modified a set of key variables representing RC clinical features, processes of care, and outcomes.

•National and international publications and guidelines were reviewed.

ASCO-CRC-2015-V2-INT.indb 6 4/8/2015 4:41:37 PM

Copyright by MT

Page 11: › medical_mx › images › ... · 5 Samantha Hendren; Ellen McKeown; Arden M . Morris; Sandra L . Wong; Mary Oerline; Lyndia Poe; Darrell A . Campbell Jr, and Nancy J . Birkmeyer

Implementation of a Hospital-Based Quality Assessment Program

7

Key points

•A standardized data abstraction manual was created, with standardized response options, diagrams, and answers to anticipated questions.

•Variables were pilot tested by the authors using de-identified charts from the surgeons’ practices.

•Eligibility criteria were cases with an International Classification of Diseases, Ninth Edition (ICD-9) diagnosis code of 154.1, and a surgical procedure code.

•A training program was designed that included detailed variable definitions, three sample test cases, conference calls, and a final test case on which each reviewer was formally scored. A score of 90% accuracy was required.

•The abstractors were asked to complete two internet surveys designed by the authors. The first survey was performed at the conclusion of the training program.

•The second survey was performed at the end of the data abstraction, with a goal of determining which variables were problematic (data not available and/or difficult to interpret) from the viewpoint of the abstractors.

•Variables that exist in both tumor registry data and the hospital chart (or for which two hospital chart variables were complimentary) were validated by calculating the concordance between the two sources.

of Surgeons’ Commission on Cancer, 2012 edition). A standardized data abstraction manual was created, with standardized response options, diagrams, and answers to anticipated questions. Variables were pilot tested by the authors using de-identified charts from the surgeons’ prac-tices, and the final list of variables had greater than 90% inter-abstractor reliability on three test cases.

■■ Patient case eligibilityRC patient cases previously captured in the MSQC database (surgery dates July 1, 2007, to June 6, 2012) were included. Eligibility criteria were cases with an International Classification of Diseases, Ninth Edition (ICD-9) diagnosis code of 154.1, and a surgical procedure code rep-resenting RC excision (44,140, 44,145, 44, 146, 44, 47, 44, 204, 44, 207, 44, 208, 45, 111, 45,112, 45, 114, 45,119, 45, 397, 45, 110, 45, 395, 45123, 45160, 45170, 45190, 44141, 44143, 44144, and 44206). Confirmation of the diagnosis of invasive adenocarcinoma, and that the cancer was primary rather than a recurrence, were additional inclusion criteria.

■■ Participating hospitals and data abstractor training

Ten MSQC hospitals that expressed inter-est in participating in a cancer surgery-focused “special project” were recruited to participate. Nine of 10 data abstractors were registered nurses, working full-time in their respective hospitals as trained data abstraction and quality improvement per-sonnel for the MSQC program. This project consisted of two activities, case abstraction and work with the tumor registrar. Abstrac-tors were offered an incentive payment ($35 per chart). Although the abstractors were experienced surgical chart abstractors, cancer variables were unfamiliar to them. A training program was designed that included detailed variable definitions, three sample test cases, conference calls, and a final test case on which each reviewer was formally scored. A score of 90% accuracy was required.

■■ SurveysThe abstractors were asked to complete two internet surveys designed by the authors. The first survey was performed at the conclusion of the training program, with a goal of determining whether the training program was effective from the abstractors’ perspective. The second sur-vey was performed at the end of the data abstraction, with a goal of determining which variables were problematic (data not available and/or difficult to interpret) from the viewpoint of the abstractors.

■■ Data analysis and human subjects protection

Variables that exist in both tumor registry data and the hospital chart (or for which two hospital chart variables were complimentary) were validated by calculating the concord-ance between the two sources. Hospital characteristics were publically available from the American Hospital Association. Descrip-tive statistics were used to analyze the pro-portion of missing data and the concordance between variables that were cross-validated. Analyses were performed using SAS for Windows and Microsoft Excel. This project was reviewed by the institution review board of the University of Michigan, and deemed institution review board exempt.

■■ Results

■■ Characteristics of participating hospitals

Ten hospitals participated in the program. Hospitals varied in their characteristics, including number of beds (five with < 400, two with 400–700, three with > 700), teaching status (six major teaching, two minor teaching, and two nonteaching), urban/rural location (nine urban, one rural), cancer center designation (three Compre-hensive Community Cancer Program, one Integrated Network Cancer Program, five Academic Comprehensive Cancer Pro-gram, one National Cancer Institute Des-ignated Comprehensive Cancer Program), and colorectal cancer surgery volume

ASCO-CRC-2015-V2-INT.indb 7 4/8/2015 4:41:37 PM

Copyright by MT

Page 12: › medical_mx › images › ... · 5 Samantha Hendren; Ellen McKeown; Arden M . Morris; Sandra L . Wong; Mary Oerline; Lyndia Poe; Darrell A . Campbell Jr, and Nancy J . Birkmeyer

Syllabus

8

(one with < 25 Medicare colorectal cancer patients per year, four with 25–50, four with 50–75, and one with >75).

■■ Success of the training programData abstractors were trained during a 6-week program that began with distribu-tion of a detailed set of definitions, abstrac-tion of test cases, and weekly conference calls. A specialist surgeon was present on conference calls to answer questions and provide education. Test cases revealed that the most important challenges lay in under-standing the technical details of operation reports and pathology reports. The train-ing program was modified in several ways. We increased the number of conference calls, established a “frequently asked ques-tions” document, created a mechanism for reviewers to ask questions of a specialist surgeon throughout the abstraction process, and modified definitions. This included selective use of referenced medical diagrams to help reviewers conceptualize

the data elements, as well as simplification or elimination of several variables.

Nine of the 10 reviewers scored at least 90% on the final of three sample cases (range, 90% to 98%), and the 10th hospi-tal’s reviewer scored 87%. To ensure data accuracy, the 10th hospital’s cases under-went physician review of de-identified reports. The abstractors’ satisfaction with the training program was assessed with a seven-question survey conducted online at the conclusion of the training program (response rate 100%). Reviewers’ question-naire responses indicate they felt the train-ing was very effective (Table 1).

■■ Analysis of data completenessOf 383 cases originally identified from the MSQC database using ICD-9 and Current Procedural Terminology (CPT) codes, 353 were eligible on the basis of chart review. Common reasons for ineligibility were squa-mous cell carcinoma, locally recurrent rectal

■■ TABLE 1 - Results

Training Program Evaluation Questionnaire

Questions Results

1. Training met my needs with respect to learning the variables and definitions.

70% strongly agree, 30% agree

2. Training provided me with sufficient opportunity to apply my knowledge to case studies.

77.8% strongly agree, 22.2% agree

3. Training was conducted in a way that allowed me sufficient opportunity to ask questions/obtain clarification.

100% strongly agree

4. The training methodology utilized (variable review + conference call, case study + conference call x 2 and graded case study with individualized feedback) was an efficient use of my time.

80% strongly agree, 20% agree

5. My expectations for the Rectal Cancer Surgery Pilot project training were met.

80% strongly agree, 20% agree

6. What aspect of the Rectal Cancer Surgery Pilot project training was of greatest value to you? (open-ended).

Comments (paraphrased): questions and answers in the conference calls; case studies; having surgeon on calls to answer questions; having someone available to call or email with questions any time

7. How would you improve the Rectal Cancer Surgery Pilot project training for the future? (open-ended).

Comments (paraphrased): a group meeting or class; decrease number of variables

Key points

•Data abstractors were trained during a 6-week program that began with distribution of a detailed set of definitions, abstraction of test cases, and weekly conference calls.

•This included selective use of referenced medical diagrams to help reviewers conceptualize the data elements, as well as simplification or elimination of several variables.

•Nine of the 10 reviewers scored at least 90% on the final of three sample cases (range, 90% to 98%), and the 10th hospital’s reviewer scored 87%. To ensure data accuracy, the 10th hospital’s cases underwent physician review of de-identified reports.

ASCO-CRC-2015-V2-INT.indb 8 4/8/2015 4:41:38 PM

Copyright by MT

Page 13: › medical_mx › images › ... · 5 Samantha Hendren; Ellen McKeown; Arden M . Morris; Sandra L . Wong; Mary Oerline; Lyndia Poe; Darrell A . Campbell Jr, and Nancy J . Birkmeyer

Implementation of a Hospital-Based Quality Assessment Program

9

Variable abstraction questionnaire

Survey question Variable (no. of reviewers)*

1. Variables that you found to be the most challenging from a data abstraction standpoint

Distance to radial margin (5)Mesorectal excision (3)Rectal cancer location (3)Distance to distal margin (3)Bowel anastomosis type (2)Preop staging test (2)TME grade (2)Iatrogenic perforation (2)Inter-sphincteric dissection (2)

2. Variables that were not available as a result of data access issues (eg, tests in outpatient not inpatient charts)

Preoperative staging test (4)Preoperative wound ostomy nurse referral (3)Preoperative CEA (3)Ostomy marked (2)Rectal cancer location and distance (2)

3. Variables that were not available as a result of absent or inconsistent clinician documentation practices

TME grade (6)Fecal incontinence (3)Mesorectal excision (3)Mesorectal excision (2)Preoperative staging test (2)Patient preference for colostomy (2)Distance from tumor to radial margin (2)

4. Variables that were not available as a result of inconsistency of location in the medical record

Rectal cancer location/distance (5)Preoperative CEA level (2) Preoperative staging test (2)

5. Variables that you believe to be least reliable Distance from tumor to distal margin (3)Distance from tumor to radial margin (3)Intersphincteric dissection (3)Mesorectal excision, total or partial (2)Rectal cancer location/distance (2)

6. Variables that you feel would benefit most from revisions to the definition or response options

Mesorectal excision, total or partial (5)Distance from tumor to distal margin (3)Distance from tumor to radial margin (3)Rectal cancer location (3)Preoperative staging test (ERUS/MRI) (2)

*Variable listed if > 1 reviewer identified difficult or unreliable abstraction.CEA, carcinoembryonic antigen; ERUS, endorectal ultrasound; MRI, magnetic resonance imaging.

■■ TABLE 1 - Results (continued )

cancer, or noninvasive tumors. Twenty cases were local excisions, and 333 were radical operations. The complete list of variables is presented in Table 2, along with the pro-portion of missing data for each variable.

There were two types of variables: those abstracted from the medical record and those obtained from each hospital’s tumor registrar. Among variables abstracted from the medical record, missing data were uncommon (< 10%, Table 2). Notable

exceptions with higher frequency of miss-ing data included distance from tumor to radial margin, total mesorectal excision grade, inter-sphincteric dissection, CEA level, preoperative clinical staging test, and number of metastatic lesions. Among the tumor registry data, only pathologic TNM stage and microsatellite instability were missing frequently.

We intentionally pilot tested several vari-ables to determine current documentation

Key points

•There were two types of variables: those abstracted from the medical record and those obtained from each hospital’s tumor registrar.

ASCO-CRC-2015-V2-INT.indb 9 4/8/2015 4:41:38 PM

Copyright by MT

Page 14: › medical_mx › images › ... · 5 Samantha Hendren; Ellen McKeown; Arden M . Morris; Sandra L . Wong; Mary Oerline; Lyndia Poe; Darrell A . Campbell Jr, and Nancy J . Birkmeyer

Syllabus

10

■■ TABLE 2 - Rectal cancer quality assessment variables

Final variables

Category Variables (variable type)

Response options Missing/total cases

No. %

Pathology Invasive adenocarcinoma (inclusion criterion)

Yes/no 0/353 (0)

Pathologic stage (clinical feature)

Pathologic T stage:Tx, T0, Tis, T1, T2, T3, T4a, T4b

23/353 (6.5)

Pathologic N stage:Nx, N0, N1, N1a, N1b, N1c, N2, N2a, N2b

29/353 (8.2)

Pathologic M stage:Mx, M0, M1, M1a, M1b

49/353 (13.9)

Total No. of lymph nodes examined (quality measure)

# 8/333 (2.4)

Positive margin (quality measure)

Yes/no (simplified variable) 17/333 (5.1)

TME grade (quality measure)

Grade 3-goodGrade 2-moderateGrade 1-poorNot graded

293/333 (88.0, including

“not graded”)

Surgery Surgical approach (clinical feature)

OpenLaparoscopicLaparoscopic, hand-assistedLaparoscopic, converted to openRoboticRobotic, converted to openLaparoscopic colon mobilization with open rectal surgeryLaparoscopic, hand-assisted colon mobilization with open rectal surgery

0/333 (0)

Mesorectal excision (quality measure)

Yes/no (simplified response options) 0/333 (0)

Bowel anastomosis (clinical feature)

Stapled with EEA stapler, end-to-endStapled with EEA stapler, side-to-endStapled with EEA stapler, pouch or coloplastyStapled with GIA stapler Hand sutured via the abdomenHand sutured via the anusNo anastomosis

0/333 (0)

Ostomy (clinical feature)

IleostomyColostomyNone

0/353 (0)

Perioperative Clinical presentation (risk adjustment variable)

Symptomatic/asymptomatic (simplified response options) 0/353 (0)

CEA level/date (clinical feature)

Nanograms per milliliter Level: 80/353 Date:

83/353

(22.7)(23.5)

Preoperative staging test (clinical feature)

Transrectal ultrasoundMRIBothTest type not documentedNo test/results documented

163/353 (46.2)

Clinical T/N staging (clinical feature)

Clinical T stage:Tx, T0, Tis, T1, T2, T3, T4a, T4b, unknown

173/353 (49.0)

ASCO-CRC-2015-V2-INT.indb 10 4/8/2015 4:41:38 PM

Copyright by MT

Page 15: › medical_mx › images › ... · 5 Samantha Hendren; Ellen McKeown; Arden M . Morris; Sandra L . Wong; Mary Oerline; Lyndia Poe; Darrell A . Campbell Jr, and Nancy J . Birkmeyer

Implementation of a Hospital-Based Quality Assessment Program

11

Final variables

Category Variables (variable type)

Response options Missing/total cases

No. %

Clinical N stage:N1, N0, unknown

188/353 (53.3)

Metastatic cancer, preoperative (clinical feature)

LiverLungPelvis or retroperitoneumOtherNone

0/353 (0)

Preoperative/intraoperative radiation therapy (quality measure)

Preoperative radiationIntraoperative radiationNo radiation

0/353 (0)

Preoperative WOCN referral/consultation (quality measure)

Yes/no 0/353 (0)

Ostomy marked preoperatively (quality measure)

Yes/no 0/353 (0)

Rectal cancer location (clinical feature)

Upper/proximal third of rectumMiddle third of rectumLower/distal third of rectum

6/62 (9.7)

Postoperative outcomes

Anastomotic leak (outcome)

NoneTreated with antibioticsTreated with percutaneous drainageReoperation with new anastomosisReoperation with proximal diversionReoperation with end ostomy

0/353 (0)

For APR/Hartmann’s procedures only

Reason for permanent colostomy documented? (clinical feature, used in assessment of the outcome: sphincter preservation rate)

Yes/No (simplified variable) 4/86 (4.4)

Tumor registry Date of initial (colorectal cancer) diagnosis (clinical feature)

Date 11/353 (3.1)

Derived AJCC-6 stage group (clinical feature)

0, I, IIA, IIB, IIC, IIIA, IIIB, IIIC, IVA, IVB 44/353 (12.5)

Derived AJCC-7 stage group (clinical feature)

0, I, IIA, IIB, IIC, IIIA, IIIB, IIIC, IVA, IVB, data not available 208/353 (58.9, unavailable for earlier

cases)

Clinical stage group (clinical feature)

0, 0A, OIS, 1, 1A, 1A1, 1A2, 1B, 1B1, 1B2, 1C, IS, 2, 2A, 2A1, 2A2, 2B, 2C, 3, 3A, 3B, 3C, 3C1, 3C2, 4, 4A, 4A1, 4A2, 4B, 4C, OC, 88, 99

7/353 (2.0)

Clinical TNM stage (clinical feature)

Clinical T stage:Tx, T0, Tis, T1, T2, T3, T4a, T4b

28/353 (7.9)

Clinical N stage:Nx, N0, N1, N1a, N1b, N1c, N2, N2a, N2b

25/353 (7.1)

Clinical M stage:Mx, M0, M1, M1a, M1b

37/353 (10.5)

■■ TABLE 2 - Rectal cancer quality assessment variables (continued)

ASCO-CRC-2015-V2-INT.indb 11 4/8/2015 4:41:38 PM

Copyright by MT

Page 16: › medical_mx › images › ... · 5 Samantha Hendren; Ellen McKeown; Arden M . Morris; Sandra L . Wong; Mary Oerline; Lyndia Poe; Darrell A . Campbell Jr, and Nancy J . Birkmeyer

Syllabus

12

Final variables

Category Variables (variable type)

Response options Missing/total cases

No. %

Date chemotherapy started (clinical feature)

Date 0/353 (0%)

Date of first recurrence (outcome)

Date 0/353 (0%)

Type of first recurrence (outcome)

00, 04, 06, 10, 13, 14, 15, 16, 17, 20, 21, 22, 25, 26, 27, 30, 36, 40, 46, 51, 52, 53, 54, 55, 56, 57, 58, 59, 60, 62, 70, 88, 99

Date of last contact or death (clinical feature)

Date 9/353 (2.5)

Vital status (outcome) Alive/dead 9/353 (2.5)

Cancer status (outcome)

No evidence of this tumor, evidence of this tumor, unknown 34/353 (9.6)

Variables eliminated

Category Variables Response options n missing/total cases

(%)

Pathology Intraoperative iatrogenic perforation

Yes/No 0/333 (0)

Positive lymph nodes No. 8/333 (2.4)

Positive radial margin Yes/No 17/333 (5.1)

Positive proximal margin Yes/No 6/333 (1.8)

Positive distal margin Yes/No 6/333 (1.8)

Distance from tumor to distal margin

Centimeters 35/333 (10.5)

Distance from tumor to radial margin

Millimeters 143/333 (42.9)

Surgery Inter-sphincteric dissection

Yes/No 141/333 (42.3)

Visible residual tumor, intraoperative

Yes/No 3/353 (0.8)

Metastatic cancer, intraoperative

Yes/No 0/333 (0)

Tumor appears to invade other organs, intraoperative

Yes/No 0/333 (0)

Leak test Negative leak testPositive leak testNo leak test

0/241 (0)

Positive leak test response

Anastomosis repairedLeak divertedAnastomosis repaired and leak divertedNothing done No documentation

0/14 (0)

Perioperative Number of metastatic lesions, preoperative

No. and site 6/21 (28.6)

■■ TABLE 2 - Rectal cancer quality assessment variables (continued)

ASCO-CRC-2015-V2-INT.indb 12 4/8/2015 4:41:38 PM

Copyright by MT

Page 17: › medical_mx › images › ... · 5 Samantha Hendren; Ellen McKeown; Arden M . Morris; Sandra L . Wong; Mary Oerline; Lyndia Poe; Darrell A . Campbell Jr, and Nancy J . Birkmeyer

Implementation of a Hospital-Based Quality Assessment Program

13

Variables eliminated

Category Variables Response options n missing/total cases

(%)

Preoperative chemotherapy

SystemicRegionalBothNeither

0/353 (0)

Rectal cancer location, site of reference

Anal vergeDentate lineAnorectal ringNone

62/353 (17.6)

Rectal cancer location, distance from anal verge/dentate line/anorectal ring

Centimeters 1/291 (0.3)

Postoperative outcomes

Perineal wound complication

Yes/No 0/79 (0)

Tumor registry data

Date of first contact Date 6/353 (1.7)

Primary site ICD-O-3 topography code 10/353 (2.8)

Grade/differentiation Grade IGrade IIGrade IIIGrade IVCode 9

7/353 (2.0)

Lymph-vascular invasion 0-absent1-present 9-unknown

10/353 (2.8)

Date of surgical and diagnostic staging procedure

Date 46/353 (13.0)

Pathologic stage group 0, 0A, OIS, 1, 1A, 1A1, 1A2, 1B, 1B1, 1B2, 1C, IS, 2, 2A, 2A1, 2A2, 2B, 2C, 3, 3A, 3B, 3C, 3C1, 3C2, 4, 4A, 4A1, 4A2, 4B, 4C, OC, 88, 99

9/353 (2.5)

Pathologic TNM stage Pathologic T stage:Tx, T0, Tis, T1, T2, T3, T4a, T4b

29/353 (8.2)

Pathologic N stage:Nx, N0, N1, N1a, N1b, N1c, N2, N2a, N2b

30/353 (8.5)

Pathologic M stage:Mx, M0, M1, M1a, M1b

159/353 (45.0)

Date of most definitive surgical resection of primary site

Date 10/353 (2.8)

Date radiation started Date 0/353 (0)

Date radiation ended Date 0/353 (0)

MSI 020-stable, 040-unstable low, 050-unstable high, 060-unstable, not stated as low or high, 999-no mention of MSI in record, 988-not a required variable

288/353 (81.6)

AJCC, American Joint Committee on Cancer; CEA, carcinoembryonic antigen; EEA, end-to-end anastomosis; GIA, GI anastomosis; ICD, International Classification of Diseases; MSI, microsatellite instability; WOCN, wound ostomy and continence nurse.

■■ TABLE 2 - Rectal cancer quality assessment variables (continued)

ASCO-CRC-2015-V2-INT.indb 13 4/8/2015 4:41:38 PM

Copyright by MT

Page 18: › medical_mx › images › ... · 5 Samantha Hendren; Ellen McKeown; Arden M . Morris; Sandra L . Wong; Mary Oerline; Lyndia Poe; Darrell A . Campbell Jr, and Nancy J . Birkmeyer

Syllabus

14

practice, for example, rectal cancer location measurements. Among cases with a docu-mented measurement, 257 (88.3%) used the anal verge as the point of reference, whereas 30 (10.3%) used the dentate line, and four (1.4%) used the anorectal ring.

■■ Analysis of data validityWe used two methods to validate the data. For selected variables, we compared data from the tumor registry and the hospital medical records to determine the rate of discordance (or used different parts of the medical record to cross-validate variables). Second, we conducted a survey of the data abstractors (Table 1). Some variables were difficult to abstract in all participating hospitals (eg, mesorectal excision grade), whereas others were difficult to abstract in only a few hospitals.

Table 3 summarizes the cross-validation of selected variables. Low rates of discordance were found for most variables; however, 14% of cases had disagreement between the CPT procedure code and the anasto-mosis variable (for example, a CPT code that includes an anastomosis listed, with no anastomosis type recorded in the “anas-tomosis type” variable). Twelve percent of cases in which radiation therapy was docu-mented in the medical record did not have radiation therapy recorded in tumor registry data. Because outpatient treatments may not be captured reliably in tumor registry data, the medical record data will be used as the source of truth in future abstraction.

■■ Tumor registryA goal of this project was to create a link between each participating hospital’s tumor registry and our chart abstraction data. The abstractors successfully obtained tumor reg-istry data from all 10 hospitals, and reported that all of the tumor registrars were amena-ble to sharing data for quality improvement.

■■ Lessons learnedWe learned important lessons in the course of implementing this program in 10 hospitals. First, when training nurses to

perform this data collection, it is important to involve clinical specialists for question-and-answer sessions, which the nurses found extremely helpful. Second, we found both positive and negative aspects of hospital-based data. The hospital medical record contains detailed pathology, surgi-cal, and perioperative information. How-ever, pathology and operative reports were found to be confusing, because of variation in terminology and in the level of detail included, which varied among providers. We were able to overcome these limitations with extra training, and diagrams were particularly helpful. Also, hospital medical records inconsistently reported data for studies performed in the outpatient setting (eg, CEA, transrectal ultrasound).

■■ Final variable listThe analyses above allowed us to establish a final list of variables that will be used for future data collection across the state of Michigan (Table 2). This includes 20 vari-ables to be abstracted from the record (to supplement the MSQC’s usual variables), plus 11 tumor registry variables. The pro-cess for selecting the final variable list was a three-step one. The first and most important consideration was whether or not the variable was essential to our quality assessment goals (ie, the quality measures listed in the Methods section). Variables essential to these measurement goals were given highest priority, even when there were missing data. The sec-ond consideration was whether the vari-able was accurate. The primary source of information for this was the feedback we received from the data abstractors and expert reviews of cases during initial train-ing and later data collection. Third, the final variable list had to be parsimonious in order to be feasible. Thus, any variables that were not essential to the planned quality measures and were either time consuming, redundant, or frequently miss-ing were eliminated. Elimina tion of redun-dant variables was considered acceptable, after validating them during the current analysis (Table 3).

Key points

•We used two methods to validate the data. For selected variables, we compared data from the tumor registry and the hospital medical records to determine the rate of discordance.

•Some variables were difficult to abstract in all participating hospitals (eg, mesorectal excision grade), whereas others were difficult to abstract in only a few hospitals.

•A goal of this project was to create a link between each participating hospital’s tumor registry and our chart abstraction data.

•The abstractors successfully obtained tumor registry data from all 10 hospitals, and reported that all of the tumor registrars were amenable to sharing data for quality improvement.

•We learned important lessons in the course of implementing this program in 10 hospitals.

•The hospital medical record contains detailed pathology, surgical, and perioperative information.

•We were able to overcome limitations with extra training, and diagrams were particularly helpful.

•The first and most important consideration was whether or not the variable was essential to our quality assessment goals.

•Variables essential to these measurement goals were given highest priority, even when there were missing data. The second consideration was whether the variable was accurate.

ASCO-CRC-2015-V2-INT.indb 14 4/8/2015 4:41:38 PM

Copyright by MT

Page 19: › medical_mx › images › ... · 5 Samantha Hendren; Ellen McKeown; Arden M . Morris; Sandra L . Wong; Mary Oerline; Lyndia Poe; Darrell A . Campbell Jr, and Nancy J . Birkmeyer

Implementation of a Hospital-Based Quality Assessment Program

15

■■ TABLE 3 - Validation of selected variables

Variable(s) Data source(s) Discordant cases (%) Actions

Primary procedure

CPT codes (primary, secondary and concurrent)

10/353 (2.8%) (primary procedure coded as secondary procedure)

Include both primary and secondary codes in eligibility

Anastomosis vs. ostomy

CPT codesOperative report

49/353 (13.9%)(CPT code and anastomosis discordant)

8/353 (2.3%)(CPT and ostomy discordant)

6/353 (1.7%)(Anastomosis and ostomy discordant)

Augmented reviewer trainingOperative report established

as benchmark

Date of surgery

Tumor registry MSQC database

28/343 (8.2%) Designated MSQC as benchmark

Clinical stage

Tumor registry clinical stage

Preoperative clinical notes

Radiology reports

13 (3.7%) Established rule for staging using radiology reports and clinical notes as benchmark

For cases missing clinical data, developed algorithm for use of tumor registry variables

Pathologic stage

Tumor registryPathology reportsRadiology reports

22 (6.2%) Established rule for using pathology and radiology reports as benchmark

Preoperative radiation

Tumor registryClinical notes

0/154 (0%) (no radiation)24/199 (12.1%) (radiation given)

Established clinical notes as benchmark

CPT, Current Procedural Terminology; MSQC, Michigan Surgical Quality Collaborative.

■■ Discussion

In September 2013, the Institute of Medi-cine issued a new report, Delivering High Quality Cancer Care: Charting a New Course for a System in Crisis. In this report, a key recommendation for quality improve-ment in cancer care is development of better systems for measuring the quality of care.19 The project described herein began with the goal of designing and implement-ing a data platform for measuring the quality of care for RC in community and academic hospitals. Working with an exist-ing surgical quality improvement organi-zation, we were able to collect RC data from the hospital medical record; link that information to tumor registry data; and overcome limitations of existing databases, which have inadequate detail regarding sur-gical quality.

The program described here will allow for the measurement of several process and outcome quality measures. These include

lymph node procurement, margin positivity rate, performance of mesorectal excision, grading of mesorectal excision, sphincter preservation rate among eligible cases, use of neoadjuvant chemoradio-therapy for clinical stage II/III cases, ostomy nurse care, anastomotic leak rate, recurrence rate, and mortality. Although audit and feedback alone can improves physician practice, the effect size is generally small.20 In the future, we plan to supplement audit and normative feedback of these quality measures with other efforts within the MSQC organization. Activities such as face-to-face meetings of surgeons, hospital site visits, and sharing of educational and technical resources have been conducted in the MSQC to address such problems as postoperative infection.16,18 Similar strategies—that may only be possible in a regional setting in which providers know and trust one another—will eventually be pursued to improve RC surgery in Michigan.

How does this project fit into the larger landscape of cancer and of surgical quality

Key points

•The project described herein began with the goal of designing and implementing a data platform for measuring the quality of care for RC in community and academic hospitals.

•We plan to supplement audit and normative feedback of these quality measures with other efforts within the MSQC organization.

•Activities such as face-to-face meetings of surgeons, hospital site visits, and sharing of educational and technical resources have been conducted in the MSQC to address such problems as postoperative infection.

ASCO-CRC-2015-V2-INT.indb 15 4/8/2015 4:41:38 PM

Copyright by MT

Page 20: › medical_mx › images › ... · 5 Samantha Hendren; Ellen McKeown; Arden M . Morris; Sandra L . Wong; Mary Oerline; Lyndia Poe; Darrell A . Campbell Jr, and Nancy J . Birkmeyer

Syllabus

16

assessment programs in the United States? On a national scale, multiple organizations including ASCO, the American College of Surgeons’ Commission on Cancer (ACoS CoC), NCCN, and NQF have worked sepa-rately and together to specify quality meas-ures for cancer care and to promote quality through various mechanisms.21 Examples of these efforts include ASCO’s QOPI reg-istry, which is an audit and feedback sys-tem based in medical oncology practices; the NCCN Outcomes Database, which is based in a limited number of large cancer centers across the US; and the accredita-tion requirements of the ACoS CoC, which requires large and small cancer centers to perform local quality assessment and improvement efforts.22 However, none of these programs has attempted to measure surgical quality for RC.

As discussed above, the surgery-focused, national ACS-NSQIP program performs audit and feedback focused on 30-day complications after surgery. However, the ACS-NSQIP database does not contain any specific variables for RC, as described above. The lack of any national program that collects data sufficient to provide meaningful feedback to RC surgeons was a major motivation for this project. On the regional level, a program similar to the current project has been established in 11 medical oncology practices in Florida, and measures some of same indicators includ-ing the quality of pathology reporting for RC.23 However, surgical detail is again absent from data collection.23 Thus, this project is unique in the United States.

We learned lessons in this project that oth-ers may find helpful in designing hospital-based quality assessment programs. First, diagnostic testing performed in the out-patient setting may not be available in the

hospital record. A solution to this problem is to work with the hospitals to improve documentation. For example, we have proposed a template for surgeons in to use when dictating operative reports that includes key information such as clinical stage and location. Second, variable defini-tions must be standardized and extremely clear. In training, we found that consul-tation with a specialist and the use of photos and diagrams were helpful. Finally, enriching hospital medical record data with tumor registry data obtained from local tumor registrars was found to be feasible.

While we are working toward scalable quality improvement, our study may be limited in its generalizability by the unique setting of the Michigan Surgical Quality Collaborative. However, we are encour-aged by the existence of other regional and national organizations; a program such as this might be implemented in other infra-structures. Another potential limitation is the volunteer bias that might have made the data abstraction nurses who partici-pated in this project more likely to succeed than data abstraction personnel at other hospitals. In fact, the participating hospitals tended to have medium-to-large colorectal surgery volumes, and to be long-standing members of the MSQC. However, the work of these hospitals has allowed us to understand how to conduct training, and to streamline and clarify the variables; we anticipate this will allow for implementa-tion statewide.

In conclusion, we report the success-ful implementation of a program for RC care quality assessment in 10 Michigan hospitals. This data platform and training program can serve as a template for other regional organizations that are interested in assessing the quality of care for RC.

Key points

•On a national scale, multiple organizations including ASCO, the American College of Surgeons’ Commission on Cancer (ACoS CoC), NCCN, and NQF have worked separately and together to specify quality measures for cancer care and to promote quality through various mechanisms.

•We learned lessons in this project that others may find helpful in designing hospital-based quality assessment programs.

•First, diagnostic testing performed in the outpatient setting may not be available in the hospital record.

•A solution to this problem is to work with the hospitals to improve documentation.

•Second, variable definitions must be standardized and extremely clear.

•Finally, enriching hospital medical record data with tumor registry data obtained from local tumor registrars was found to be feasible.

•Our study may be limited in its generalizability by the unique setting of the Michigan Surgical Quality Collaborative.

ASCO-CRC-2015-V2-INT.indb 16 4/8/2015 4:41:38 PM

Copyright by MT

Page 21: › medical_mx › images › ... · 5 Samantha Hendren; Ellen McKeown; Arden M . Morris; Sandra L . Wong; Mary Oerline; Lyndia Poe; Darrell A . Campbell Jr, and Nancy J . Birkmeyer

Implementation of a Hospital-Based Quality Assessment Program

17

■■ References 1. van Gijn W, Marijnen CA, Nagtegaal ID, et al: Preoperative

radiotherapy combined with total mesorectal excision for resectable rectal cancer: 12-year follow-up of the multicentre, randomised controlled TME trial. Lancet Oncol 12:575–582, 2011.

2. Heald RJ, Ryall RD: Recurrence and survival after total mesorectal excision for rectal cancer. Lancet 1:1479–1482, 1986.

3. Quirke P, Steele R, Monson J, et al: Effect of the plane of surgery achieved on local recurrence in patients with operable rectal cancer: A prospective study using data from the MRC CR07 and NCIC-CTG CO16 randomised clinical trial. Lancet 373:821–828, 2009.

4. Kapiteijn E, Marijnen CA, Nagtegaal ID, et al: Preoperative radiotherapy combined with total mesorectal excision for resectable rectal cancer. N Engl J Med 345:638–646, 2001.

5. Sauer R, Liersch T, Merkel S, et al: Preoperative versus postoperative chemoradiotherapy for locally advanced rectal cancer: Results of the German CAO/ ARO/AIO-94 randomized phase III trial after a median follow-up of 11 years. J Clin Oncol 30:1926–1933, 2012.

6. Wibe A, Møller B, Norstein J, et al: A national strategic change in treatment policy for rectal cancer–implementation of total mesorectal excision as routine treatment in Norway. A national audit. Dis Colon Rectum 45:857–866, 2002.

7. van Gijn W, van de Velde CJ: 2010SSO John Wayne clinical research lecture: Rectal cancer outcome improvements in Europe: Population-based outcome registrations will conquer the world. Ann Surg Oncol 18:691–696, 2010.

8. Lawson EH, Louie R, Zingmond DS, et al: A comparison of clinical registry versus administrative claims data for reporting of 30-day surgical complications. Ann Surg 256:973–981.

9. Campbell DA Jr, Henderson WG, Englesbe MJ, et al: Surgical site infection prevention: The importance of operative duration and blood transfusion–results of the first American College of Surgeons-National Surgical Quality Improvement Program Best Practices Initiative. J Am Coll Surg 207:810–820, 2008.

10. Ingraham AM, Richards KE, Hall BL, et al: Quality improvement in surgery: The American College of Surgeons National Surgical Quality Improvement Program approach. Adv Surg 44:251–267, 2010.

11. Merkow RP, Bentrem DJ, Mulcahy MF, et al: Effect of postoperative complications on adjuvant chemotherapy use for stage III colon cancer. Ann Surg 258:847–853, 2013.

12. Merkow RP, Bentrem DJ, Cohen ME, et al: Effect of cancer surgery complexity on short-term outcomes, risk predictions, and hospital comparisons. J Am Coll Surg 217:685–693, 2013.

13. Merkow RP, Bentrem DJ, Winchester DP, et al: Effect of including cancer-specific variables on risk-adjusted hospital surgical quality comparisons. Ann Surg Oncol 20:1766–1773, 2013.

14. Merkow RP, Kmiecik TE, Bentrem DJ, et al: Effect of including cancer-specific variables on models examining short-term outcomes. Cancer 119:1412–1419, 2013.

15. Share DA, Campbell DA, Birkmeyer N, et al: How a regional collaborative of hospitals and physicians in Michigan cut costs and improved the quality of care. Health Aff (Millwood) 30:636–645, 2011.

16. Englesbe MJ, Dimick JB, Sonnenday CJ, et al: The Michigan surgical quality collaborative: Will a statewide quality improvement initiative pay for itself? Ann Surg 246: 1100–1103, 2007.

17. Campbell DA Jr, Kubus JJ, Henke PK, et al: The Michigan Surgical Quality Collaborative: A legacy of Shukri Khuri. Am J Surg 198:S49–S55, 2009.

18. Hendren S, Fritze D, Banerjee M, et al: Antibiotic choice is independently associated with risk of surgical site infection after colectomy: A population-based cohort study. Ann Surg 257:469–475, 2013.

19. Institute of Medicine: Delivering High-Quality Cancer Care: Charting a New Course for a System in Crisis. Washington, DC, National Academies Press, 2013.

20. Ivers N, Jamtvedt G, Flottorp S, et al: Audit and feedback: Effects on professional practice and healthcare outcomes. Cochrane Database Syst Rev 6:CD000259, 2012.

21. DeMartino JK: Measuring quality in oncology: Challenges and opportunities. J Natl Compr Canc Netw 11:1482–1491, 2013.

22. Romanus D, Weiser MR, Skibber JM, et al: Concordance with NCCN colorectal cancer guidelines and ASCO/NCCN quality measures: An NCCN institutional analysis. J Natl Compr Canc Netw 7:895–904, 2009.

23. Siegel EM, Jacobsen PB, Malafa M, et al: Evaluating the quality of colorectal cancer care in the state of Florida: Results from the Florida Initiative for Quality Cancer Care. J Oncol Pract 8:239–245, 2012

ASCO-CRC-2015-V2-INT.indb 17 4/8/2015 4:41:38 PM

Copyright by MT

Page 22: › medical_mx › images › ... · 5 Samantha Hendren; Ellen McKeown; Arden M . Morris; Sandra L . Wong; Mary Oerline; Lyndia Poe; Darrell A . Campbell Jr, and Nancy J . Birkmeyer

ASCO-CRC-2015-V2-INT.indb 18 4/8/2015 4:41:38 PM

Copyright by MT

Page 23: › medical_mx › images › ... · 5 Samantha Hendren; Ellen McKeown; Arden M . Morris; Sandra L . Wong; Mary Oerline; Lyndia Poe; Darrell A . Campbell Jr, and Nancy J . Birkmeyer

19

■■ Nadine J . McCleary; Efrat Dotan, and Ilene Browner

Refining the Chemotherapy Approach for Older Patients With Colon Cancer(J Clin Oncol 2014;32(24):2570–2580.)

Abstract: Population studies support an increased incidence of most cancers among older adults . Colorectal cancer has high prevalence in the aging population, with a median age of 69 years at diagnosis and 74 years at death . The vast majority of patients with colon cancer (CC) will require chemotherapy treatments during their disease course, challenging oncologists with the task of tailoring therapy for older patients with CC in the face of limited evidence-based data to guide them . Factors such as comorbidity, performance status, cognitive function, and social support may affect decision making and complicate tolerance of any recommended therapy . In recent years, atten-tion to the specific needs of the aging population with cancer has given rise to the field of geriatric oncology in general, and has generated an increasing fund of knowledge on which to base chemotherapy delivery for this specific population of patients with CC . This article will review the available data specifically for chemotherapy management of older patients with CC in the postoperative and metastatic settings .

■■ Introduction

Colon cancer (CC) is a disease occurring in older adults, with a median age of 69 years at diagnosis and highest incidence among patients age 75 to 84 years1 (Figure 1). However, older patients, particularly those with a higher number of comorbidities, are less likely to be referred to a medical oncologist after diagnosis of CC, and once referred, they have a lower likelihood of receiving chemotherapy.2 Patient-related factors such as functional status, burden of comorbidities, and limited social sup-port affect the ability to deliver treatment and thus have a direct effect on the sur-vival of older adults.3 An early analysis of chemotherapy receipt among patients with stage III CC in the SEER-Medicare data set showed that only 55% of the 6,956 patients received adjuvant chemotherapy within 3 months of surgical resection, reflecting a precipitous decline in receipt of chemotherapy with advancing age, from 78% for those age 65 years to 69% to 11% for those age 85 to 89 years.4

A subsequent updated SEER-Medicare analysis, including three other population-based data sets, conducted by Sanoff et al5 noted little improvement in rates of chemo-therapy receipt, with only 44% of the 5,491 patients evaluated receiving adju-vant chemotherapy. As with prior studies, increasing age and number of comorbidi-ties were associated with lower rates of chemotherapy administration. Decreased chemotherapy use among older adults with CC is also seen in the metastatic set-ting. A population-based study by Ho et al6 reported that < 50% of older patients with metastatic CC received systemic palliative chemotherapy. Although the variability in chemotherapy referral and receipt may be appropriate for some older patients with CC, it remains unclear how providers arrive at these decisions and the degree to which geriatric factors and patients’ wishes contribute to these decisions. Referral and treatment decisions should be driven by goals of treatment, which for older patients with competing causes of death must extend beyond survival to include gains in

Key points

•Colon cancer (CC) is a disease occurring in older adults, with a median age of 69 years at diagnosis and highest incidence among patients age 75 to 84 years.

•Older patients, particularly those with a higher number of comorbidities, are less likely to be referred to a medical oncologist after diagnosis of CC, and once referred, they have a lower likelihood of receiving chemotherapy.

•Patient-related factors such as functional status, burden of comorbidities, and limited social sup port affect the ability to deliver treatment and thus have a direct effect on the sur-vival of older adults.

ASCO-CRC-2015-V2-INT.indb 19 4/8/2015 4:41:38 PM

Copyright by MT

Page 24: › medical_mx › images › ... · 5 Samantha Hendren; Ellen McKeown; Arden M . Morris; Sandra L . Wong; Mary Oerline; Lyndia Poe; Darrell A . Campbell Jr, and Nancy J . Birkmeyer

Syllabus

20

quality of life (QOL), symptom control, and preserved functional status. The limited prospective data to guide oncologists in the management of older patients with CC seen in clinical practice further hin-der the ability to deliver evidence-based care. To optimize clinical outcomes for all patients, additional data are needed to determine which older patients will derive maximal disease and holistic benefits from treatment for CC.

■■ Geriatric assessment

The population is aging, with an average life expectancy of 78 years for men and 81 years for women in the United States.7 Life expectancy considerations carry sig-nificant weight, especially in the setting of metastatic colorectal cancer, where the estimated life expectancy from diagnosis is in the range of 2 years. Older patients with cancer often have competing issues that affect life expectancy and cancer outcomes. How to best define who is considered old is an active area of research and should not be based solely on chronologic age but rather on a composite of physiologic and functional changes related to the aging process as well as other factors such as social support. These age-associated

changes penetrate with significant vari-ability among this population and influ-ence global health status and tolerance and benefit of cancer therapies. Comorbid conditions have an impact on life expec-tancy and a direct effect on a patient’s ability to tolerate therapy (Table 1). Gross et al23 reported a median of four comorbid conditions among patients with cancer, with direct correlation between type, number, and severity of comorbidities and 5-year survival rate. Physiologic changes in organ function that accompany the aging process are often underestimated by assessment of performance status alone. Decrements in renal and hepatic func-tion, altered GI motility, loss of cardiac and marrow reserves, changes in cognition, and decrements in bone and muscle mass may increase the risk for treatment-related toxicity. A retrospective database review of newly diagnosed older patients with CC found that comorbid conditions, geriatric factors, and functional deficits were associ-ated with nonreceipt of standard-of-care therapies and an increased risk for disease-specific and overall mortality.24 In older patients receiving first-line chemotherapy for cancer, including CC, pretreatment deficits in instrumental activities of daily living (IADLs) and depression predicted for a decline in functional status (odds ratio,

Key points

•Life expectancy considerations carry significant weight, especially in the setting of metastatic colorectal cancer, where the estimated life expectancy from diagnosis is in the range of 2 years.

•Older patients with cancer often have competing issues that affect life expectancy and cancer outcomes.

•Comorbid conditions have an impact on life expectancy and a direct effect on a patient’s ability to tolerate therapy.

•Decrements in renal and hepatic function, altered GI motility, loss of cardiac and marrow reserves, changes in cognition, and decrements in bone and muscle mass may increase the risk for treatment-related toxicity.

FIGURE 1 ■■■SEER data on (A) incidence of new colorectal cancer cases and (B) colorectal cancer–related deaths. Highest rates of new cases and deaths among those age 75 to 84 years; median age at diagnosis, 69 years; median age at death, 74 years.

Data adapted.1

ASCO-CRC-2015-V2-INT.indb 20 4/8/2015 4:41:39 PM

Copyright by MT

Page 25: › medical_mx › images › ... · 5 Samantha Hendren; Ellen McKeown; Arden M . Morris; Sandra L . Wong; Mary Oerline; Lyndia Poe; Darrell A . Campbell Jr, and Nancy J . Birkmeyer

Refining the Chemotherapy Approach for Older Patients With Colon Cancer

21

Key points

•Comprehensive geriatric assessment (CGA) and geriatric screening tools are used to assess life expectancy, appraise global health status, and detect geriatric syndromes.

•Recently, several elderly specific models for predict ing chemotherapy-associated toxicity, using CGA factors, have been developed, and additional validation studies are in progress.

2.87; 95% CI, 1.06 to 7.79; P = .04).25 In the Fédération Francophone de Cancé-rologie Digestive (FFCD) 2001–02 study of palliative fluoro-pyrimidine with or without irinotecan, baseline decrements in cognitive function and IADLs predicted for grade 3 to 4 toxicity and risk for hospitalization.26

Comprehensive geriatric assessment (CGA) and geriatric screening tools are used to assess life expectancy, appraise global health status, and detect geriatric syndromes. Recently, several elderly specific models for predicting chemotherapy-associated toxicity, using CGA factors, have been developed, and additional validation studies are in

progress.27,28 The goal of these pretreatment assessments is to preserve independence, aid in treatment planning, and predict for treat-ment-related toxicity,.29 In a study by Kenis et al,30 70% of 967 patients age > 70 years had an abnormal screening assessment at diagnosis or disease progression. Subsequent CGA unmasked geriatric issues in 51% of participants. These observations resulted in planned interventions and changes in cancer-directed therapy in 25% of these patients. Similar findings were documented in a trial of 937 patients age ≥ 70 years, the majority of whom (72%) had an Eastern Cooperative Oncology Group performance status ≤ 1. CGA detected 73% of patients to be at risk

■■ TABLE 1 - Drug-specific considerations among older adults with colon cancer

Drug Toxicity of interest Specific considerations in older adults

FU Mucositis, diarrhea, cytopenias, cardiotoxicity

Mild increase in toxicity rates seen among younger and older patients treated with single-agent FU8,9

Better tolerance of infusional vs. bolus treatment regimens10,11

Capecitabine Kidney Renal excretion of drug requires dose adjustment based on creatinine clearance < 50 mL/min12

Mucositis, diarrhea, cytopenias, hand-foot syndrome, drug interaction with warfarin

Higher rates of GI toxicity in older patients compared with FU have been reported13

QOL No significant improvement in QOL compared with infusional FU with capecitabine was seen in older patients13

Adherence Use of oral medication carries risk of poor adherence14

Oxaliplatin Kidney Renal excretion requires dose adjustment based on creatinine clearance < 30 mL/min15

Cytopenias Slightly higher rates of cytopenias noted with FOLFOX regimen among older patients16

Neuropathy No significant difference in rate of neuropathy between younger and older patients;16 neuropathy may contribute to functional limitations and increase in fall risk among older patients

Irinotecan GI toxicity No significant difference in rates of GI toxicity between younger and older patients;17 diarrhea may increase risk of dehydration among older patients

Bevacuzimab Thromboembolic events Higher risk of arterial thrombombolic events among older patients18–21

Cetuximab/ panitumumab

GI toxicity Similar toxicity pattern was seen between younger and older patients;22 diarrhea may increase risk of dehydration among older patients

AflibriceptRegorafenib

– Older adult–specific data are lacking

– Older adult–specific data are lacking

Adherence Use of oral medication carries risk of poor adherence14

FOLFOX, infusional fluorouracil, leucovorin, and oxaliplatin; FU, fluorouracil; QOL, quality of life.

ASCO-CRC-2015-V2-INT.indb 21 4/8/2015 4:41:39 PM

Copyright by MT

Page 26: › medical_mx › images › ... · 5 Samantha Hendren; Ellen McKeown; Arden M . Morris; Sandra L . Wong; Mary Oerline; Lyndia Poe; Darrell A . Campbell Jr, and Nancy J . Birkmeyer

Syllabus

22

(57% with IADL and 51% with ADL defi-cits).31 Among at risk and fit patients, 54.7% and 33.9%, respectively, did not receive standard therapy for their cancer.

Despite the evolving data on CGA and patient fitness for therapy, data regarding chemotherapy selection in this population is limited because of underenrollment of older patients onto clinical therapeutic trials. A retrospective review of registra-tion trials revealed that for each half decade > 65 years of age, there was a precipitous fall in older participants, despite older patients comprising the majority of those with cancer.32 Without participa-tion of older patients with comorbidities, geriatric risk factors, and poorer functional status, there is inadequate information on which to base therapeutic combinations, drug dosing, and toxicity profiles, and results do not necessarily reflect the gen-eral population of patients with CC seen in clinical practice, making development of concise treatment algorithms for older adults with CC challenging. To increase trial participation, changes to trial design should be considered, including: relaxing inclusion criteria with respect to organ function and functional status, facilitating solutions to logistic issues such as visit frequency, and improving communication to accommo-date for differences in sensory, organ, and cognitive function.33 In addition, standard trial end points such as overall survival (OS), disease-free survival (DFS), progression-free survival (PFS), and response rate (RR) may not adequately define the risks and benefits of chemotherapy in older adults because of competing mortality risks.34 Other end points, such as QOL, preserved functional and physiologic status, and disease-specific mortality, may better inform difficult treat-ment decisions.

■■ Considerations in treatment of early-stage CC in older patients

Efficacy of intravenous fluorouracil/leucovorin (FU/LV) in the adjuvant setting

among older patients with CC was estab-lished in 2001. A pooled analysis of 3,351 patients from seven phase III randomized trials showed a 29% reduction in risk of death at 5 years.8 Age did not seem to affect this survival benefit (interaction P = .61 for OS). Furthermore, older patients did not experience higher rates of grade ≥ 3 treatment-related toxicity with this adjuvant therapy. Subgroup analysis of 397 patients age ≥ 70 years treated with capecitabine in the adjuvant setting found this oral fluoropyrimidine to be equally effective as FU/LV.35 Predefined grade ≥ 3 treatment-related toxicities were reduced and delayed with capecitabine in com-parison with intravenous FU/LV, with no apparent difference by age (< 65 vs. ≥ 65 years).35,36

Considerable debate regarding the additive benefit of oxaliplatin in the adjuvant setting for older patients continues. Oxaliplatin is associated with an improvement in DFS (20% reduction in risk of CC recurrence) and OS (16% reduction in risk of death) at 6 years, as shown in the MOSAIC (Mul-ticenter International Study of Oxaliplatin/FU/LV in the Adjuvant Treatment of CC) study.37 However, this survival advantage does not extend to all subpopulations evaluated. A subpopulation analysis of 315 patients age 70 to 75 years enrolled onto the MOSAIC trial (N = 2,246; age 18 to 75 years) demonstrated that age did not confound the impact of treatment on sur-vival outcome (interaction P = .42 and .18 for DFS and OS, respectively [not statisti-cally significant]). Although the previously observed survival benefit of fluoropyrimi-dine persisted in the older subgroup (DFS: hazard ratio [HR], 0.78; 95% CI, 0.66 to 0.92; OS: HR, 0.80; 95% CI, 0.66 to 0.97), there was no survival advantage with the addition of oxaliplatin to fluoropyrimidine chemotherapy (DFS: HR, 1.06; 95% CI, 0.80 to 1.42; OS: HR, 1.27; 95% CI, 0.93 to 1.74).38 Another prospective study with subpopulation analysis of survival outcomes in older patients with CC is the NO16968, or XELOXA (Adjuvant Treatment

Key points

•Efficacy of intravenous fluorouracil/leucovorin (FU/LV) in the adjuvant setting among older patients with CC was established in 2001.

•A pooled analysis of 3,351 patients from seven phase III randomized trials showed a 29% reduction in risk of death at 5 years.

• Age did not seem to affect this survival benefit (interaction P = .61 for OS).

•Subgroup analysis of 397 patients age ≥ 70 years treated with capecitabine in the adjuvant setting found this oral fluoropyrimidine to be equally effective as FU/LV.

•Considerable debate regarding the additive benefit of oxaliplatin in the adjuvant setting for older patients continues.

•A subpopulation analysis of 315 patients age 70 to 75 years enrolled onto the MOSAIC trial (N = 2,246; age 18 to 75 years) demonstrated that age did not confound the impact of treatment on survival outcome.

•There was no survival advantage with the addition of oxaliplatin to fluoropyrimidine chemotherapy.

•Study demonstrated an improved DFS for patients treated with oxaliplatin and capecitabine as compared with bolus FU/LV, without any significant difference by age.

ASCO-CRC-2015-V2-INT.indb 22 4/8/2015 4:41:39 PM

Copyright by MT

Page 27: › medical_mx › images › ... · 5 Samantha Hendren; Ellen McKeown; Arden M . Morris; Sandra L . Wong; Mary Oerline; Lyndia Poe; Darrell A . Campbell Jr, and Nancy J . Birkmeyer

Refining the Chemotherapy Approach for Older Patients With Colon Cancer

23

With Capecitabine and Oxaliplatin), trial. Although OS data are maturing, the study demonstrated an improved DFS for patients treated with oxaliplatin and capecitabine as compared with bolus FU/LV, without any significant difference by age.39 The study supported noninferiority of oral fluoropyrimidine-based therapy in younger patients; however, it included a bolus FU/LV control arm that is no longer the standard of care. As with the MOSAIC study, the subset of older patients was lim-ited to those ≤ 75 years of age, making it difficult to extrapolate the findings to the general population of older patients with resected CC. An updated pooled analysis of the ACCENT (Adjuvant CC End Points) database (including seven randomized tri-als of newer adjuvant chemotherapy, such as MOSAIC, NSABP [National Surgical Adjuvant Breast and Bowel Project] C-07, and NO16968) failed to show a DFS or OS benefit among older patients receiving oxaliplatin-based adjuvant chemotherapy (DFS: HR, 0.94; 95% CI, 0.78 to 1.13; OS: HR, 1.04; 95% CI, 0.85 to 1.27).40

Sanoff et al5 updated a SEER-Medicare population-based cohort analysis of receipt and effectiveness of adjuvant chemother-apy by age for CC.4 Evaluation of four data sets (SEER-Medicare, National Comprehen-sive Cancer Network [NCCN], New York State Cancer Registry [NYSCR], and Cancer Care Outcomes Research and Surveillance Consortium) showed a trend toward a marginal survival benefit in two of three cohorts with available data on oxaliplatin use in patients age > 75 years.5 This result may be a function of differences in patients by data set, because the NCCN is associ-ated with tertiary care centers in contrast with the general older population included in SEER-Medicare and NYSCR data sets. The lack of data regarding receipt and type of second-line therapy administered and cause of death, along with the large num-ber of older patients who did not receive any chemotherapy, limits the ability to make conclusive treatment decisions based on this data alone.

Although retrospective observational studies in the general older population support the benefit of fluoropyrimidine chemotherapy, they fail to show a compa-rable survival benefit with the addition of oxaliplatin.5,41 Reasons offered for lack of extension of the OS benefit of oxaliplatin observed in younger as compared with older patients in the postoperative setting include increased rate of death resulting from other causes and death after CC relapse.38 Abrams et al42 have also shown that receipt of oxaliplatin is inversely related to advancing age, and older patients are more likely to prematurely discontinue this therapy. The addition of oxaliplatin to fluoropyrimidine adjuvant therapy in some older patients may also exhaust their physiologic reserves, compete with comor-bid illness, and induce or unmask frailty. Development of chronic oxaliplatin-induced peripheral neuropathy may increase the risk of falls and limit functional status in some older patients with CC, both of which are independent poor prognostic markers in this population.43

To best determine the full impact of oxaliplatin on survival outcomes for older patients, an elderly specific prospective trial in the adjuvant setting is warranted. Such a trial must include long-term patient-related outcome measures of functional status and QOL in addition to clinical efficacy data. An attempt was made within the North Central Cancer Treatment Group (NCCTG), in collaboration with the Cancer and Leu-kemia Group B (CALGB; currently known as ALLIANCE), but was stalled when it met with extensive debate regarding the appro-priate comparison treatment arms and study end points.

Adjuvant fluoropyrimidine treatment for stage II CC could be considered for older patients with high-risk features and per-haps microsatellite-stable disease.44–46 Aparicio et al47 demonstrated an increased prevalence of microsatellite instability (MSI) with increasing patient age. Because MSI is predictive of a poorer response to

Key points

•An updated pooled analysis of the ACCENT (Adjuvant CC End Points) database (including seven randomized trials of newer adjuvant chemotherapy.

•Evaluation of four data sets (SEER-Medicare, National Comprehensive Cancer Network [NCCN], New York State Cancer Registry [NYSCR], and Cancer Care Outcomes Research and Surveillance Consortium) showed a trend toward a marginal survival benefit in two of three cohorts with available data on oxaliplatin use in patients age > 75 years.

•The addition of oxaliplatin to fluoropyrimidine adjuvant therapy in some older patients may also exhaust their physiologic reserves, compete with comorbid illness, and induce or unmask frailty.

•Development of chronic oxaliplatin-induced peripheral neuropathy may increase the risk of falls and limit functional status in some older patients with CC, both of which are independent poor prognostic markers in this population.

•To best determine the full impact of oxaliplatin on survival outcomes for older patients, an elderly specific prospective trial in the adjuvant setting is warranted.

•Adjuvant fluoropyrimidine treatment for stage II CC could be considered for older patients with high-risk features and perhaps microsatellite-stable disease.

ASCO-CRC-2015-V2-INT.indb 23 4/8/2015 4:41:39 PM

Copyright by MT

Page 28: › medical_mx › images › ... · 5 Samantha Hendren; Ellen McKeown; Arden M . Morris; Sandra L . Wong; Mary Oerline; Lyndia Poe; Darrell A . Campbell Jr, and Nancy J . Birkmeyer

Syllabus

24

fluoropyrimidine therapy and prognostic for a decrease in cancer-specific mortality, MSI should be taken into consideration when evaluating older patients for adjuvant chemotherapy, especially those with stage II disease. The QUASAR (Quick and Simple and Reliable) study of adjuvant fluoropy-rimidine for stage II CC did not show an OS or DFS benefit for the subgroup analysis of patients age ≥ 70 years.48 The updated MOSAIC subgroup analysis examining the benefit of the addition of oxaliplatin to intravenous FU/LV in the adjuvant set-ting for stage II disease demonstrated no benefit for standard-risk patients.38 Those with high-risk features, including T4 stage disease, perforation or obstruction at pres-entation, < 10 nodes examined, poorly differentiated tumor, or venous invasion, had a nonstatistically significant trend toward improvement in DFS (HR, 0.72; 95% CI, 0.51 to 1.02) without any OS benefit. There was no further exploration by age, but based on the lack of support-ive data in the stage III setting for older patients and lack of statistically significant benefit in stage II disease, oxaliplatin would not be beneficial, regardless of age group examined.

■■ Challenges in treatment of older patients with metastatic CC

An average of 20% of patients with CC are diagnosed with metastatic disease at presentation, and many more develop metastatic spread after treatment for early-stage disease.49 The management of older patients with metastatic CC must take into account the goals of therapy, which are often palliative in nature, the toxicity profile of the recommended therapy, and the patient’s functional status, comorbidity burden, social support, and goals of care. The benefit of palliative chemotherapy must be weighed against the inherent risks of adverse events and decreased QOL; thus, holistic review is required, and thoughtful approaches to noncurative care are useful.

Publication of elderly specific prospective and retrospective data in the metastatic setting has helped guide these difficult treatment decisions (Table 2). The use of infusional FU/LV or capecit-abine mono-therapy may be an appropriate option in a subset of older patients with metastatic CC. Three phase III studies failed to show a survival benefit from combination chemo-therapy as first-line treatment compared with FU/LV monotherapy.13,53,54 Fol-precht at el10 retrospectively reviewed data from 22 European studies evaluating FU/LV therapy; of the total 3,825 patients, 629 were age > 70 years. Overall RR, PFS, and OS were similar among younger and older patients. Thus, treatment with combination chemotherapy should be carefully consid-ered in older patients with metastatic CC, especially when monotherapy is noninferior and likely to be better tolerated.

Treatment with combination FU and oxali-platin for patients with metastatic CC age > 70 years (n = 614) was evaluated in a post hoc analysis of four large clinical trials (total of 3,742 patients).16,55 Older patients experienced a similar clinical benefit at the cost of increased rates of hematologic toxicity, with similar rates of other toxici-ties, including neurologic and GI, as com-pared with the younger cohort.16 In older patients with metastatic CC, small phase II studies demonstrated good tolerability and efficacy similar to that in younger patients with the use of combined oxaliplatin and capecitabine.56,57 The MRC FOCUS2 (UK Medical Research Council Fluorouracil, Oxaliplatin, and Capecitabine, Use and Sequencing) trial was the first published randomized clinical trial conducted specifi-cally among older and frail patients with untreated metastatic CC.13 Patients were randomly assigned to receive capecit-abine or FU, with or without oxaliplatin, at a 20% dose reduction. Capecitabine and FU showed similar clinical benefit in PFS and OS. Interestingly, capecitabine was associated with an increased rate of adverse events and with lower QOL. The addition of oxaliplatin with a 20% dose

Key points

•MOSAIC subgroup analysis examining the benefit of the addition of oxaliplatin to intravenous FU/LV in the adjuvant setting for stage II disease demonstrated no benefit for standard-risk patients.

•An average of 20% of patients with CC are diagnosed with metastatic disease at presentation, and many more develop metastatic spread after treatment for early-stage disease.

•The benefit of palliative chemotherapy must be weighed against the inherent risks of adverse events and decreased QOL.

•Publication of elderly specific prospective and retrospective data in the metastatic setting has helped guide these difficult treatment decisions.

•Treatment with combination chemotherapy should be carefully considered in older patients with metastatic CC, especially when monotherapy is noninferior and likely to be better tolerated.

•Older patients experienced a similar clinical benefit at the cost of increased rates of hematologic toxicity, with similar rates of other toxicities, including neurologic and GI, as compared with the younger cohort.

•The rate of grade ≥ 3 adverse events was not significantly increased with the addition of oxaliplatin at this reduced dose.

ASCO-CRC-2015-V2-INT.indb 24 4/8/2015 4:41:39 PM

Copyright by MT

Page 29: › medical_mx › images › ... · 5 Samantha Hendren; Ellen McKeown; Arden M . Morris; Sandra L . Wong; Mary Oerline; Lyndia Poe; Darrell A . Campbell Jr, and Nancy J . Birkmeyer

Refining the Chemotherapy Approach for Older Patients With Colon Cancer

25

■■

TAB

LE 2

- S

tud

ies

of

adju

van

t ch

emo

ther

apy

trea

tmen

ts f

or

old

er a

du

lts

wit

h c

olo

n c

ance

r

Stu

dy

Des

ign

Res

ult

sTo

xici

ty

Tour

niga

nd e

t al

38 (2

012)

Age

< 7

0 ye

ars

(n =

1,9

31)

5-ye

ar D

FS5-

year

TTR

6-ye

ar O

S

Age

70

to 7

5 ye

ars

(n =

315

)5-

year

DFS

5-ye

ar T

TR

6-ye

ar O

S

Phas

e III

RC

T M

OSA

IC

subg

roup

ana

lysi

s of

pat

ient

s ag

e 70

to

75

year

s; F

U/L

V

vs. F

OLF

OX

4

No

trea

tmen

t in

tera

ctio

n by

age

for

DFS

(P =

.418

), TT

R

(P =

.719

), or

OS

(P =

.18)

; low

rep

orte

d po

st h

oc p

ower

fo

r in

tera

ctio

n te

sts

of 6

% t

o 35

%; F

OLF

OX

tre

atm

ent

was

as

soci

ated

with

low

er m

edia

n O

S af

ter

dise

ase

recu

rren

ce

(FO

LFO

X4,

3.6

vs.

FU

/LV,

13.

7 m

onth

s), l

ower

rec

eipt

of

subs

eque

nt li

nes

of c

hem

othe

rapy

± r

esec

tion

of m

etas

tatic

di

seas

e (1

6 vs

. 30

patie

nts;

P =

.01)

, and

hig

her

inci

denc

e of

se

cond

can

cers

(17

vs. 1

0 pa

tient

s)

HR,

0.7

8; 9

5% C

l, 0.

66 t

o 0.

92; P

= .0

03H

R, 0

.74;

95%

Cl,

0.62

to

0.88

; P =

.001

HR,

1.1

0; 9

5% C

l, 0.

73 t

o 1.

65; P

= .6

61

69.1

%; 9

5% C

l, 61

.3%

to

75.8

% v

s. 6

5.8%

; 95%

Cl,

57.8

% t

o 72

.7%

; HR,

0.9

3; 9

5% C

l, 0.

64 t

o 1.

35; P

= .7

1078

.8%

; 95%

Cl,

71.2

% t

o 84

.6%

vs.

69.

9%; 9

5% C

l, 61

.9%

to

76.5

%; H

R, 0

.72;

95%

Cl,

0.47

to

1.11

; P =

.14

75.8

%; 9

5% C

l, 0.

73 t

o 1.

65 v

s. 7

6.1%

; 95%

Cl,

68.6

to

82.1

; H

R, 1

.10;

95%

Cl,

0.73

to

1.65

; P =

.661

Serio

us A

Es r

epor

ted

for

30 p

atie

nts

in F

OLF

OX

4 ar

m v

s. 1

5 in

FU

/LV

arm

(P

 = .0

18);

sim

ilar

toxi

city

pro

file

amon

g yo

unge

r an

d ol

der

patie

nts

Hal

ler

et a

l39 (2

011)

Ove

rall

coho

rt (N

= 1

,886

)D

FSRF

SO

S

Phas

e III

RC

T N

0169

68; F

U/F

A v

s.

XEL

OX

Sim

ilar

appe

aran

ce b

y ag

e su

bgro

up a

naly

sis

on f

ores

t pl

ot (<

65

vs. ≥

65

year

s); n

o ch

ange

in D

FS o

r O

S by

age

rep

orte

d

HR,

0.8

0; 9

5% C

l, 0.

69 t

o 0.

93; P

= .0

045

HR,

0.7

8; 9

5% C

l, 0.

67 t

o 0.

92; P

= .0

24H

R, 0

.87;

95%

Cl,

0.72

to

1.05

; P =

.148

6

No

age-

spec

ific

toxi

city

dat

a

Yoth

ers

et a

l50 (2

011)

Age

< 7

0 ye

ars

(n =

2,0

13)

DFS

OS

Age

≥ 7

0 ye

ars

(n =

396

)D

FS

OS

Phas

e III

RC

T N

SABP

C

-07

subg

roup

an

alys

is o

f pa

tient

s ≥

70 y

ears

; FU

/LV

vs

. FLO

X64

.7%

vs.

70.

7%81

.8%

vs.

78.

8%

62%

v62

.8%

; HR,

1.1

7; 9

5% C

l, 0.

94 t

o 1.

46;

P =

.16

(age

< 7

0 ye

ars

as r

efer

ent)

76.3

% v

s. 7

1.6%

; HR,

1.3

2; 9

5% C

l, 1.

03 t

o 1.

70;

P =

.30

(age

< 7

0 ye

ars

as r

efer

ent)

For

patie

nts

age

≥ 70

yea

rs, g

rade

4 t

o 5

toxi

city

: FU

/LV,

13%

vs.

FLO

X, 2

0%; O

R,

1.59

; 95%

Cl,

0.93

to

2.73

; for

pat

ient

s ag

e <

70

year

s, g

rade

4 t

o 5

toxi

city

: FU

/LV

, 9%

vs.

FLO

X, 1

0%; O

R, 1

.13;

95%

C

l, 0.

84 t

o 1.

51; d

ose

mod

ifica

tion

of

oxal

ipla

tin b

y ap

prox

imat

ely

25%

in

patie

nts

age

≥ 70

yea

rs w

ith s

imila

r do

se-in

tens

ity

ASCO-CRC-2015-V2-INT.indb 25 4/8/2015 4:41:39 PM

Copyright by MT

Page 30: › medical_mx › images › ... · 5 Samantha Hendren; Ellen McKeown; Arden M . Morris; Sandra L . Wong; Mary Oerline; Lyndia Poe; Darrell A . Campbell Jr, and Nancy J . Birkmeyer

Syllabus

26

Stu

dy

Des

ign

Res

ult

sTo

xici

ty

Lem

bers

ky e

t al

51 (2

006)

Ove

rall

coho

rt (N

= 1

,551

)5-

year

DFS

5-ye

ar O

S

Age

≥ 6

0 ye

ars

(n =

939

)5-

year

DFS

5-ye

ar O

S

Phas

e III

non

infe

riorit

y st

udy

NSA

BP C

-06;

FU

/LV

vs.

UFT

/LV

DFS

and

OS

adju

sted

for

age

and

No.

of

lym

ph n

odes

; gl

obal

tes

t fo

r in

tera

ctio

n of

tre

atm

ent

with

age

, sex

, and

No.

of

lym

ph n

odes

was

not

sta

tistic

ally

sig

nific

ant

at P

= .7

4 (O

S)

and

0.83

(DFS

)68

.2%

vs.

67%

; HR,

1.0

05; 9

5% C

l, 0.

848

to 1

.192

; P =

.95

78.7

% v

s. 7

8.5%

; sta

ge il

l onl

y: 7

1.5%

vs.

69.

6%; H

R, 1

.010

; 95

% C

l, 0.

822

to 1

.242

; P =

.92

HR,

1.4

1; 9

5% C

l, 1.

18 to

1.6

9; P

= .0

02 (a

ge <

60

year

s as

 refe

rent

)H

R, 1

.40;

95%

Cl,

1.12

to 1

.74;

P =

.03

(age

< 6

0 ye

ars

as re

fere

nt)

No

age-

spec

ific

toxi

city

dat

a

Twel

ves

et a

l52 (2

012)

Ove

rall

coho

rt (N

= 1

,987

)5-

year

DFS

5-ye

ar O

SA

ge ≥

70

year

s (n

= 3

97)

5-ye

ar D

FS5-

year

OS

Phas

e III

non

infe

riorit

y st

udy

X-A

CT,

in

clud

ing

subg

roup

an

alys

is o

f th

ose

age

≥ 70

yea

rs;

bolu

s FU

/LV

vs.

ca

peci

tabi

ne

No

trea

tmen

t in

tera

ctio

n by

age

(DFS

: P =

.50;

OS:

P =

.78)

60.8

% v

s. 5

6.7%

; HR,

0.8

8; 9

5% C

l, 0.

77 t

o 1.

0171

.4%

vs.

68.

4%; H

R, 0

.86;

95%

Cl,

0.74

to

1.01

58.1

% v

s. 5

5.8%

; HR,

0.9

7; 9

5% C

l, 0.

72 t

o 1.

3168

.8%

vs.

65.

0%; H

R, 0

.91;

95%

Cl,

0.65

to

1.26

Low

er d

ose-

inte

nsity

am

ong

olde

r pa

tient

s (c

apec

itabi

ne a

rm: 8

5% a

ge <

70,

74%

ag

e ≥

70 y

ears

; FU

/FA

arm

: 89%

age

< 7

0,

83%

age

≥ 7

0 ye

ars)

; hig

her

rate

of

dose

m

odifi

catio

n (c

apec

itabi

ne a

rm: 5

5% a

ge

< 7

0, 6

5% a

ge ≥

70

year

s; F

U/F

A a

rm:

50%

age

< 7

0, 6

1% a

ge ≥

70

year

s)

McC

lear

y et

al40

(201

3)

Oxa

lipla

tin-b

ased

the

rapy

(a

ge <

70

year

s, n

= 5

,420

; ag

e ≥

70 y

ears

, n =

1,1

19)

DFS

OS

TTR

Ora

l flu

orop

yrim

idin

e (a

ge <

70

year

s, n

= 2

,783

; ag

e ≥

70 y

ears

, n =

757

)D

FS

OS

TTR

Pool

ed a

naly

sis

of

seve

n ad

juva

nt

stud

ies;

ora

l/lV

FV

±

irin

otec

an o

r ox

alip

latin

Diff

eren

ce in

rat

e of

dea

ths

with

in 6

mon

ths

not

stat

istic

ally

si

gnifi

cant

bet

wee

n co

ntro

l and

exp

erim

enta

l arm

s or

by

age

Age

< 7

0 vs

. ≥ 7

0 ye

ars:

HR,

0.7

8; 9

5% C

l, 0.

71 t

o 0.

86 v

s. H

R, 0

.94;

95%

Cl,

0.78

to

1.13

; P =

.09

HR,

0.8

3; 9

5% C

l, 0.

74 t

o 0.

92 v

s. H

R, 1

.04;

95%

Cl,

0.85

to

1.27

; P =

.05

HR,

0.7

7; 9

5% C

l, 0.

69 t

o 0.

85 v

s. H

R, 0

.86;

95%

Cl,

0.69

to

1.06

; P =

.36

Age

< 7

0 vs

. ≥ 7

0 ye

ars:

HR,

0.9

1; 9

5% C

l, 0.

80 t

o 1.

02 v

s. H

R, 0

.92;

95%

Cl,

0.92

to

1.41

; P =

.13

HR,

0.9

0; 9

5% C

l, 0.

79 t

o 1.

03 v

s. H

R, 1

.13;

95%

Cl,

0.90

to

1.41

; P =

.16

HR,

0.9

0; 9

5% C

l, 0.

80 t

o 1.

02 v

s. H

R, 1

.20;

95%

Cl,

0.93

to

1.54

; P =

.09

No

age-

spec

ific

toxi

city

dat

a

Beca

use

of la

ck o

f su

rviv

al b

enef

it fo

r iri

note

can

in a

djuv

ant

sett

ing,

irin

otec

an-b

ased

tria

ls (C

ALG

B [C

ance

r an

d Le

ukem

ia G

roup

B] 8

9803

and

PET

AC

C-3

[Pan

Eur

opea

n Tr

ial A

djuv

ant

Col

on C

ance

r]) a

re n

ot in

clud

ed.

AE,

adv

erse

eve

nt; D

FS, d

isea

se-f

ree

surv

ival

; FA

, fol

inic

aci

d; F

LOX

, bol

us f

luor

oura

cil,

leuc

ovor

in, a

nd o

xalip

latin

; FO

LFO

X, f

luor

oura

cil,

leuc

ovor

in, a

nd o

xalip

latin

; FU

, flu

orou

raci

l; H

R, h

azar

d ra

tio;IV

, int

rave

nous

; LV,

le

ucov

orin

; MO

SAIC

, Mul

ticen

ter

Inte

rnat

iona

l Stu

dy o

f O

xalip

latin

/Flu

orou

raci

l/Leu

covo

rin in

the

Adj

uvan

t Tr

eatm

ent

of C

olon

Can

cer;

NSA

BP, N

atio

nal S

urgi

cal A

djuv

ant

Brea

st a

nd B

owel

Pro

ject

; OR,

odd

s ra

tio; O

S,

over

all s

urvi

val;

RCT,

ran

dom

ized

con

trol

led

tria

l; RF

S, r

elap

se-f

ree

surv

ival

; TTR

, tim

e to

tum

or r

ecur

renc

e; U

FT, t

egaf

ur-u

raci

l; X

-AC

T, X

elod

a in

Adj

uvan

t C

olon

Can

cer

Ther

apy;

XEL

OX

, cap

ecita

bine

and

oxa

lipla

tin.

■■

TAB

LE 2

- S

tud

ies

of

adju

van

t ch

emo

ther

apy

trea

tmen

ts f

or

old

er a

du

lts

wit

h c

olo

n c

ance

r (c

on

tin

ued

)

ASCO-CRC-2015-V2-INT.indb 26 4/8/2015 4:41:39 PM

Copyright by MT

Page 31: › medical_mx › images › ... · 5 Samantha Hendren; Ellen McKeown; Arden M . Morris; Sandra L . Wong; Mary Oerline; Lyndia Poe; Darrell A . Campbell Jr, and Nancy J . Birkmeyer

Refining the Chemotherapy Approach for Older Patients With Colon Cancer

27

reduction to FU or capecitabine resulted in improved RRs (13% vs. 35%; P < .001), a nonstatistically significant trend toward improvement in PFS, and a lack of improvement in OS. The use of a prespeci-fied dose reduction may have contributed to limited clinical benefit in this study. The rate of grade ≥ 3 adverse events was not significantly increased with the addition of oxaliplatin at this reduced dose. Results of this prospective study called into question the benefit of the addition of reduced-dose oxaliplatin to fluoropyrimidine-based therapy in older and frail patients with metastatic CC.

The combination of irinotecan and FU, evaluated in a retrospective analysis of a large phase III clinical trial and small phase II study, demonstrated similar benefit and toxicity profiles among younger and older patients.17,58 Final results are pending from the French phase III FFCD 2001–02 trial, which randomly assigned 282 untreated patients with metastatic CC age > 75 years to FU with or without the addition of irinotecan. A substudy performed geriatric evaluations of approximately half of the accrued patients and demonstrated a 58% rate of grade 3 to 4 toxicity, 54% rate of unexpected hospitalization, and 33% rate of dose reduction. The strongest predictors for toxicity were dependence in IADLs and decreased cognitive function. Data regard-ing the efficacy of this regimen among older patients are eagerly awaited to deter-mine the risk-benefit ratio of this treatment approach.26 Maintaining treatment of older patients with these regimens for prolonged periods of time can be challenging. The OPTIMOX-1 and -2 (Optimized Leucovorin-Fluorouracil Oxaliplatin) treatment sched-ule, alternating combination chemotherapy with FU maintenance or chemotherapy-free intervals, can be useful when treating some older patients, resulting in decreased toxicity and increased patient tolerance of the therapy.59,60 Figer et al60 demonstrated good tolerance of and improved QOL with the OPTIMOX-1 treatment schedule among older patients with metastatic CC.

■■ Use of biologic agents in the older population

Bevacizumab, the vascular endothelial growth factor (VEGF) antibody, is only used in approximately one third of eligible older patients with metastatic CC out of concern for increased toxicity with limited clinical benefit in this patient population.61 The BEAT (Bevacizumab Expanded Access Trial) and BRiTE (Bevacizumab Regimens Investigation of Treatment Effects) stud-ies demonstrated similar clinical outcomes between younger and older patients.18,62 Pooled analyses of phase II and III rand-omized clinical trials reported improved PFS and OS with the addition of bevacizumab to standard chemotherapy in patients age > 6521 and > 70 years of age,63 similar to improvements seen in younger patients. Overall, bevacizumab was well tolerated, but there was an overall increase in arterial thromboembolic events among patients age > 65 years (HR, 2.17; 95% CI, 1.17 to 4.01; P < .01).18–21

Prospective studies evaluating the clinical benefit of combining bevacizumab with FU or capecitabine as first-line therapy for older patients with metastatic CC have shown promising results. A phase II study evaluating treatment with FU/LV with or without bevacizumab among 168 frail patients age > 65 years demonstrated a statistically significant increase in PFS (5.5 vs. 9.2 months; P < .001) and a nonsig-nificant improvement in median OS (12.9 vs. 16.6 months; P = .16).64 The recently published prospective, randomized phase III AVEX (Avastin in Elderly With Xeloda) study, comparing capecitabine alone or in combination with bevacizumab, demon-strated favorable outcomes in the combina-tion arm.65 The study enrolled untreated patients with metastatic CC age > 70 years who were not candidates for oxaliplatin- or irinotecan-based therapy. PFS was longer in the combination arm, and a trend was seen toward improved OS.2 The improvement in outcomes came at the expense of increased toxicity, mainly

Key points

•The combination of irinotecan and FU, evaluated in a retrospective analysis of a large phase III clinical trial and small phase II study, demonstrated similar benefit and toxicity profiles among younger and older patients.

•A substudy performed geriatric evaluations of approximately half of the accrued patients and demonstrated a 58% rate of grade 3 to 4 toxicity, 54% rate of unexpected hospitalization, and 33% rate of dose reduction.

•The OPTIMOX-1 and -2 (Optimized Leucovorin- Fluorouracil Oxaliplatin) treatment schedule, alternating combination chemotherapy with FU maintenance or chemotherapy-free intervals, can be useful when treating some older patients, resulting in decreased toxicity and increased patient tolerance of the therapy.

•Bevacizumab, the vascular endothelial growth factor (VEGF) antibody, is only used in approximately one third of eligible older patients with metastatic CC out of concern for increased toxicity with limited clinical benefit in this patient population.

•Overall, bevacizumab was well tolerated, but there was an overall increase in arterial thromboembolic events among patients age > 65 years.

ASCO-CRC-2015-V2-INT.indb 27 4/8/2015 4:41:40 PM

Copyright by MT

Page 32: › medical_mx › images › ... · 5 Samantha Hendren; Ellen McKeown; Arden M . Morris; Sandra L . Wong; Mary Oerline; Lyndia Poe; Darrell A . Campbell Jr, and Nancy J . Birkmeyer

Syllabus

28

grade 3 hand-foot syndrome and venous thromboembolism. The NCCTG attempted to conduct a prospective study evaluat-ing the benefit of oxaliplatin added to FU/capecitabine and bevacizumab among untreated older patients with metastatic CC (N0949). However, this study was una-ble to accrue patients nationally and was ultimately terminated early. The reasons for poor accrual remain unclear, but this could be related to concern among oncologists about over- or undertreatment of their older patients or perceived limited benefit of the addition of oxaliplatin at the expense of increased toxicity.

The data regarding the use of anti–epidermal growth factor receptor therapy in older patients are available from retro-spective or small prospective studies. Sub-group analysis of patients age > 65 years, who took part in the large randomized clinical trials that led to the approval of cetuximab and panitumumab, showed mixed efficacy results.66 An observational study from Germany found similar toxic-ity and efficacy among 305 patients age > 65 years treated with the combination of cetuximab and irinotecan compared with their younger counterparts.22 Similar find-ings were reported by the Spanish Group for Digestive Tumors Therapy (TTD), which studied cetuximab as a single agent and in combination with irinotecan or capecit-abine in the older population.67–69

The two recently approved biologic agents for treatment of metastatic CC—regorafenib, a multikinase inhibitor, and aflibricept, a VEGF receptor inhibitor—demonstrated clinical benefit in large phase III studies.70,71 Subgroup analyses of patients age > 65 years in both studies demonstrated a minimal benefit in PFS, with lack of benefit in OS. Evaluation of the safety profile of regorafenib found similar results in patients age > and < 65 years.72 Although the oncologic community is still learning how to use these drugs in the general population of patients with meta-static CC, data are significantly lacking

regarding the utility and role of these drugs in the management of older patients with refractory metastatic CC.

Overall, the available data demonstrate clinical benefit of most approved treat-ment options for metastatic CC among older patients (Table 3). Additional studies are warranted to clarify further the most appropriate treatment approach for this patient population. At present, limited robust data exist beyond first-line treat-ment with fluoropyrimidine/bevacizumab with or without oxaliplatin/irinotecan to guide management in older adults with CC. Using the available data, oncologists must weigh the risks and benefits of each treatment regimen, taking into account patients’ ability to tolerate therapy and their treatment goals.

■■ Discussion

Older patients account for half of those diagnosed with CC annually. This review demonstrates clinical and survival benefits from FU/LV monotherapy in the adjuvant setting for stage III CC. The benefit of the addition of oxaliplatin is uncertain, and further investigation is warranted. In the metastatic setting, RR improves with multiagent therapy, although without a significant survival benefit and often at the cost of increased toxicity. However, many older patients with CC will not receive rec-ommended chemotherapy in the postop-erative or metastatic setting. Although the reasons for this continue to be explored, trial-based recommendations regarding patient selection for treatment are lack-ing. How clinicians triage older patients for chemotherapy varies, incorporating repro-ducible metrics like comorbid conditions and laboratory studies of physiologic func-tion as well as criteria potentially fraught with bias, such as perception of functional status, social support, and frailty. Ongo-ing efforts in the geriatric oncology com-munity to refine our ability to predict for toxicity before chemotherapy initiation

Key points

•The data regarding the use of anti-epidermal growth factor receptor therapy in older patients are available from retrospective or small prospective studies.

•Subgroup analysis of patients age > 65 years, who took part in the large randomized clinical trials that led to the approval of cetuximab and panitumumab, showed mixed efficacy results.

•An observational study from Germany found similar toxicity and efficacy among 305 patients age > 65 years treated with the combination of cetuximab and irinotecan compared with their younger counterparts.

•The two recently approved biologic agents for treatment of metastatic CC—regorafenib, a multikinase inhibitor, and aflibricept, a VEGF receptor inhibitor—demonstrated clinical benefit in large phase III studies.

•Overall, the available data demonstrate clinical benefit of most approved treatment options for metastatic CC among older patients.

•Oncologists must weigh the risks and benefits of each treatment regimen, taking into account patients’ ability to tolerate therapy and their treatment goals.

•This review demonstrates clinical and survival benefits from FU/LV monotherapy in the adjuvant setting for stage III CC. The benefit of the addition of oxaliplatin is uncertain, and further investigation is warranted.

ASCO-CRC-2015-V2-INT.indb 28 4/8/2015 4:41:40 PM

Copyright by MT

Page 33: › medical_mx › images › ... · 5 Samantha Hendren; Ellen McKeown; Arden M . Morris; Sandra L . Wong; Mary Oerline; Lyndia Poe; Darrell A . Campbell Jr, and Nancy J . Birkmeyer

Refining the Chemotherapy Approach for Older Patients With Colon Cancer

29

■■

TAB

LE 3

- S

tud

ies

of

chem

oth

erap

y tr

eatm

ents

fo

r o

lder

ad

ult

s w

ith

met

asta

tic

colo

n c

ance

r

Stu

dy

Des

ign

Res

ult

sTo

xici

ty

Folp

rech

t et

al10

(200

4)M

edia

n O

SRR M

edia

n PF

S

Pool

ed a

naly

sis

of 2

2 st

udie

s; s

ingl

e-ag

ent

FUA

ge <

70

(n =

3,1

96) v

s. ≥

70

year

s (n

= 6

29):

11.3

vs.

10.

8 m

onth

s; P

= .3

121

.1%

vs.

23.

9%; P

= .1

45.

3 vs

. 5.5

mon

ths;

P =

.01

Not

rep

orte

d

Gol

dber

g et

al16

(200

6)

RFS

Med

ian

OS

Pool

ed a

naly

sis

of f

our

clin

ical

tria

ls w

ith

FOLF

OX

che

mot

hera

py; F

U a

nd o

xalip

latin

Age

< 7

0 (n

= 3

,128

) vs.

≥ 7

0 ye

ars

(n =

614

) ±

oxal

ipla

tin:

HR,

0.7

; 95%

Cl,

0.63

to

0.77

vs.

HR,

0.6

5; 9

5%

Cl,

0.52

to

0.81

; P=

 .42

HR,

0.7

7; 9

5% C

l, 0.

67 t

o 0.

88 v

s. H

R, 0

.82;

95

% C

l, 0.

63 t

o 1.

06; P

= .7

9

Hig

her

grad

e ≥

3 he

mat

olog

ic t

oxic

ity

in p

atie

nts

age

≥ 70

yea

rs (4

9%

vs. 4

3%; P

= .0

4); n

o di

ffer

ence

in

inci

denc

e of

neu

rolo

gic

or G

l AEs

MRC

FO

CU

S2 s

tudy

; Se

ymou

r et

 al13

(201

1)

PFS

OS

QO

L

RCT

phas

e III

stu

dy o

f 45

9 el

derly

or

frai

l pa

tient

s un

fit f

or s

tand

ard

chem

othe

rapy

; fir

st-li

ne F

U o

r ca

peci

tabi

ne ±

oxa

lipla

tin a

t 20

% d

ose

redu

ctio

nW

ith v

s. w

ithou

t do

se-r

educ

ed o

xapl

iatin

: HR,

0.

84; 9

5% C

l, 0.

69 t

o 1.

01; P

= .0

7; F

U v

s.

cape

cita

bine

: HR,

0.9

9; 9

5% C

l, 0.

82 t

o 1.

20;

P =

.93

With

vs.

with

out d

ose-

redu

ced

oxal

ipla

tin: H

R, 0

.99;

95

% C

l, 0.

81 to

1.1

8; P

= .9

1; F

U v

s. c

apec

itabi

ne:

HR,

0.9

6; 9

5% C

l, 0.

79 to

1.1

7; P

= .7

1N

ot im

prov

ed w

ith c

apec

itabi

ne c

ompa

red

with

in

fusi

onal

FU

No

incr

ease

in r

isk

of a

ny g

rade

3

toxi

city

with

add

ition

of

oxal

ipla

tin;

high

er r

isk

of a

ny g

rade

3 t

oxic

ity

with

cap

ecita

bine

com

pare

d w

ith

FU

Folp

rech

t et

al17

(200

8)

RR Med

ian

PFS

Med

ian

OS

Pool

ed a

naly

sis

of f

our

phas

e III

stu

dies

ev

alua

ting

bene

fit o

f ad

ding

irin

otec

an t

o FU

; firs

t-lin

e iri

note

can

mon

othe

rapy

or

in

com

bina

tion

with

FU

Age

< 7

0 (n

= 2

,092

) vs.

≥ 7

0 ye

ars

(n =

599

) ±

irino

teca

n:

46.6

% v

s. 2

9.0%

(P <

.001

) vs.

50.

5% v

s. 3

0.3%

(P

 < .0

01);

P =

.33

HR,

0.7

7; 9

5% C

l, 0.

70 t

o 0.

85 (P

 < .0

01) v

s. H

R,

0.75

; 95%

Cl,

0.61

to

0.95

(P =

.002

6); P

= .8

4H

R, 0

.83;

95%

Cl,

0.75

to

0.92

(P <

.001

) vs.

HR,

0.

87; 9

5% C

l, 0.

72 t

o 1.

05 (P

= .1

5); P

= .6

1

Add

ition

of

irino

teca

n re

sulte

d in

si

mila

rto

xici

ty p

rofil

e am

ong

youn

ger

and

olde

r pa

tient

s

Fige

r et

al60

(200

7)

ORR

Med

ian

PFS

Med

ian

OS

Subg

roup

ana

lysi

s of

OPT

IMO

X-1

stu

dy;

first

-line

con

tinuo

us F

OLF

OX

vs.

six

cyc

les

follo

wed

by

FU m

aint

enan

ce

Age

< 7

6 (n

= 6

20) v

s. 7

6 to

80

year

s (n

= 3

7):

59%

vs.

59.

4%; P

= N

S 9

vs. 9

mon

ths;

P =

.63

20.2

vs.

20.

7 m

onth

s; P

= .5

7

Tren

d to

war

d in

crea

sed

grad

e >

3 A

Es

in o

lder

pat

ient

s (6

5% v

s. 4

8%; P

=

.06)

; hig

her n

eutr

open

ia (4

1%

vs. 2

4%; P

 = .0

3) a

nd n

euro

toxi

city

(2

2% v

s. 1

1%; P

= .0

6)

ASCO-CRC-2015-V2-INT.indb 29 4/8/2015 4:41:40 PM

Copyright by MT

Page 34: › medical_mx › images › ... · 5 Samantha Hendren; Ellen McKeown; Arden M . Morris; Sandra L . Wong; Mary Oerline; Lyndia Poe; Darrell A . Campbell Jr, and Nancy J . Birkmeyer

Syllabus

30

Stu

dy

Des

ign

Res

ult

sTo

xici

ty

BEA

T da

taba

se18

,73 (2

009)

Med

ian

PFS

Med

ian

OS

Subg

roup

ana

lysi

s; F

OLF

OX

/FO

LFIR

I/FU

plu

s be

vaci

zum

abA

ge <

65

(n =

1,2

86) v

s. 6

5 to

74

(n =

499

) vs.

≥ 

75 y

ears

(n =

129

):10

.8 v

s. 1

1.2

vs. 1

0.0

mon

ths:

P =

NR

23.5

vs.

22.

8 vs

. 16.

6 m

onth

s; P

= N

R

Sim

ilar

toxi

city

pro

file

amon

g yo

unge

r an

d ol

der

patie

nts

BRiT

E st

udy74

(201

0)

Med

ian

PFS

Med

ian

OS

Subg

roup

ana

lysi

s; f

irst-

line

chem

othe

rapy

pl

us b

evac

izum

abA

ge <

65

(n =

1,0

57) v

s. 6

5 to

74

(n =

533

) vs.

75

to 7

9 (n

= 2

02) v

s. ≥

80

year

s (n

= 1

61):

9.8

vs. 9

.6 v

s. 1

0 vs

. 8.6

mon

ths;

P =

NR

26 v

s. 2

1.1

vs. 2

0.3

vs. 1

6.2

mon

ths;

P =

NR

Incr

ease

d in

cide

nce

of a

rter

ial

thro

mbo

embo

lism

with

incr

easi

ng

age

Cas

sidy

et

al21

(201

0)

Age

< 6

5 ye

ars

(n =

1,8

64)

± b

evac

izum

abM

edia

n PF

S

Med

ian

OS

Age

≥ 6

5 ye

ars

(n =

1,1

42)

± b

evac

izum

abM

edia

n PF

S

Med

ian

OS

Age

≥ 7

0 ye

ars

(n =

712

) ±

beva

cizu

mab

Med

ian

PFS

Med

ian

OS

Retr

ospe

ctiv

e an

alys

is o

f el

derly

pat

ient

s en

rolle

d on

to f

our

RCTs

of

beva

cizu

mab

pl

us c

hem

othe

rapy

; firs

t-an

d se

cond

-line

ch

emot

hera

py p

lus

beva

cizu

mab

9.5

vs. 6

.7 m

onth

s; P

 < .0

01; H

R, 0

.59;

95%

Cl,

0.52

to

0.66

19.9

vs.

16.

5 m

onth

s; P

 < .0

01; H

R, 0

.77;

95%

C

l, 0.

69 t

o 0.

86

9.3

vs. 6

.9 m

onth

s; P

 < .0

01; H

R, 0

.59;

95%

Cl,

0.52

to

0.66

17.9

vs.

15.

0 m

onth

s; P

= .0

15; H

R, 0

.85;

95%

C

l, 0.

74 t

o 0.

97

9.2

vs. 6

.4 m

onth

s; P

> .0

01; H

R, 0

.54;

95%

Cl,

0.44

to

0.66

17.4

vs.

14.

1 m

onth

s; P

= .0

05; H

R, 0

.79;

95%

C

l, 0.

66 t

o 0.

93

Incr

ease

d in

cide

nce

of a

rter

ial

thro

mbo

embo

lic e

vent

s in

old

er

patie

nts

Kab

bina

var

et a

l63 (2

009)

Med

ian

PFS

Med

ian

OS

ORR

Pool

ed a

naly

sis

of t

wo

RCTs

of

patie

nts

age

≥ 65

yea

rs (n

= 4

39);

first

-line

ch

emot

hera

py ±

bev

aciz

umab

With

vs.

with

out

beva

cizu

mab

:

9.2

vs. 6

.2 m

onth

s; P

< .0

0119

.3 v

s. 1

4.3

mon

ths;

P =

.006

34.4

% v

s. 2

9%; P

= .2

2

AE

rate

s w

ere

cons

iste

nt w

ith t

hose

re

port

ed in

ful

l coh

orts

of

two

orig

inal

stu

dies

AV

EX s

tudy

75 (2

013)

Ph

ase

III R

CT

of p

atie

nts

age

≥ 70

yea

rs w

ho

wer

e no

t ca

ndid

ates

for

oxa

lipla

tin-

or

irino

teca

n-ba

sed

ther

apy

(n =

280

); fir

st-

line

cape

cita

bine

± b

evac

izum

ab

With

vs.

with

out

beva

cizu

mab

:Ra

te o

f an

y gr

ade

≥ 3

AEs

was

hig

her

with

bev

aciz

umab

(40%

vs.

22%

)

■■

TAB

LE 3

- S

tud

ies

of

chem

oth

erap

y tr

eatm

ents

fo

r o

lder

ad

ult

s w

ith

met

asta

tic

colo

n c

ance

r (c

on

tin

ued

)

ASCO-CRC-2015-V2-INT.indb 30 4/8/2015 4:41:40 PM

Copyright by MT

Page 35: › medical_mx › images › ... · 5 Samantha Hendren; Ellen McKeown; Arden M . Morris; Sandra L . Wong; Mary Oerline; Lyndia Poe; Darrell A . Campbell Jr, and Nancy J . Birkmeyer

Refining the Chemotherapy Approach for Older Patients With Colon Cancer

31

Med

ian

PFS

Med

ian

OS

ORR

9.1

vs. 5

.1 m

onth

s; H

R, 0

.53;

95%

Cl,

0.41

to

0.69

; P <

.001

20.7

vs.

18.

8 m

onth

s; H

R, 0

.79;

95%

Cl,

0.57

to

1.09

; P =

.18

19%

vs.

10%

; P =

.04

Jehn

et

al22

(201

2)

ORR

Med

ian

PFS

Obs

erva

tiona

l stu

dy; c

hem

othe

rapy

plu

s ce

tuxi

mab

(any

line

)A

ge <

65

(n =

309

) vs.

≥ 6

5 ye

ars

(n =

305

):

38%

vs.

 36%

; P =

.89

6.5

vs. 7

.0 m

onth

s; P

= .1

2

Gra

de 3

or

4 to

xici

ties

in 2

0% o

f pa

tient

cas

es, r

egar

dles

s of

age

Sast

re e

t aI

68 (2

011)

ORR

Med

ian

PFS

Med

ian

OS

Phas

e II

sing

le-a

rm s

tudy

of

patie

nts

age

70 y

ears

(n =

41)

; firs

t-lin

e si

ngle

-age

nt

cetu

xim

ab14

%2.

9 m

onth

s11

.1 m

onth

s

Gra

de 3

ras

h, 1

2.2%

Sast

re e

t al

69 (2

012)

ORR

Med

ian

PFS

Med

ian

OS

Phas

e II

stud

y of

pat

ient

s ag

e ≥

70 y

ears

(n

 = 6

6); f

irst-

line

cape

cita

bine

plu

s ce

tuxi

mab

31.8

%; 9

5% C

l, 20

.9 t

o 44

.47.

1 m

onth

s; 9

5% C

l, 5.

3 to

8.4

16.1

mon

ths;

95%

Cl,

12.0

to

18.8

Gra

de 3

to

4 ra

sh, 2

8.2%

; gra

de 3

to

4 di

arrh

ea, 1

8.5%

Abd

elw

ahab

et

al67

(201

2)

Med

ian

PFS

Med

ian

OS

Phas

e II

stud

y of

pat

ient

s ag

e ≥

65 y

ears

(n

= 4

6); ≥

sec

ond-

line

cetu

xim

ab p

lus

irino

teca

n4

mon

ths

7 m

onth

s

Gra

de 3

to

4 ra

sh, 2

0%; g

rade

3

to 4

dia

rrhe

a, 1

8%; g

rade

3 t

o 4 

neut

rope

nia,

28%

AE,

adv

erse

eve

nt; A

VEX

, Ava

stin

in E

lder

ly W

ith X

elod

a; B

EAT,

Bev

aciz

umab

Exp

ande

d A

cces

s Tr

ial;

BRiT

E, B

evac

izum

ab R

egim

ens

Inve

stig

atio

n of

Tre

atm

ent

Effe

cts;

MRC

FO

CU

S2, U

K M

edic

al R

esea

rch

Cou

ncil

Fluo

rour

acil,

Oxa

lipla

tin, a

nd C

apec

itabi

ne, U

se a

nd S

eque

ncin

g; F

OLF

IRI,

fluor

oura

cil,

leuc

ovor

in, a

nd ir

inot

ecan

; FO

LFO

X, f

luor

oura

cil,

leuc

ovor

in, a

nd o

xalip

latin

; FU

, flu

orou

raci

l; H

R, h

azar

d ra

tio; N

R, n

ot r

each

ed;

NS,

not

sig

nific

ant;

OPT

IMO

X-1

, Opt

imiz

ed L

euco

vorin

-Flu

orou

raci

l Oxa

lipla

tin 1

; ORR

, ove

rall

resp

onse

rat

e; O

S, o

vera

ll su

rviv

al; P

FS, p

rogr

essi

on-f

ree

surv

ival

; QO

L, q

ualit

y of

life

; RC

T, r

ando

miz

ed c

ontr

olle

d tr

ial;

RFS,

rel

apse

-fre

e su

rviv

al; R

R, r

espo

nse

rate

.

ASCO-CRC-2015-V2-INT.indb 31 4/8/2015 4:41:40 PM

Copyright by MT

Page 36: › medical_mx › images › ... · 5 Samantha Hendren; Ellen McKeown; Arden M . Morris; Sandra L . Wong; Mary Oerline; Lyndia Poe; Darrell A . Campbell Jr, and Nancy J . Birkmeyer

Syllabus

32

■■ References 1. National Cancer Institute: Surveillance, Epidemiology, and

End Results Program: SEER Fact Sheets—Colon and Rectum Cancer. http://seer.cancer.gov/statfacts/html/colorect.html

2. Luo R, Giordano SH, Freeman JL, et al: Referral to medical oncology: A crucial step in the treatment of older patients with stage III colon cancer. Oncologist 11:1025–1033, 2006.

3. Gross CP, Guo Z, McAvay GJ, et al: Multimorbidity and survival in older persons with colorectal cancer. J Am Geriatr Soc 54:1898–1904, 2006.

4. Schrag D, Cramer LD, Bach PB, et al: Age and adjuvant chemotherapy use after surgery for stage III colon cancer. J Natl Cancer Inst 93:850–857, 2001.

5. Sanoff HK, Carpenter WR, Stürmer T, et al: Effect of adjuvant chemotherapy on survival of patients with stage III colon cancer diagnosed after age 75 years. J Clin Oncol 30: 2624–2634, 2012.

6. Ho C, Ng K, O’Reilly S, et al: Outcomes in elderly patients with advanced colorectal cancer treated with capecitabine: A population-based analysis. Clin Colorectal Cancer 5: 279–282, 2005.

7. Centers for Disease Control and Prevention: National Vital Statistics Report: United States Life Tables, 2004. http://www.cdc.gov/nchs/data/nvsr/nvsr56/nvsr56_09.pdf

8. Sargent DJ, Goldberg RM, Jacobson SD, et al: A pooled analysis of adjuvant chemotherapy for resected colon cancer in elderly patients. N Engl J Med 345:1091–1097, 2001.

9. D’Andre S, Sargent DJ, Cha SS, et al: 5-Fluorouracil-based chemotherapy for advanced colorectal cancer in elderly patients: A North Central Cancer Treatment Group study. Clin Colorectal Cancer 4:325–331, 2005.

10. Folprecht G, Cunningham D, Ross P, et al: Efficacy of 5-fluorouracil-based chemotherapy in elderly patients with metastatic colorectal cancer: A pooled analysis of clinical trials. Ann Oncol 15:1330–1338, 2004.

11. de Gramont A, Bosset JF, Milan C, et al: Randomized trial comparing monthly low-dose leucovorin and fluorouracil bolus with bimonthly high-dose leucovorin and fluorouracil bolus plus continuous infusion for advanced colorectal cancer: A French Intergroup study. J Clin Oncol 15:808–815, 1997.

12. Product Information: XELODA oral tablets, capecitabine oral tablets. Genentech, South San Francisco, CA.

13. Seymour MT, Thompson LC, Wasan HS, et al: Chemotherapy options in elderly and frail patients with metastatic colorectal cancer (MRC FOCUS2): An open-label, randomised factorial trial. Lancet 377:1749–1759, 2011.

14. MacLaughlin EJ, Raehl CL, Treadway AK, et al: Assessing medication adherence in the elderly: Which tools to use in clinical practice? Drugs Aging 22:231–255, 2005.

15. Product information: ELOXATIN intravenous lyophilized powder for solution, intravenous concentrate for solution, oxaliplatin intravenous lyophilized powder for solution, intravenous concentrate for solution. sanofi-aventis, Bridgewater, NJ.

16. Goldberg RM, Tabah-Fisch I, Bleiberg H, et al: Pooled analysis of safety and efficacy of oxaliplatin plus fluorouracil/leucovorin administered bimonthly in elderly patients with colorectal cancer. J Clin Oncol 24:4085–4091, 2006.

17. Folprecht G, Seymour MT, Saltz L, et al: Irinotecan/fluorouracil combination in first-line therapy of older and younger patients with metastatic colorectal cancer: Combined analysis of 2,691 patients in randomized controlled trials. J Clin Oncol 26:1443–1451, 2008.

18. Van Cutsem E, Rivera F, Berry S, et al: Safety and efficacy of first-line bevacizumab with FOLFOX, XELOX, FOLFIRI, and fluoropyrimidines in metastatic colorectal cancer: The BEAT study. Ann Oncol 20:1842–1847, 2009.

19. Kozloff M, Yood MU, Berlin J, et al: Clinical outcomes associated with bevacizumab-containing treatment of metastatic colorectal cancer: The BRiTE observational cohort study. Oncologist 14:862–870, 2009.

20. Scappaticci FA, Skillings JR, Holden SN, et al: Arterial thromboembolic events in patients with metastatic carcinoma treated with chemotherapy and bevacizumab. J Natl Cancer Inst 99:1232–1239, 2007.

21. Cassidy J, Saltz LB, Giantonio BJ, et al: Effect of bevacizumab in older patients with metastatic colorectal cancer: Pooled analysis of four randomized studies. J Cancer Res Clin Oncol 136:737–743, 2010.

22. Jehn CF, Böning L, Kröning H, et al: Cetuximab-based therapy in elderly comorbid patients with metastatic colorectal cancer. Br J Cancer 106:274–278, 2012.

23. Gross CP, McAvay GJ, Krumholz HM, et al: The effect of age and chronic illness on life expectancy after a diagnosis of

will significantly improve our ability to tai-lor chemotherapy recommendations.12,76 Recent studies regarding the benefit of CGA may help guide decision making. However, until we are able to conduct randomized clinical trials that include a representative proportion of older patients (ie, age-inclusive trials), the available ret-rospective, pooled, and meta-analyses, which are limited by potential confound-ing, will primarily guide decision making in this population. Furthermore, the addition

of geriatric-specific end points to clinical trials will enhance our ability to study the implication of each therapy with regard to multiple aspects of the older patient’s cancer and noncancer experiences and enable us to generalize results to the popu-lation of older patients with CC at large. It behooves us as an oncology community to develop concerted efforts to study and define strategies to optimize chemotherapy receipt for older patients with CC in the United States.

ASCO-CRC-2015-V2-INT.indb 32 4/8/2015 4:41:40 PM

Copyright by MT

Page 37: › medical_mx › images › ... · 5 Samantha Hendren; Ellen McKeown; Arden M . Morris; Sandra L . Wong; Mary Oerline; Lyndia Poe; Darrell A . Campbell Jr, and Nancy J . Birkmeyer

Refining the Chemotherapy Approach for Older Patients With Colon Cancer

33

colorectal cancer: Implications for screening. Ann Intern Med 145:646–653, 2006.

24. Koroukian SM, Xu F, Bakaki PM, et al: Comorbidities, functional limitations, and geriatric syndromes in relation to treatment and survival patterns among elders with colorectal cancer. J Gerontol A Biol Sci Med Sci 65:322–329, 2010.

25. Hoppe S, Rainfray M, Fonck M, Hoppenreys L, Blanc JF, Ceccaldi J, et al: Functional decline in older patients with cancer receiving first-line chemotherapy. J Clin Oncol 31(31):3877–3882, 2013.

26. Aparicio T, Jouve JL, Teillet L, et al: Geriatric factors predict chemotherapy feasibility: Ancillary results of FFCD 2001–02 phase III study in first-line chemotherapy for metastatic colorectal cancer in elderly patients. J Clin Oncol 31: 1464–1470, 2013.

27. Hurria A, Togawa K, Mohile SG, et al: Predicting chemotherapy toxicity in older adults with cancer: A prospective multicenter study. J Clin Oncol 29:3457–3465, 2011.

28. Extermann M, Boler I, Reich R, et al: The Chemotherapy Risk Assessment Scale for High-Age Patients (CRASH) score: Design and validation. J Clin Oncol 28:636s, 2010 (suppl; abstr 9000).

29. Stuck AE, Siu AL, Wieland GD, et al: Comprehensive geriatric assessment: A meta-analysis of controlled trials. Lancet 342:1032–1036, 1993.

30. Kenis C, Bron D, Libert Y, et al: Relevance of a systematic geriatric screening and assessment in older patients with cancer: Results of a prospective multicentric study. Ann Oncol 24:1306–1312, 2013.

31. Decoster L, Kenis C, Van Puyvelde K, et al: The influence of clinical assessment (including age) and geriatric assessment on treatment decisions in older patients with cancer. J Geriatr Oncol 4:235–241, 2013.

32. Talarico L, Chen G, Pazdur R: Enrollment of elderly patients in clinical trials for cancer drug registration: A 7-year experience by the US Food and Drug Administration. J Clin Oncol 22:4626–4631, 2004.

33. Hempenius L, Slaets JP, Boelens MA, et al: Inclusion of frail elderly patients in clinical trials: Solutions to the problems. J Geriatr Oncol 4:26–31, 2013.

34. Wildiers H, Mauer M, Pallis A, et al: End points and trial design in geriatric oncology research: A joint European Organisation for Research and Treatment of Cancer-Alliance for Clinical Trials in Oncology–International Society of Geriatric Oncology position article. J Clin Oncol 31: 371 1–3718, 2013.

35. Twelves C, Wong A, Nowacki MP, et al: Capecitabine as adjuvant treatment for stage III colon cancer. N Engl J Med 352:2696–2704, 2005.

36. Scheithauer W, McKendrick J, Begbie S, et al: Oral capecitabine as an alternative to i.v. 5-fluorouracil-based adjuvant therapy for colon cancer: Safety results of a randomized, phase III trial. Ann Oncol 14:1735–1743, 2003.

37. André T, Boni C, Navarro M, et al: Improved overall survival with oxaliplatin, fluorouracil, and leucovorin as adjuvant treatment in stage II or III colon cancer in the MOSAlC trial. J Clin Oncol 27:3109–3116, 2009.

38. Tournigand C, André T, Bonnetain F, et al: Adjuvant therapy with fluorouracil and oxaliplatin in stage ll and elderly patients (between ages 70 and 75 years) with colon cancer: Subgroup analyses of the Multicenter lnternational Study of Oxaliplatin, Fluorouracil, and Leucovorin in the Adjuvant

Treatment of Colon Cancer trial. J Clin Oncol 30: 3353–3360, 2012.

39. Haller DG, Tabernero J, Maroun J, et al: Capecitabine plus oxaliplatin compared with fluorouracil and folinic acid as adjuvant therapy for stage lll colon cancer. J Clin Oncol 29:1465–1471, 2011.

40. McCleary NJ, Meyerhardt JA, Green E, et al: lmpact of age on the efficacy of newer adjuvant therapies in patients with stage ll/lll colon cancer: Findings from the ACCENT database. J Clin Oncol 31:2600–2606, 2013.

41. Iwashyna TJ, Lamont EB: Effectiveness of adjuvant fluorouracil in clinical practice: A population-based cohort study of elderly patients with stage lll colon cancer. J Clin Oncol 20:3992–3998, 2002.

42. Abrams TA, Brightly R, Mao J, et al: Patterns of adjuvant chemotherapy use in a population-based cohort of patients with resected stage ll or lll colon cancer. J Clin Oncol 29:3255–3262, 2011.

43. Centers for Disease Control and Prevention: Fatalities and injuries from falls among older adults: United States, 1993–2003 and 2001–2005. MMWR Morb Mortal Wkly Rep 55:1303, 2006.

44. Sargent DJ, Marsoni S, Monges G, et al: Defective mismatch repair as a predictive marker for lack of efficacy of fluorouracil-based adjuvant therapy in colon cancer. J Clin Oncol 28:3219–3226, 2010.

45. Ribic CM, Sargent DJ, Moore MJ, et al: Tumor microsatellite-instability status as a predictor of benefit from fluorouracil-based adjuvant chemotherapy for colon cancer. N Engl J Med 349:247–257, 2003.

46. Bertagnolli MM, Niedzwiecki D, Compton CC, et al: Microsatellite instability predicts improved response to adjuvant therapy with irinotecan, fluorouracil, and leucovorin in stage lll colon cancer: Cancer and Leukemia Group B Protocol 89803. J Clin Oncol 27:1814–1821, 2009.

47. Aparicio T, Schischmanoff O, Poupardin C, et al: Deficient mismatch repair phenotype is a prognostic factor for colorectal cancer in elderly patients. Dig Liver Dis 45: 245–250, 2013.

48. Gray R, Barnwell J, McConkey C, et al: Adjuvant chemotherapy versus observation in patients with colorectal cancer: A randomised study. Lancet 370:2020–2029, 2007.

49. Siegel R, Naishadham D, Jemal A: Cancer statistics, 2013. CA Cancer J Clin 63:11–30, 2013.

50. Yothers G, O’Connell MJ, Allegra CJ, et al: Oxaliplatin as adjuvant therapy for colon cancer: Updated results of NSABP C-07 trial, including survival and subset analyses. J Clin Oncol 29:3768–3774, 2011.

51. Lembersky BC, Wieand HS, Petrelli NJ, et al: Oral uracil and tegafur plus leucovorin compared with intravenous fluorouracil and leucovorin in stage ll and lll carcinoma of the colon: Results from National Surgical Adjuvant Breast and Bowel Project Protocol C-06. J Clin Oncol 24:2059–2064, 2006.

52. Twelves C, Scheithauer W, McKendrick J, et al: Capecitabine versus 5-fluorouracil/folinic acid as adjuvant therapy for stage lll colon cancer: Final results from the X-ACT trial with analysis by age and preliminary evidence of a pharmacodynamic marker of efficacy. Ann Oncol 23: 1190–1197, 2012.

53. Koopman M, Antonini NF, Douma J, et al: Sequential versus combination chemotherapy with capecitabine, irinotecan, and oxaliplatin in advanced colorectal cancer (CAlRO): A phase lll randomised controlled trial. Lancet 370:135–142, 2007.

ASCO-CRC-2015-V2-INT.indb 33 4/8/2015 4:41:40 PM

Copyright by MT

Page 38: › medical_mx › images › ... · 5 Samantha Hendren; Ellen McKeown; Arden M . Morris; Sandra L . Wong; Mary Oerline; Lyndia Poe; Darrell A . Campbell Jr, and Nancy J . Birkmeyer

Syllabus

34

54. Ducreux M, Malka D, Mendiboure J, et al: Sequential versus combination chemotherapy for the treatment of advanced colorectal cancer (FFCD 2000–05): An open-label, randomised, phase 3 trial. Lancet Oncol 12:1032–1044, 2011.

55. Goldberg RM, Sargent DJ, Morton RF, et al: A randomized controlled trial of fluorouracil plus leucovorin, irinotecan, and oxaliplatin combinations in patients with previously untreated metastatic colorectal cancer. J Clin Oncol 22:23–30, 2004.

56. Feliu J, Salud A, Escudero P, et al: XELOX (capecitabine plus oxaliplatin) as first-line treatment for elderly patients over 70 years of age with advanced colorectal cancer. Br J Cancer 94:969–975, 2006.

57. Twelves CJ, Butts CA, Cassidy J, et al: Capecitabine/oxaliplatin, a safe and active first-line regimen for older patients with metastatic colorectal cancer: Post hoc analysis of a large phase ll study. Clin Colorectal Cancer 5:101–107, 2005.

58. Souglakos J, Pallis A, Kakolyris S, et al: Combination of irinotecan (CPT-11) plus 5-fluorouracil and leucovorin (FOLFlRl regimen) as first line treatment for elderly patients with metastatic colorectal cancer: A phase ll trial. Oncology 69:384–390, 2005.

59. Maindrault-Goebel F, Lledo G, Chibaudel B, et al: Final results of OPTlMOX2, a large randomized phase ll study of maintenance therapy or chemotherapy-free intervals (CFl) after FOLFOX in patients with metastatic colorectal cancer (MRC): A GERCOR study. J Clin Oncol 25:166s, 2007 (suppl; abstr 4013).

60. Figer A, Perez-Staub N, Carola E, et al: FOLFOX in patients aged between 76 and 80 years with metastatic colorectal cancer: An exploratory cohort of the OPTIMOX1 study. Cancer 110:2666–2671, 2007.

61. Pasetto LM, Falci C, Sinigaglia G, et al: How many colorectal cancer (CRC) patients older than 70 years may be safely treated with bevacizumab? J Clin Oncol 24:623s, 2006 (suppl; abstr 13589).

62. Kozloff M, Sugrue MM, Berlin J, et al: Safety and effectiveness of bevacizumab (BV) and chemotherapy (CT) in elderly patients (pts) with metastatic colorectal cancer (mCRC): Results from the BRiTE Prospective Cohort Study. J Clin Oncol 26, 2008 (suppl; abstr 454).

63. Kabbinavar FF, Hurwitz Hl, Yi J, et al: Addition of bevacizumab to fluorouracil-based first-line treatment of metastatic colorectal cancer: Pooled analysis of cohorts of older patients from two randomized clinical trials. J Clin Oncol 27:199–205, 2009.

64. Kabbinavar FF, Schulz J, McCleod M, et al: Addition of bevacizumab to bolus fluorouracil and leucovorin in first-line metastatic colorectal cancer: Results of a randomized phase ll trial. J Clin Oncol 23:3697–3705, 2005.

65. Cunningham D, Lang l, Marcuello E, et al: Bevacizumab plus capecitabine versus capecitabine alone in elderly patients with previously untreated metastatic colorectal cancer (AVEX): An open-label, randomised phase 3 trial. Lancet Oncol 14:1077–1085, 2013.

66. Douillard JY, Siena S, Cassidy J, et al: Randomized, phase lll trial of panitumumab with infusional fluorouracil, leucovorin, and oxaliplatin (FOLFOX4) versus FOLFOX4 alone as first-line treatment in patients with previously untreated metastatic colorectal cancer: The PRlME study. J Clin Oncol 28: 4697–4705, 2010.

67. Abdelwahab S, Azmy A, Abdel-Aziz H, et al: Anti-EGFR (cetuximab) combined with irinotecan for treatment of elderly patients with metastatic colorectal cancer (mCRC). J Cancer Res Clin Oncol 138:1487–1492, 2012.

68. Sastre J, Aranda E, Grávalos C, et al: First-line single-agent cetuximab in elderly patients with metastatic colorectal cancer: A phase ll clinical and molecular study of the Spanish group for digestive tumor therapy (TTD). Crit Rev Oncol Hematol 77:78–84, 2011.

69. Sastre J, Grávalos C, Rivera F, et al: First-line cetuximab plus capecitabine in elderly patients with advanced colorectal cancer: Clinical outcome and subgroup analysis according to KRAS status from a Spanish TTD Group Study. Oncologist 17:339–345, 2012.

70. Grothey A, Van Cutsem E, Sobrero A, et al: Regorafenib monotherapy for previously treated metastatic colorectal cancer (CORRECT): An international, multicentre, randomised, placebo-controlled, phase 3 trial. Lancet 381:303–312, 2013.

71. Tabernero J, Van Cutsem E, Lakomý R, et al: Aflibercept versus placebo in combination with fluorouracil, leucovorin and irinotecan in the treatment of previously treated metastatic colorectal cancer: Prespecified subgroup analyses from the VELOUR trial. Eur J Cancer 50:320–331, 2014.

72. Van Cutsem E, Sobrero A, Siena S, et al: Regorafenib (REG) in progressive metastatic colorectal cancer (mCRC): Analysis of age subgroups in the phase lll CORRECT trial. J Clin Oncol 31, 2013 (suppl 15s; abstr 3636).

73. Van Cutsem E, Rivera F, Berry S, et al: Safety and efficacy of first-line bevacizumab with FOLFOX, XELOX, FOLFIRI and fluoropyrimidines in metastatic colorectal cancer: The BEAT study. Ann Oncol 20: 1842–1847, 2009.

74. Kozloff MF, Berlin J, Flynn PJ, et al: Clinical outcomes in elderly patients with metastatic colorectal cancer receiving bevacizumab and chemotherapy: Results from the BRiTE observational cohort study. Oncology 78:329–339, 2010.

75. Cunningham D, Lang l, Lorusso V, et al: Bevacizumab (bev) in combination with capecitabine (cape) for the first-line treatment of elderly patients with metastatic colorectal cancer (mCRC): Results of a randomized international phase Ill trial (AVEX). J Clin Oncol 30, 2012 (suppl; abstr 337).

76. Extermann M, Boler l, Reich RR, et al: Predicting the risk of chemotherapy toxicity in older patients: The Chemotherapy Risk Assessment Scale for High-Age Patients (CRASH) score. Cancer 118:3377–3386, 2012.

ASCO-CRC-2015-V2-INT.indb 34 4/8/2015 4:41:40 PM

Copyright by MT

Page 39: › medical_mx › images › ... · 5 Samantha Hendren; Ellen McKeown; Arden M . Morris; Sandra L . Wong; Mary Oerline; Lyndia Poe; Darrell A . Campbell Jr, and Nancy J . Birkmeyer

35

■■ Anthony C . Wong; Shannon Stock; Deborah Schrag; Katherine L . Kahn; Talya Salz; Mary E . Charlton; Selwyn O . Rogers Jr; Karyn A . Goodman, and Nancy L . Keating

Physicians’ Beliefs About the Benefits and Risks of Adjuvant Therapies for Stage II and Stage III Colorectal Cancer(J Oncol Pract 2014;10(5):e360–e367.)

Background: Adjuvant therapy plays a major role in treating colorectal cancer, and physicians’ views of its effec-tiveness influence treatment decisions . We assessed physicians’ views of the relative benefits and risks of adjuvant chemotherapy and radiotherapy for stages II and III colon and rectal cancers .Methods: The Cancer Care Outcomes Research and Surveillance Consortium surveyed a geographically dispersed population of medical oncologists, radiation oncologists, and surgeons in the United States about the benefits and risks of adjuvant therapies for colorectal cancer . We used logistic regression to assess the association of physician and practice characteristics with beliefs about adjuvant therapies .Results: Among 1,296 respondents, > 90% believed the benefits of adjuvant therapies for stage III colorectal cancer outweigh the risks . Only 21 .9%, 50%, and 50 .4% believed in the net benefit of chemotherapy for stage II colon cancer, chemotherapy for stage II rectal cancer, and radiation for stage II rectal cancer, respectively . Younger physicians were less likely than others to perceive adjuvant therapy for stage II colorectal cancer as beneficial . Medical oncologists were more likely than surgeons and radiation oncologists to endorse the benefits of adjuvant chemotherapy and radiation for stage II rectal cancer, but less likely for stage II colon cancer .Conclusions: Physicians largely agreed that the benefits of adjuvant chemotherapy for stage III colon cancer, as well as chemotherapy, and radiation for stage III rectal cancer, outweigh the risks, consistent with strong evi-dence, but were divided over the net benefit of adjuvant therapies for stage II colorectal cancer, where evidence is inconsistent .

■■ Introduction

Patterns-of-care studies have demonstrated significant variability in colorectal cancer care.1–6 Patients with colorectal cancer who receive guideline-concordant therapy survive longer than those who do not,3,7,8 yet many patients do not receive recom-mended therapies. Physicians are less likely to offer adjuvant therapy to older patients with colorectal cancer with medical comor-bidities than to younger and healthier patients.3,4,9,10 However, few data are avail-able regarding physicians’ beliefs about the risks and benefits of adjuvant therapy, even though physicians’ recommendations are

important determinants of patients’ treat-ment decisions.11

Current national guidelines recommend adjuvant chemotherapy for stage III and high-risk stage II colon cancer, and neo-adjuvant chemoradiotherapy and adju-vant chemotherapy for stages II–III rectal cancer.12,13 Features that confer high risk of recurrence in stage II colon cancer include large tumors (T4), bowel perforation or obstruction, lymphovascular invasion, poorly differentiated histology, positive margins, or inadequate lymph node sampling.14–16 Guideline recommendations for patients with stage III colorectal cancer are based

Key points

•Patterns-of-care studies have demonstrated significant variability in colorectal cancer care.

•Physicians are less likely to offer adjuvant therapy to older patients with colorectal cancer with medical comorbidities than to younger and healthier patients.

ASCO-CRC-2015-V2-INT.indb 35 4/8/2015 4:41:40 PM

Copyright by MT

Page 40: › medical_mx › images › ... · 5 Samantha Hendren; Ellen McKeown; Arden M . Morris; Sandra L . Wong; Mary Oerline; Lyndia Poe; Darrell A . Campbell Jr, and Nancy J . Birkmeyer

Syllabus

36

on high-quality randomized controlled trials that have demonstrated that adju-vant chemotherapy for stage III colon cancer17–21 and neoadjuvant radiation with or without chemotherapy for stage III rectal cancer22–26 improve outcomes. Treat-ment of stage II colon and rectal cancers is more controversial.17–20,25,27,28 For stage II colon cancer, studies that assessed survival after adjuvant fluorouracil (FU) chemo-therapy after curative resection have been mixed,17,18,20 and no overall survival benefit for adjuvant folinic acid (leucovorin)-FU-oxaliplatin (FOLFOX) chemotherapy has been found, even in high-risk stage II patients in whom recurrence rates approach those of stage III patients.21,29 In stage II rectal cancer, for which radiotherapy primarily impacts local recurrence, pooled data from adjuvant therapy trials demonstrate that patho-logically staged T3N0 rectal cancers may be intermediate risk and therefore may not require adjuvant radiotherapy.30 However, it may be premature to omit radiotherapy for cT3N0 tumors given the uncertainties of clinical staging. Chemotherapy for rectal cancer is also controversial; a randomized study showed no survival benefit with adjuvant FU.31 The adjuvant chemotherapy guidelines for rectal cancer are based on extrapolations from colon cancer trials.

We surveyed a geographically dispersed, representative population of US medical oncologists, radiation oncologists, colo-rectal surgeons, and general surgeons to understand how physicians who treat colorectal cancer perceive the balance of benefits and harms of adjuvant therapy. We also examined how physician and practice characteristics correlated with the more var-ied beliefs about the benefits of adjuvant therapy for stage II colon and rectal cancer.

■■ Methods

■■ DesignThe Cancer Care Outcomes Research and Surveillance Consortium (CanCORS) col-lected information from patient surveys,

medical records, and physician surveys for approximately 5,000 patients with colorec-tal cancer diagnosed during 2003 to 2005 in Northern California, Los Angeles County, North Carolina, Iowa, or Alabama or who received care in one of five large health maintenance organizations or 15 Veterans Affairs Medical Centers.32,33 This analysis used only physician survey data. The study was approved by human subjects commit-tees at all participating institutions.

■■ PopulationAs described previously,9 physicians named by patients as providing important roles in their care were surveyed from 2004 to 2007 (97% of surveys were mailed between January 2005 and May 2006). Contact information for 6,871 physicians was verified, and 4,188 (61.0%) responded. Respondents did not differ by sex (P = .97). Radiation oncologists and those who grad-uated from medical school before 1976 or after 1989 versus 1976 to 1989 responded more frequently (both P ≤ .005).

We restricted the sample to the 1,382 physicians who self-identified as sur-geons, radiation oncologists, or medical oncol-ogists; cared for more than one patient with colorectal cancer in the past year; and were not still in training. We focused on the 1,296 physicians with complete data on the six ques-tions of primary interest (described below).

■■ SurveyTo assess physicians’ beliefs about the benefits versus risks of adjuvant colorectal cancer therapies, each physician was asked, “For an otherwise healthy 55-year-old man with colorectal cancer, how likely is it that the benefits outweigh the risks for each of the following treatments? (1) adjuvant chemotherapy for stage II colon cancer, (2) adjuvant chemotherapy for stage III colon cancer, (3) adjuvant chemotherapy for stage II rectal cancer, (4) adjuvant chemotherapy for stage III rectal cancer, (5) adjuvant radiotherapy for stage II rectal cancer, (6) adjuvant radiotherapy for stage III rectal cancer.” Physicians responded “very

Key points

•On high-quality randomized controlled trials that have demonstrated that adjuvant chemotherapy for stage III colon cancer and neoadjuvant radiation with or without chemotherapy for stage III rectal cancer improve outcomes.

•For stage II colon cancer, studies that assessed survival after adjuvant fluorouracil (FU) chemotherapy after curative resection have been mixed, and no overall survival benefit for adjuvant folinic acid (leucovorin)-FU-oxaliplatin (FOLFOX) chemotherapy has been found.

• In stage II rectal cancer, for which radiotherapy primarily impacts local recurrence, pooled data from adjuvant therapy trials demonstrate that pathologically staged T3N0 rectal cancers may be intermediate risk and therefore may not require adjuvant radio-therapy.

•Chemotherapy for rectal cancer is also controversial; a randomized study showed no survival benefit with adjuvant FU.

•The adjuvant chemotherapy guidelines for rectal cancer are based on extrapolations from colon cancer trials.

•Physician surveys for approximately 5,000 patients with colorectal cancer diagnosed during 2003 to 2005 in Northern California.

•We focused on the 1,296 physicians with complete data on the six questions of primary interest.

ASCO-CRC-2015-V2-INT.indb 36 4/8/2015 4:41:40 PM

Copyright by MT

Page 41: › medical_mx › images › ... · 5 Samantha Hendren; Ellen McKeown; Arden M . Morris; Sandra L . Wong; Mary Oerline; Lyndia Poe; Darrell A . Campbell Jr, and Nancy J . Birkmeyer

Physicians’ Beliefs About the Benefits

37

Key points

•For most variables, item nonresponse ranged from less than 2% to 3%. No subjects were missing the dependent variables of interest on the basis of the inclusion criteria described.

•We examined physician and practice characteristics associated with believing that the benefits very likely outweigh the risks of adjuvant therapy for stage II colorectal cancer (nearly all physicians endorsed the net benefits of adjuvant therapies for stage III disease).

•We used logistic regression to evaluate the association of independent variables with P < .20 in bivariable analyses with any of the therapies with the likelihood of responding that benefits very likely outweigh the risks of adjuvant therapy.

•There was consensus among physicians that the benefits of adjuvant therapy outweigh the risks for an otherwise healthy middle-aged patient with stage III colorectal cancer, with nearly all indicating that the benefits of adjuvant chemotherapy in stage III colon cancer (92%), and adjuvant chemotherapy (93%) and radiation (90%) for stage III rectal cancer were very likely to outweigh the risks.

•There was weaker consensus regarding stage II colon and rectal cancers.

unlikely,” “somewhat unlikely,” “somewhat likely,” “very likely,” or “don’t know.” Physi-cians also reported specialty, age, United States/Canadian medical graduate status, practice site, whether they practice at a National Cancer Institute (NCI)–designated cancer center, number of colorectal cancer patients cared for in the last month, whether they enroll patients onto clinical trials, teach-ing involvement, attendance at tumor board meetings, percentage of patients in man-aged care, and base clinical income. Vari-ables were categorized as shown in Table 1.

■■ Statistical analysisFor most variables, item nonresponse ranged from less than 2% to 3%. No sub-jects were missing the dependent variables of interest on the basis of the inclusion criteria described above. We used multiple imputation to impute missing data.34,35 We categorized physicians’ reports about the relative benefits outweighing risks of each treatment as “very likely” versus “some-what likely/somewhat unlikely/very unlikely/don’t know.” “Don’t know” responses were ≤ 3% for all scenarios.

We examined physician and practice char-acteristics associated with believing that the benefits very likely outweigh the risks of adjuvant therapy for stage II colorectal can-cer (nearly all physicians endorsed the net benefits of adjuvant therapies for stage III disease). We used logistic regression to evaluate the association of independent variables with P < .20 in bivariable analyses with any of the therapies with the likeli-hood of responding that benefits very likely outweigh the risks of adjuvant therapy. All tests of statistical significance were two-sided. Statistical analyses were performed using the SAS version 9.2 statistical pack-age (SAS Institute, Cary, NC).

■■ Results

The 1,296 physicians were medical oncologists (36%), general surgeons (40%), radiation oncologists (16%), and surgeons

or surgical oncologists (8%); had a median age of 50 years; and saw a median of five patients with colorectal cancer per month (Table 1). On average, the survey respond-ents were 23 years post–medical school graduation, and 94% were board certified.

There was consensus among physicians that the benefits of adjuvant therapy out-weigh the risks for an otherwise healthy middle-aged patient with stage III colorectal cancer (Figure 1), with nearly all indicating that the benefits of adjuvant chemotherapy in stage III colon cancer (92%), and adju-vant chemotherapy (93%) and radiation (90%) for stage III rectal cancer were very likely to outweigh the risks.

There was weaker consensus regarding stage II colon and rectal cancers (Figure 1). Twenty-two percent and 46% of physicians reported that chemotherapy in stage II colon cancer was very likely and somewhat likely to have a net benefit, respectively. Physicians were more enthusiastic, but still divided, over adjuvant therapy in stage II rectal can-cer, with about half indicating that the ben-efits of both chemotherapy and radiotherapy were very likely to outweigh the risks.

Because nearly all physicians agreed on adjuvant therapies for stage III disease, we focused additional analyses on stage II colorectal cancer. In unadjusted and adjusted analyses, physician specialty was strongly associated with beliefs regarding the net benefit of treatment for stage II colorectal cancer (Tables 1 and 2). Com-pared with medical oncologists, radiation oncologists (odds ratio [OR] = 2.29; 95% CI, 1.39 to 3.79) and general surgeons (OR = 2.79; 95% CI, 1.77 to 4.40) were more likely to report that the benefits of chemotherapy for stage II colon cancer were very likely to outweigh the risks. For stage II rectal cancers, radiation oncologists (OR = 0.54; 95% CI, 0.36 to 0.80) and surgical oncologists (OR = 0.50; 95% CI, 0.31 to 0.79) were less likely than medical oncologists to report that the benefits of chemotherapy were very likely to outweigh

ASCO-CRC-2015-V2-INT.indb 37 4/8/2015 4:41:40 PM

Copyright by MT

Page 42: › medical_mx › images › ... · 5 Samantha Hendren; Ellen McKeown; Arden M . Morris; Sandra L . Wong; Mary Oerline; Lyndia Poe; Darrell A . Campbell Jr, and Nancy J . Birkmeyer

Syllabus

38

■■ TABLE 1 - Percentage of physicians who reported that the benefits of adjuvant therapy are very likely to outweigh the risks for stage II colon and rectal cancers

Physician characteristic

No. % Chemotherapy for stage II

colon cancer

χ2 P Chemotherapy for stage II

rectal cancer

χ2 P Radiotherapy for stage II

rectal cancer

χ2 P

All physicians 21.9 50 50.4

Specialty

Medical oncologist 466 36 14.2 < .001 56.4 < .001 62 < . 001

Radiation oncologist 211 16 23.2 41.2 44.6

Colorectal surgeon/surgical oncologist

103 8 9.7 38.8 44.7

General surgeon 516 40 30.8 50 43.4

Age, years

< 40 228 18 16.9 < .001 48.5 .1 45.9 .35

40–49 400 31 17.7 48.5 50.7

50–54 217 17 19.3 49.2 51.5

55–59 217 17 25.8 46.6 48.3

≥ 60 228 18 33.2 58.3 55.2

Missing 6 0

US medical graduate

Yes 1,088 84 21.5 .38 48.2 .002 48.6 .002

No 201 16 24.3 59.9 60.3

Missing 7 1

Practice site

Office, solo 150 12 35 < .001 53.4 .41 56.1 .27

Office, single-specialty group

346 27 21.1 53.3 48.8

Office, multi-specialty group

103 8 21.9 53.1 55.1

HMO 243 19 19.8 48.6 52.3

VA/government 102 8 11.8 47.1 52.9

Hospital 352 27 21.6 46.3 46.1

NCI cancer center

Yes 300 23 22.2 .87 49.8 .96 46.5 .12

No/don’t know 974 75 21.8 50.1 51.6

Missing 22 2

No. of patients per month

≤ 2 325 25 29.9 < .001 47 .38 46.2 < . 001

2 to 5 395 30 21.2 51.6 44.1

5 to 10 269 21 17.4 48 52.7

> 10 276 21 18.2 53.1 61.8

Missing 31 2

ASCO-CRC-2015-V2-INT.indb 38 4/8/2015 4:41:41 PM

Copyright by MT

Page 43: › medical_mx › images › ... · 5 Samantha Hendren; Ellen McKeown; Arden M . Morris; Sandra L . Wong; Mary Oerline; Lyndia Poe; Darrell A . Campbell Jr, and Nancy J . Birkmeyer

Physicians’ Beliefs About the Benefits

39

risks, and radiation oncologists (OR = 0.59; 95% CI, 0.40 to 0.88), surgeons (OR = 0.54; 95% CI, 0.38 to 0.77), and surgical oncolo-gists (OR = 0.50; 95% CI, 0.32 to 0.81) were less likely than medical oncologists to

report that the benefits of radiation were very likely to outweigh risks (Table 2).

Physician age was also significantly associ-ated with beliefs about the net benefit of

Physician characteristic

No. % Chemotherapy for stage II

colon cancer

χ2 P Chemotherapy for stage II

rectal cancer

χ2 P Radiotherapy for stage II

rectal cancer

χ2 P

Enroll patients onto clinical trials

Yes 719 55 18.2 < .001 50.8 .52 53.9 .002

No 523 40 27.3 48.9 45.2

Missing 54 4

Teaching

None 654 50 21.8 .18 50.7 .56 49.7 .61

1–5 d/mo 289 22 25.5 51.3 53

≥ 6 d/mo 332 26 19.1 47.4 49.5

Missing 21 2

Attend tumor board

Weekly 705 54 18.3 < .001 48.8 .64 51.2 .78

Monthly 326 25 22.4 50.7 49.9

Quarterly or less often

250 19 31.3 52.3 48.7

Missing 15 1

% patients in managed care

0–20 319 25 20.2 .15 51.5 .8 51.6 .68

21–49 267 21 21.9 50.8 47

50–78 316 24 26.1 50.5 51

79–100 286 22 19.4 47.5 51.3

Missing 108 8

Base clinical income

Mostly fee-for-service 485 37 28.5 .002 51.8 .79 50.4 .71

Mixture fee-for-service and capitation

332 26 17.2 47.6 42.5

Salary, productivity based

391 30 18.9 48.5 51.6

Salary, not productivity based

42 3 19.4 50.5 49.7

Missing 46 4

HMO, health maintenance organization; VA, Veterans Affairs; NCI, National Cancer Institute.

■■ TABLE 1 - Percentage of physicians who reported that the benefits of adjuvant therapy are very likely to outweigh the risks for stage II colon and rectal cancers (continued )

ASCO-CRC-2015-V2-INT.indb 39 4/8/2015 4:41:41 PM

Copyright by MT

Page 44: › medical_mx › images › ... · 5 Samantha Hendren; Ellen McKeown; Arden M . Morris; Sandra L . Wong; Mary Oerline; Lyndia Poe; Darrell A . Campbell Jr, and Nancy J . Birkmeyer

Syllabus

40

adjuvant therapy. Younger physicians were less likely than older physicians to believe that the benefits of adjuvant therapies for stage II colorectal cancers were very likely to outweigh the risks (Table 2).

Physicians who were not US medical graduates were more likely to endorse the net benefit of adjuvant chemotherapy for stage II rectal cancer, but graduation from a non-US medical institution was not associated with beliefs regarding chemotherapy for stage II colon cancer or radiotherapy for stage II rectal cancer.

Regarding practice setting, compared with physicians in Veterans Affairs Medi-cal Centers, those in office-based solo practices (OR = 2.66; 95% CI, 1.16 to 6.09) were more likely to believe that the benefits very likely outweigh the risks of adjuvant chemotherapy for stage II colon cancer. Physicians in single-specialty (OR = 2.14; 95% CI, 1.00 to 4.59) and multispecialty office-based practices (OR = 2.37; 95% CI, 1.00 to 5.65) were also more likely than physicians in Veterans Affairs settings to believe that the benefits of chemotherapy for stage II colon cancer

FIGURE 1 ■■■Unadjusted frequencies of physician responses regarding the likelihood of benefits outweighing the risks of adju-vant therapy in an asymptomatic, otherwise healthy 55-year-old male patient with colon or rectal cancer. (A) medical oncolo-gists, (B) radiation oncologists, (C) surgical oncologists or colorectal surgeons, (D) general surgeons.

RT, radiotherapy.

Key points

•Younger physicians were less likely than older physicians to believe that the benefits of adjuvant therapies for stage II colorectal cancers were very likely to outweigh the risks.

•Physicians who were not US medical graduates were more likely to endorse the net benefit of adjuvant chemotherapy for stage II rectal cancer.

ASCO-CRC-2015-V2-INT.indb 40 4/8/2015 4:41:41 PM

Copyright by MT

Page 45: › medical_mx › images › ... · 5 Samantha Hendren; Ellen McKeown; Arden M . Morris; Sandra L . Wong; Mary Oerline; Lyndia Poe; Darrell A . Campbell Jr, and Nancy J . Birkmeyer

Physicians’ Beliefs About the Benefits

41

■■ TABLE 2 - Adjusted odds ratios and 95% CIs for physicians who reported benefit of adjuvant therapy very likely to outweigh risk for stage II colon and rectal cancers

Characteristic Chemotherapy for stage II colon cancer

Chemotherapy for stage II rectal cancer

Radiotherapy for stage II rectal cancer

OR CI P OR CI P OR CI P

Specialty

Radiation oncologist 2.29 1.39 to 3.79 .001 0.54 0.36 to 0.80 .002 0.59 0.40 to 0.88 .009

General surgery 2.79 1.77 to 4.40 < .001 0.76 0.54 to 1.09 .13 0.54 0.38 to 0.77 < .001

Colorectal surgeon/surgical oncologist

0.68 0.32 to 1.44 .31 0.50 0.31 to 0.79 .004 0.50 0.32 to 0.81 .004

Medical oncologist 1.0 1.0 1.0

Age, years

< 40 0.48 0.30 to 0.78 .003 0.65 0.44 to 0.96 .03 0.66 0.45 to 0.98 .04

40–49 0.49 0.33 to 0.74 .001 0.66 0.47 to 0.93 .02 0.85 0.60 to 1.20 .36

50–54 0.50 0.32 to 0.80 .003 0.64 0.43 to 0.93 .02 0.79 0.54 to 1.17 .24

55–59 0.69 0.45 to 1.07 .10 0.56 0.38 to 0.83 .004 0.68 0.46 to 1.00 .048

≥ 60 1.0 1.0 1.0

US medical graduate

Yes 1.0 1.0 1.0

No 1.27 0.86 to 1.89 .23 1.60 1.16 to 2.21 .004 1.38 1.00 to 1.92 .053

Practice type

VA/government 1.0 1.0 1.0

HMO 1.85 0.85 to 4.00 .12 1.26 0.73 to 2.16 .40 1.06 0.62 to 1.82 .82

Office, solo 2.66 1.16 to 6.09 .02 1.06 0.57 to 1.98 .85 1.21 0.65 to 2.26 .55

Office, single-specialty group

2.14 1.00 to 4.59 .051 1.28 0.75 to 2.19 .36 0.86 0.50 to 1.47 .58

Office, multispecialty group

2.37 1.00 to 5.65 .051 1.31 0.70 to 2.46 .40 1.18 0.63 to 2.21 .61

Hospital 1.89 0.92 to 3.88 .08 1.17 0.71 to 1.91 .54 0.99 0.60 to 1.62 .97

NCI cancer center

Yes 1.0 1.0 1.0

No or don’t know 0.89 0.63 to 1.27 .52 0.92 0.70 to 1.22 .58 1.17 0.88 to 1.56 .27

No. of colorectal patients per month

≤ 12 0.81 0.49 to 1.33 .41 0.92 0.62 to 1.34 .65 0.72 0.48 to 1.08 .12

> 2 and ≤ 5 0.65 0.41 to 1.03 .07 1.10 0.78 to 1.55 .59 0.60 0.42 to 0.85 .005

> 5 and ≤ 10 0.65 0.39 to 1.07 .09 0.89 0.63 to 1.26 .50 0.79 0.55 to 1.14 .20

> 10 1.0 1.0 1.0

Enroll patients onto clinical trials

Yes 1.0 1.0 1.0

No 1.00 0.71 to 1.43 .98 1.02 0.76 to 1.36 .92 0.89 0.67 to 1.20 .44

ASCO-CRC-2015-V2-INT.indb 41 4/8/2015 4:41:41 PM

Copyright by MT

Page 46: › medical_mx › images › ... · 5 Samantha Hendren; Ellen McKeown; Arden M . Morris; Sandra L . Wong; Mary Oerline; Lyndia Poe; Darrell A . Campbell Jr, and Nancy J . Birkmeyer

Syllabus

42

outweighed risks, but these differences were not statistically significant.

NCI cancer center status, clinical trials par-ticipation, tumor board participation, pro-portion of patients in managed care, and base clinical income were not associated with beliefs about the relative risk and ben-efits of adjuvant therapy for stage II colon or rectal cancers (Table 2).

■■ Discussion

This large, multiregional study of US oncolo-gists and surgeons demonstrated widespread

consensus among physicians that the benefits of adjuvant chemotherapy and chemora-diotherapy outweigh the risks for healthy, middle-aged patients with stage III colon and rectal cancer, consistent with the strong evi-dence from randomized trials and guideline recommendations. In contrast, physicians had divergent opinions about the net benefit of adjuvant therapies for stage II colorectal cancers. The limited endorsement of adjuvant chemotherapy for patients with stage II colon cancer likely reflects the unclear evidence of benefit in this setting17,18,20 and varied inter-pretations of existing data by individual physi-cians. Some physicians may believe that the relative benefit of adjuvant therapy in stage III

Characteristic Chemotherapy for stage II colon cancer

Chemotherapy for stage II rectal cancer

Radiotherapy for stage II rectal cancer

OR CI P OR CI P OR CI P

Teaching

No teaching 0.80 0.53 to 1.21 .28 1.00 0.72 to 1.37 .98 0.92 0.67 to 1.26 .60

Teach 1–5 d/mo 1.25 0.80 to 1.95 .33 1.00 0.71 to 1.42 .99 0.94 0.66 to 1.33 .72

≥ 6 d/mo 1.0 1.0 1.0

Attend tumor board

Weekly 1.0 1.0 1.0

Monthly 0.96 0.67 to 1.39 .85 1.06 0.79 to 1.42 .69 1.09 0.81 to 1.46 .58

Quarterly or less often 1.40 0.94 to 2.09 .10 1.15 0.81 to 1.64 .42 1.20 0.85 to 1.70 .30

% patients in managed care

0–20 1.00 0.60 to 1.66 .99 1.19 0.81 to 1.75 .37 0.99 0.66 to 1.47 .95

21–49 1.14 0.69 to 1.90 .60 1.17 0.79 to 1.73 .43 0.83 0.55 to 1.24 .35

50–78 1.22 0.76 to 1.95 .41 1.18 0.80 to 1.74 .39 1.08 0.75 to 1.57 .67

79–100 1.0 1.0 1.0

Base clinical income

Mixture fee-for-service and capitation

1.0 1.0 1.0

Salary, productivity based

1.48 0.61 to 3.57 .38 1.08 0.56 to 2.07 .82 1.52 0.79 to 2.92 .21

Salary, not productivity based

1.19 0.49 to 2.85 .70 1.05 0.55 to 2.01 .88 1.46 0.75 to 2.81 .26

Mostly fee-for-service 1.70 0.71 to 4.05 .23 1.09 0.57 to 2.09 .79 1.47 0.76 to 2.86 .25

HMO, health maintenance organization; NCI, National Cancer Institute; OR, odds ratio; VA, Veterans Affairs.

■■ TABLE 2 - Adjusted odds ratios and 95% CIs for physicians who reported benefit of adjuvant therapy very likely to outweigh risk for stage II colon and rectal cancers (continued )

Key points

•Multiregional study of US oncologists and surgeons demonstrated widespread consensus among physicians that the benefits of adjuvant chemotherapy and chemoradiotherapy outweigh the risks for healthy, middle-aged patients with stage III colon and rectal cancer.

ASCO-CRC-2015-V2-INT.indb 42 4/8/2015 4:41:41 PM

Copyright by MT

Page 47: › medical_mx › images › ... · 5 Samantha Hendren; Ellen McKeown; Arden M . Morris; Sandra L . Wong; Mary Oerline; Lyndia Poe; Darrell A . Campbell Jr, and Nancy J . Birkmeyer

Physicians’ Beliefs About the Benefits

43

patients may extend to stage II patients, yet perhaps as a result of small samples in rand-omized trials, the absolute benefit cannot be detected. Alternatively, trials have not clearly demonstrated a survival advantage, so physi-cians may be reluctant to recommend therapy without well-established and/or sizeable benefits.

Guidelines also recommend chemotherapy and radiation for all patients with stage II rectal cancer, even those without high-risk features; however, only half of physicians in our study believed that chemotherapy and radiation were very likely to have net benefit in this setting. Some experts have suggested that low-risk stage II, as well as stage II or III disease located high in the rectum, might be adequately treated by surgery and chem-otherapy alone.36,37 Moreover, the benefit of radiotherapy has primarily been in local control rather than in overall survival. These perspectives may explain the incomplete adoption of neoadjuvant chemoradio-therapy for locally advanced rectal cancers; in 2006, only 60% of patients with stage II or III rectal cancer received radiotherapy.38,39 The benefit of chemoradiotherapy over chemotherapy alone in stage II or III rectal cancer is currently being studied in the Alli-ance N1048 phase II/III randomized trial (clinicaltrials.gov NCT01515787).

The additional benefit of adjuvant chemo-therapy in stage II rectal cancer, particularly in patients with a good response to chemo-radiotherapy, is controversial. When chemo-radiotherapy is delivered preoperatively, postoperative adjuvant chemotherapy is not always administered.2,6 We found, some-what paradoxically, that medical oncologists were more likely than radiation oncologists and surgeons to endorse the net benefits of adjuvant chemotherapy and radiotherapy for stage II rectal cancers, but less likely to endorse adjuvant chemotherapy for stage II colon cancer. Because radiation oncologists and general surgeons are less often involved in treating colon cancer patients, they may be less familiar with the limited indications for chemotherapy in stage II disease.

We also observed that older physicians were more likely than their younger col-leagues to endorse the net benefit of adjuvant therapies for stage II colorectal cancer. Physicians in office-based solo prac-tices were more likely than Veterans Affairs physicians to believe that chemotherapy has net benefit for stage II colon cancer, potentially resulting from differences in the patients they see or financial incentives for providing chemotherapy, although we found no association with the structure of physicians’ base clinical income. Also, foreign medical graduates were more likely than US graduates to endorse the net benefit of chemotherapy for patients with stage II rectal cancer, although they did not differ from US graduates in other scenarios.

This study has several limitations. First, phy-sicians reported their beliefs about adjuvant therapies for hypothetical patients. Clini-cal vignettes, however, have been previ-ously validated as a method for studying clinical practice.40 The study is also subject to nonresponse bias, although response rates were relatively high. Furthermore, the survey questions themselves did not include details about certain clinical con-siderations such as preoperative versus postoperative timing of adjuvant therapy, and high– versus low-risk features in stage II disease or details of the adjuvant regimens (eg, whether they contained oxaliplatin, an agent that considerably augments toxicity).19 In addition, the lack of consen-sus about the benefits of chemotherapy and radiation for stage II rectal cancer may reflect the preference for administration of chemoradiotherapy in the neoadjuvant setting after publication of a randomized trial that compared pre- and postoperative treatment in 2004.24 The survey questions did not permit distinction between physi-cians whose beliefs were influenced by such nuances in the literature and those who lacked knowledge.

Our findings are based on data mostly collected between January 2005 and May 2006, which may limit their applicability

Key points

•Trials have not clearly demonstrated a survival advantage, so physicians may be reluctant to recommend therapy without well-established and/or sizeable benefits.

•Guidelines recommend chemotherapy and radiation for all patients with stage II rectal cancer, even those without high-risk features.

•Half of physicians in our study believed that chemotherapy and radiation were very likely to have net benefit in this setting.

•Some experts have suggested that low-risk stage II, as well as stage II or III disease located high in the rectum, might be adequately treated by surgery and chemotherapy alone.

•The benefit of chemoradiotherapy over chemotherapy alone in stage II or III rectal cancer is currently being studied in the Alliance N1048 phase II/III randomized trial (clinicaltrials.gov NCT01515787).

•We observed that older physicians were more likely than their younger col leagues to endorse the net benefit of adjuvant therapies for stage II colorectal cancer.

•The survey questions did not permit distinction between physi cians whose beliefs were influenced by such nuances in the literature and those who lacked knowledge.

ASCO-CRC-2015-V2-INT.indb 43 4/8/2015 4:41:41 PM

Copyright by MT

Page 48: › medical_mx › images › ... · 5 Samantha Hendren; Ellen McKeown; Arden M . Morris; Sandra L . Wong; Mary Oerline; Lyndia Poe; Darrell A . Campbell Jr, and Nancy J . Birkmeyer

Syllabus

44

to current clinical practice. For example, clinicians today occasionally use molecular testing and genetic profiling to stratify risk of recurrence, and these assays were not routinely performed when our sur-vey was administered.41 Nevertheless, the results of several landmark trials of adjuvant therapies in locally advanced colorectal cancer19,22,24,26 were avail-able when our survey was conducted, and practice guidelines indicating which patients should receive chemotherapy and radiation were already established and have not changed substantially since then. Future work will determine the extent to which genetic and molecular information influences physician recommendations for adjuvant therapy.

In conclusion, we found nearly all physi-cians agreed with evidence-based guide-lines regarding the net benefit of adjuvant therapy for stage III colorectal cancers, but there was no consensus about the

treatment of stage II tumors. Our results suggest that, in the absence of strong evi-dence from clinical trials, individual physi-cian characteristics, such as age, specialty, and practice type, may inappropriately play a role in determining adjuvant treatment. Although clinical trials large enough to establish a survival advantage for stage II colorectal cancer are unlikely, studies using molecular profiling to identify the highest risk patients may help to refine treatment recommendations. The disagreement over management of stage II disease also sug-gests a role for a formal consensus-making process, such as the RAND/UCLA Appro-priateness Methodology, to develop qual-ity indicators in the delivery of colorectal cancer adjuvant therapy. In the absence of such consensus guidelines, given the variability in the beliefs of physicians with different specialty training and experience, multidisciplinary input may help to optimize decisions about adjuvant treatment for patients with stage II colorectal cancer.

Key points

• In conclusion, we found nearly all physicians agreed with evidence-based guidelines regarding the net benefit of adjuvant therapy for stage III colorectal cancers, but there was no consensus about the treatment of stage II tumors.

•Results suggest that, in the absence of strong evidence from clinical trials, individual physician characteristics, such as age, specialty, and practice type, may inappropriately play a role in determining adjuvant treatment. 

■■ References 1. Sinclair AH, Schymura MJ, Boscoe FP, et al: Measuring

colorectal cancer care quality for the publicly insured in New York State. Cancer Med 1:363–71, 2012.

2. Abraham NS, Gossey JT, Davila JA, et al: Receipt of recommended therapy by patients with advanced colorectal cancer. Am J Gastroenterol 101:1320–1328, 2006.

3. Cronin DP, Harlan LC, Potosky AL, et al: Patterns of care for adjuvant therapy in a random population-based sample of patients diagnosed with colorectal cancer. Am J Gastroenterol 101:2308–2318, 2006.

4. Chagpar R, Xing Y, Chiang YJ, et al: Adherence to stage-specific treatment guidelines for patients with colon cancer. J Clin Oncol 30:972–979, 2012.

5. Romanus D, Weiser MR, Skibber JM, et al: Concordance with NCCN colorectal cancer guidelines and ASCO/NCCN quality measures: An NCCN institutional analysis. J Natl Compr Canc Netw 7:895–904, 2009.

6. Khrizman P, Niland JC, ter Veer A, et al: Postoperative adjuvant chemotherapy use in patients with stage II/III rectal cancer treated with neoadjuvant therapy: A National Comprehensive Cancer Network analysis. J Clin Oncol 31:30–38, 2013.

7. Boland GM, Chang GJ, Haynes AB, et al: Association between adherence to National Comprehensive Cancer Network treatment guidelines and improved survival in patients with colon cancer. Cancer 119:1593–1601, 2013.

8. Sanoff HK, Carpenter WR, Stürmer T, et al: Effect of adjuvant chemotherapy on survival of patients with stage III colon

cancer diagnosed after age 75 years. J Clin Oncol 30: 2624–2634, 2012.

9. Keating NL, Landrum MB, Klabunde CN, et al: Adjuvant chemotherapy for stage III colon cancer: Do physicians agree about the importance of patient age and comorbidity? J Clin Oncol 26:2532–2537, 2008.

10. Kahn KL, Adams JL, Weeks JC, et al: Adjuvant chemotherapy use and adverse events among older patients with stage III colon cancer. JAMA 303:1037–1045, 2010.

11. Sanders T, Skevington S: Do bowel cancer patients participate in treatment decision-making? Findings from a qualitative study. Eur J Cancer Care (Engl) 12:166–175, 2003.

12. National Comprehensive Cancer Network: NCCN Clinical Practice Guidelines in Oncology: Colon Cancer (version 3.2014). http://www.nccn.org/professionals/physician_gls/pdf/colon.pdf.

13. National Comprehensive Cancer Network. NCCN Clinical Practice Guidelines in Oncology: Rectal Cancer (version 3.2014). http://www.nccn.org/professionals/physician_gls/pdf/rectal.pdf.

14. Petersen VC, Baxter KJ, Love SB, et al: Identification of objective pathological prognostic determinants and models of prognosis in Dukes’ B colon cancer. Gut 51:65–69, 2002.

15. Quah HM, Chou JF, Gonen M, et al: Identification of patients with high-risk stage II colon cancer for adjuvant therapy. Dis Colon Rectum 51:503–507, 2008.

16. Figueredo A, Coombes ME, Mukherjee S: Adjuvant therapy for completely resected stage II colon cancer. Cochrane Database Syst Rev:CD005390, 2008.

ASCO-CRC-2015-V2-INT.indb 44 4/8/2015 4:41:42 PM

Copyright by MT

Page 49: › medical_mx › images › ... · 5 Samantha Hendren; Ellen McKeown; Arden M . Morris; Sandra L . Wong; Mary Oerline; Lyndia Poe; Darrell A . Campbell Jr, and Nancy J . Birkmeyer

Physicians’ Beliefs About the Benefits

45

17. Efficacy of adjuvant fluorouracil and folinic acid in colon cancer. International Multicentre Pooled Analysis of Colon Cancer Trials (IMPACT) investigators. Lancet 345:939–944, 1995.

18. Efficacy of adjuvant fluorouracil and folinic acid in B2 colon cancer. International Multicentre Pooled Analysis of B2 Colon Cancer Trials (IMPACT B2) Investigators. J Clin Oncol 17:1356–1363, 1999.

19. André T, Boni C, Navarro M, et al: Improved overall survival with oxaliplatin, fluorouracil, and leucovorin as adjuvant treatment in stage II or III colon cancer in the MOSAIC trial. J Clin Oncol 27:3109–3116, 2009.

20. Quasar Collaborative Group, Gray R, Barnwell J, et al: Adjuvant chemotherapy versus observation in patients with colorectal cancer: A randomised study. Lancet 370:2020–2029, 2007.

21. Yothers G, O’Connell MJ, Allegra CJ, et al: Oxaliplatin as adjuvant therapy for colon cancer: Updated results of NSABP C-07 trial, including survival and subset analyses. J Clin Oncol 29:3768–3774, 2011.

22. Kapiteijn E, Marijnen CA, Nagtegaal ID, et al: Preoperative radiotherapy combined with total mesorectal excision for resectable rectal cancer. N Engl J Med 345:638–646, 2001.

23. van Gijn W, Marijnen CA, Nagtegaal ID, et al: Preoperative radiotherapy combined with total mesorectal excision for resectable rectal cancer: 12-year follow-up of the multicentre, randomised controlled TME trial. Lancet Oncol 12:575–582, 2011.

24. Sauer R, Becker H, Hohenberger W, et al: Preoperative versus postoperative chemoradiotherapy for rectal cancer. N Engl J Med 351:1731–1740, 2004.

25. Ceelen W, Fierens K, Van Nieuwenhove Y, et al: Preoperative chemoradiation versus radiation alone for stage II and III resectable rectal cancer: A systematic review and meta-analysis. Int J Cancer 124:2966–2972, 2009.

26. Bosset JF, Collette L, Calais G, et al: Chemotherapy with preoperative radiotherapy in rectal cancer. N Engl J Med 355:1114–1123, 2006.

27. Cercek A, Weiser MR, Goodman KA, et al: Complete pathologic response in the primary of rectal or colon cancer treated with FOLFOX without radiation. J Clin Oncol 28, 2010 (suppl; abstr 3649).

28. Wu X, Zhang J, He X, et al: Postoperative adjuvant chemotherapy for stage II colorectal cancer: A systematic review of 12 randomized controlled trials. J Gastrointest Surg 16:646–55, 2012.

29. Tournigand C, André T, Bonnetain F, et al: Adjuvant therapy with fluorouracil and oxaliplatin in stage II and elderly patients (between ages 70 and 75 years) with colon cancer: Subgroup analyses of the Multicenter International Study of Oxaliplatin, Fluorouracil, and Leucovorin in the Adjuvant Treatment of Colon Cancer trial. J Clin Oncol 30:3353–3360, 2012.

30. Wo JY, Mamon HJ, Ryan DP, et al: T3N0 rectal cancer: Radiation for all? Semin Radiat Oncol 21:212–219, 2011.

31. Tepper JE, O’Connell M, Niedzwiecki D, et al: Adjuvant therapy in rectal cancer: Analysis of stage, sex, and local control–Final report of intergroup 0114. J Clin Oncol 20:1744–1750, 2002.

32. Ayanian JZ, Chrischilles EA, Fletcher RH, et al: Understanding cancer treatment and outcomes: The Cancer Care Outcomes Research and Surveillance Consortium. J Clin Oncol 22: 2992–2996, 2004.

33. National Cancer Institute: Cancer Care Outcomes Research and Surveillance Consortium. http://appliedresearch.cancer.gov/cancors

34. He Y, Zaslavsky AM, Harrington DP, et al: Multiple imputation in a large-scale complex survey: A practical guide. Stat Methods Med Res 19:653–670, 2010.

35. Little RJ: Statistical Analysis with Missing Data. New York, NY, Wiley, 1986.

36. Lai LL, Fuller CD, Kachnic LA, et al: Can pelvic radiotherapy be omitted in select patients with rectal cancer? Semin Oncol 33:S70–S74, 2006.

37. Gunderson LL, Sargent DJ, Tepper JE, et al: Impact of T and N stage and treatment on survival and relapse in adjuvant rectal cancer: A pooled analysis. J Clin Oncol 22:1785–96, 2004.

38. Baxter NN, Rothenberger DA, Morris AM, et al: Adjuvant radiation for rectal cancer: Do we measure up to the standard of care? An epidemiologic analysis of trends over 25 years in the United States. Dis Colon Rectum 48:9–15, 2005.

39. Mak RH, McCarthy EP, Das P, et al: Adoption of preoperative radiation therapy for rectal cancer from 2000 to 2006: A Surveillance, Epidemiology, and End Results patterns-of-care study. Int J Radiat Oncol Biol Phys 80:978–984, 2011.

40. Peabody JW, Luck J, Glassman P, et al: Measuring the quality of physician practice by using clinical vignettes: A prospective validation study. Ann Intern Med 141:771–780, 2004.

41. Benson AB 3rd, Hamilton SR: Path toward prognostication and prediction: An evolving matrix. J Clin Oncol 29: 4599–4601, 2011.

ASCO-CRC-2015-V2-INT.indb 45 4/8/2015 4:41:42 PM

Copyright by MT

Page 50: › medical_mx › images › ... · 5 Samantha Hendren; Ellen McKeown; Arden M . Morris; Sandra L . Wong; Mary Oerline; Lyndia Poe; Darrell A . Campbell Jr, and Nancy J . Birkmeyer

ASCO-CRC-2015-V2-INT.indb 46 4/8/2015 4:41:42 PM

Copyright by MT

Page 51: › medical_mx › images › ... · 5 Samantha Hendren; Ellen McKeown; Arden M . Morris; Sandra L . Wong; Mary Oerline; Lyndia Poe; Darrell A . Campbell Jr, and Nancy J . Birkmeyer

■■ Influential Papers

ASCO-CRC-2015-V2-INT.indb 47 4/8/2015 4:41:42 PM

Copyright by MT

Page 52: › medical_mx › images › ... · 5 Samantha Hendren; Ellen McKeown; Arden M . Morris; Sandra L . Wong; Mary Oerline; Lyndia Poe; Darrell A . Campbell Jr, and Nancy J . Birkmeyer

ASCO-CRC-2015-V2-INT.indb 48 4/8/2015 4:41:42 PM

Copyright by MT

Page 53: › medical_mx › images › ... · 5 Samantha Hendren; Ellen McKeown; Arden M . Morris; Sandra L . Wong; Mary Oerline; Lyndia Poe; Darrell A . Campbell Jr, and Nancy J . Birkmeyer

49

Influential Papers

1. Massarweh NN, Hu CY, You YN, Bednarski BK, Rodriguez-Bigas MA, Skibber JM, Cantor SB, Cormier JN, Feig BW, Chang GJ. Risk-adjusted pathologic margin positivity rate as a quality indicator in rectal cancer surgery. J Clin Oncol 2014;32(27):2967–74.

■■ PURPOSE: Margin positivity after rectal cancer resection is associated with poorer outcomes. We previously devel-oped an instrument for calculating hospital risk-adjusted margin positivity rate (RAMP) that allows identification of performance-based outliers and may represent a rec-tal cancersurgery quality metric. METHODS: This was an observational cohort study of patients with rectal cancer within the National Cancer Data Base (2003 to 2005). Hospital performance was categorized as low outlier (bet-ter than expected), high outlier (worse than expected), or non-RAMP outlier using standard observed-to-expected methodology. The association between outlier status and overall risk of death at 5 years was evaluated using Cox shared frailty modeling. RESULTS: Among 32,354 patients with cancer (mean age, 63.8 +/-13.2 years; 56.7% male; 87.3% white) treated at 1,349 hospitals (4.9% high outlier, 0.7% low outlier), 5.6% of patients were treated at high outliers and 3.0% were treated at low outliers. Various structural (academic status and volume), process (pathologic nodal evaluation and neoadjuvant radiation therapy use), and outcome (sphincter preservation, read-mission, and 30-day postoperative mortality) measures were significantly associated with outlier status. Five-year overall survival was better at low outliers (79.9%) com-pared with high outliers (64.9%) and nonoutliers (68.9%; log-rank test, P < .001). Risk of death was lower at low outliers compared with high outliers (hazard ratio [HR], 0.61; 95% CI, 0.50 to 0.75) and nonoutliers (HR, 0.69; 95% CI, 0.57 to 0.83). Risk of death was higher at high outliers compared with nonoutliers (HR, 1.12; 95% CI, 1.03 to 1.23). CONCLUSION: Hospital RAMP outlier status is a rectal cancer surgery composite metric that reliably captures hospital quality across all levels of care and could be integrated into existing quality improvement initiatives for hospital performance.

2. Hong YS, Nam BH, Kim KP, Kim JE, Park SJ, Park YS, Park JO, Kim SY, Kim TY, Kim JH, Ahn JB, Lim SB, Yu CS, Kim JC, Yun SH, Kim JH, Park JH, Park HC, Jung KH, Kim TW. Oxaliplatin, fluorouracil, and leucovorin versus fluorouracil and leucovorin as adjuvant chem-otherapy for locally advanced rectal cancer after pre-operative chemoradiotherapy (ADORE): an open-label, multicentre, phase 2, randomised controlled trial. Lan-cet Oncol 2014;15(11):1245–53.

■■ BACKGROUND: The role of adjuvant chemotherapy for patients with rectal cancer is controversial, especially

when used after preoperative chemoradiotherapy. Fluoropyrimidine-based adjuvant chemotherapy, including fluorouracil and leucovorin, has been widely used; how-ever, the addition of oxaliplatin to fluorouracil and leu-covorin (FOLFOX), a standard adjuvant regimen for colon cancer, has not been tested in rectal cancer. We aimed to compare the efficacy and safety of adjuvant fluorouracil and leucovorin with that of FOLFOX in patients with locally advanced rectalcancer after preoperative chemo-radiotherapy. METHODS: In this open-label, multicentre, phase 2, randomised trial, patients with postoperative pathological stage II (ypT3-4N0) or III (ypTanyN1-2) rectal cancer after preoperative fluoropyrimidine-based chemoradiotherapy and total mesorectal excision were recruited and randomly assigned (1:1) via a web-based software platform to receive adjuvant chemotherapy with either four cycles of fluorouracil and leucovorin (fluorouracil 380 mg/m(2) and leucovorin 20 mg/m(2) on days 1–5, every 4 weeks) or eight cycles of FOLFOX (oxaliplatin 85 mg/m(2), leucovorin 200 mg/m(2), and fluorouracilbolus 400 mg/m(2) on day 1, and fluorouracil infusion 2400 mg/m(2) for 46 h, every 2 weeks). Strati-fication factors were pathological stage (II vs. III) and centre. Neither patients nor investigators were masked to group assignment. The primary endpoint was 3-year disease-free survival, analysed by intention to treat. This study is fully enrolled, is in long-term follow-up, and is registered with ClinicalTrials.gov, number NCT00807911. FINDINGS: Between Nov 19, 2008, and June 12, 2012, 321 patients were randomly assigned to fluorouracil and leucovorin (n = 161) and FOLFOX (n = 160). 141 (95%) of 149 patients in the fluorouracil plus leucovorin group and 141 (97%) of 146 in the FOLFOX group completed all planned cycles of adjuvant treatment. Median follow-up was 38·2 months (IQR 26·4–50·6). 3-year disease-free survival was 71·6% (95% CI 64·6–78·6) in the FOLFOX group and 62·9% (55·4–70·4) in the fluorouracil plus leucovorin group (hazard ratio 0·657, 95% CI 0·434–0·994; p = 0·047). Any grade neutropenia, thrombocy-topenia, fatigue, nausea, and sensory neuropathy were significantly more common in the FOLFOX group than in thefluorouracil plus leucovorin group; however, we noted no significant difference in the frequency of these events at grade 3 or 4. The most common grade 3 or worse adverse events were neutropenia (38 [26%] of 149 patients in the fluorouracil plus leucovorin group vs. 52 [36%] of 146 patients in the FOLFOX group), leucopenia (eight [5%] vs. 12 [8%]), febrile neutropenia (four [3%] vs. one [< 1%]), diarrhoea (four [3%] vs. two [1%]), and nausea (one [< 1%] vs. two [1%]). INTERPRETATION: Adjuvant FOLFOX improves disease-free survival com-pared with fluorouracil plus leucovorin in patients with locally advancedrectal cancer after preoperative chemo-radiotherapy and total mesorectal excision, and warrants further investigation.

ASCO-CRC-2015-V2-INT.indb 49 4/8/2015 4:41:42 PM

Copyright by MT

Page 54: › medical_mx › images › ... · 5 Samantha Hendren; Ellen McKeown; Arden M . Morris; Sandra L . Wong; Mary Oerline; Lyndia Poe; Darrell A . Campbell Jr, and Nancy J . Birkmeyer

50

Influential Papers

3. Bosset JF, Calais G, Mineur L, Maingon P, Stojanovic-Rundic S, Bensadoun RJ, Bardet E, Beny A, Ollier JC, Bolla M, Marchal D, Van Laethem JL, Klein V, Giralt J, Clavère P, Glanzmann C, Cellier P, Collette L; EORTC Radiation Oncology Group. Fluorouracil-based adju-vant chemotherapy after preoperative chemoradio-therapy in rectal cancer: long-term results of the EORTC 22921 randomised study. Lancet Oncol 2014;15(2): 184–90.

■■ BACKGROUND: EORTC trial 22921 examined the addi-tion of preoperative or postoperative chemotherapy to preoperative radiotherapy in patients with rectal cancer. After a median follow-up of 5 years, chemotherapy-irrespective of timing-significantly improved local control. Adjuvantchemotherapy did not improve survival, but the Kaplan-Meier curves diverged, suggesting possible delayed benefit. Here, we report the updated long-term results. METHODS: We randomly assigned patients with clinical stage T3 or T4 resectable rectal cancer to receive preoperative radiotherapy with or without concomitant chemotherapy before surgery followed by either adjuvant chemotherapy or surveillance. Randomisation was done using minimisation with factors of institution, sex, T stage, and distance from the tumour to the anal verge. Study coordinators, clinicians, and patients were aware of assign-ment. Radiotherapy consisted of 45 Gy to the posterior pelvis in 25 fractions of 1·8 Gy over 5 weeks. Each course of chemotherapy consisted of fluorouracil (350 mg/m(2) per day intravenous bolus) and folinic acid (leucovorin; 20 mg/m(2) per day intravenous bolus). For preoperative chemotherapy, two courses were given (during weeks 1 and 5 of radiotherapy). Adjuvant chemotherapy was given in four cycles, every 3 weeks. The primary endpoint was overall survival. This analysis was done by intention to treat. The trial is registered with ClinicalTrials.gov, number NCT00002523. FINDINGS: 1011 patients were randomly assigned to treatment between April, 1993, and March, 2003 (252 to preoperative radiotherapy and 253 to each of the other three groups). After a median follow-up of 10·4 years (IQR 7·8–13·1), 10-year overall survival was 49·4% (95% CI 44·6–54·1) for thepreoperative radio-therapy group and 50·7% (45·9–55·2) for the preoperative radiotherapy and chemotherapy group (HR 0·99, 95% CI 0·83–1·18; p = 0·91). 10-year overall survival was 51·8% (95% CI 47·0–56·4) for the adjuvant chemotherapy group and 48·4% (43·6–53·0) for the surveillance group (HR 0·91, 95% CI 0·77–1·09, p = 0·32). 10-year disease-free survival was 44·2% (95% CI 39·5–48·8) for the preopera-tive radiotherapy group and 46·4% (41·7–50·9) for the preoperative radiotherapy and chemotherapy group (HR 0·93, 95% CI 0·79–1·10; p = 0·38). 10-year disease-free survival was 47·0% (95% CI 42·2–51·6) for the adjuvant chemotherapy group and 43·7% (39·1–48·2) for the sur-veillance group (HR 0·91, 95% CI 0·77–1·08, p = 0·29). At 10 years, cumulative incidence of local relapse was 22·4% (95% CI 17·1–27·6) with radiotherapy alone, 11·8%

(7·8–15·8) with neoadjuvant radiotherapy and chemother-apy, 14·5% (10·1–18·9) with radiotherapy and adjuvant chemotherapy and 11·7% (7·7–15·6) with bothadjuvant and neoadjuvant chemotherapy (p = 0·0017). There was no difference in cumulative incidence of distant metastases (p = 0·52). The frequency of long-term side-effects did not differ between the four groups (p = 0·22). INTERPRETA-TION: Adjuvant fluorouracil-based chemotherapy after preoperative radiotherapy (with or without chemotherapy) does not affect disease-free survival or overall survival. Our trial does not support the current practice of adjuvant chemotherapy after preoperative radiotherapy with or without chemotherapy. New treatment strategies incorpo-rating neoadjuvant chemotherapy are required.

4. Taieb J, Tabernero J, Mini E, Subtil F, Folprecht G, Van Laethem JL, Thaler J, Bridgewater J, Petersen LN, Blons H, Collette L, Van Cutsem E, Rougier P, Salazar R, Bedenne L, Emile JF, Laurent-Puig P, Lepage C; PETACC-8 Study Investigators. Oxaliplatin, fluorouracil, and leucovorin with or without cetuximab in patients with resected stage III colon cancer (PETACC-8): an open-label, randomised phase 3 trial. Lancet Oncol 2014;15(8):862–73.

■■ BACKGROUND: Since the 1990s, fluorouracil-based adjuvant chemotherapy has significantly reduced the risk of tumour recurrence in patients with stage III colon cancer. We aimed to assess whether the addition of cetuximab to standard adjuvant oxaliplatin, fluorouracil, and leucovorin chemotherapy (FOLFOX4) in patients with stage III colon cancer improved disease-free survival (DFS). METHODS: For this open-label, randomised phase 3 study done in nine European countries, we enrolled patients through an interactive voice response system to the central randomisation centre, with a central stratified permuted block randomisation procedure. We randomly assigned patients with resected (R0) stage III disease (1:1) to receive 12 cycles of FOLFOX4 twice a week with or without cetuximab. Patients were stratified by N-status (N1 vs. N2), T-status (T1-3 vs. T4), and obstruction or perforation status (no obstruction and no perforation vs. obstruction or per-foration or both). A protocol amendment (applied in June, 2008, after 2096 patients had been randomly assigned to treatment-restricted enrolment to patients with tumours wild-type at codons 12 and 13 in exon 2 of the KRAS gene (KRAS exon 2 wild-type). The primary endpoint was DFS. Analysis was intention to treat in all patients with KRAS exon 2 wild-type tumours. The study is registered at EudraCT, number 2005-003463-23. FINDINGS: Between Dec 22, 2005, and Nov 5, 2009, 2559 patients from 340 sites in Europe were randomly assigned. Of these patients, 1602 had KRAS exon 2 wild-type tumours (intention-to-treat population), 791 in the FOLFOX4 plus cetuximab group and 811 in the FOLFOX4 group. Median follow-up was 3·3 years (IQR 3·2–3·4). In the experimental and

ASCO-CRC-2015-V2-INT.indb 50 4/8/2015 4:41:42 PM

Copyright by MT

Page 55: › medical_mx › images › ... · 5 Samantha Hendren; Ellen McKeown; Arden M . Morris; Sandra L . Wong; Mary Oerline; Lyndia Poe; Darrell A . Campbell Jr, and Nancy J . Birkmeyer

51

Influential Papers

control groups, DFS was similar in the intention-to-treat population (hazard ratio [HR] 1·05; 95% CI 0·85–1·29; p = 0·66), and in patients with KRAS exon 2/BRAF wild-type (n = 984, HR 0·99; 95% CI 0·76–1·28) or KRAS exon 2-mutated tumours (n = 742, HR 1·06; 95% CI 0·82–1·37). We noted heterogeneous responses to the addition of cetuximab in preplanned subgroup analyses. Grade 3 or 4 acne-like rash (in 209 of 785 patients [27%] vs. four of 805 [< 1%]), diarrhoea (113 [14%] vs. 70 [9%]), mucositis (63 [8%] vs. 10 [1%]), and infusion-related reactions (55 [7%] vs. 30 [4%]) were more frequent in patients treated with FOLFOX4 plus cetuximab than in those patients who received FOLFOX4 alone. INTERPRETATION: The addition of cetuximab to FOLFOX4 did not improve DFS com-pared with FOLFOX4 alone in patients with KRAS exon 2 wild-type resected stage III colon cancer. This trial cannot conclude on the benefit of cetuximab in the studied popu-lation, but the heterogeneity of response suggests that further investigation of the role of FOLFOX4 plus cetuxi-mab in specific patient subgroups is warranted.

5. Daniel J. Sargent, Qian Shi, Greg Yothers, Sabine Tejpar, Monica M. Bertagnolli, Stephen N. Thibodeau, Thierry Andre, Roberto Labianca, Steven Gallinger, Stanley R. Hamilton, Genevieve Monges, Katherine L. Pogue-Geile, Soonmyung Paik, Dirk Klingbiel, Arnaud Roth, Emily S Pavey, George P. Kim, Frank A. Sinicrope. Prognostic impact of deficient mismatch repair (dMMR) in 7,803 stage II/III colon cancer (CC) patients (pts): A pooled individual pt data analysis of 17 adjuvant trials in the ACCENT database. J Clin Oncol 2014;32:5s, (suppl; abstr 3507).

■■ BACKGROUND: MMR status has been established as an important prognostic factor in CC pts. We determined the association of dMMR status with clinical/pathologi-cal features and prognosis using the ACCENT database. METHODS: Microsatellite instability (MSI, 14 studies) or immunohistochemical analysis (1 study) for MLH1/MSH2/MLH6 proteins was performed on 7,803 stage II/III pts enrolled in 17 trials; 2 studies tested both. 571 pts received surgery alone; 3,878 5FU monotherapy (mrx); 2,299 5FU+oxaliplatin (oxal); and 1,055 5-FU+irinotecan (iri). Tumors with MSI-high or an absent protein were classi-fied as dMMR; remainder were MMR-proficient (pMMR). Median follow-up was 7 years. Outcomes included overall survival (OS) and time to recurrence (TTR). Correlation analyses included all pts; prognostic analyses were limited to pts receiving surgery alone or 5FU mrx excluding pts receiving oxal/iri. RESULTS: 524 (23.1%) of 2,270 stage II and 823 (14.9%) of 5,533 stage III pts exhibited dMMR. MMR status was associated with female sex (dMMR, 19% F vs. 16% M, p = 0.004), higher T stage (dMMR, 11% T1/2 vs. 18% T3 vs. 21% T4, p < 0.001), and right-sided tumor location (dMMR, left 9% vs. right 27%, p < 0.001). Compared to pMMR, dMMR was strongly associated with

improved OS (HR = 0.27, p = 0.01) and TTR (HR = 0.27, p = 0.01) in stage II pts treated with surgery alone (Table). Association of MMR was prognostic although of attenu-ated impact in 5FU treated stage II and in stage III pts , with significance confined to 5FU-mrx treated stage III pts (HR = 0.80, p = 0.02 for TTR; HR = 0.79, p = 0.02 for OS). CONCLUSIONS: Our study confirms prognostic utility of MMR status in stage II CCs. MMR also impacts out-come in stage III patients, but does not currently alter pt management.

6. Mayer RJ, Venook AP, Schilsky RL. Progress against GI cancer during the American Society of Clinical Oncol-ogy’s first 50 years. J Clin Oncol 2014;32(15):1521–30.

■■ Tumors of the GI tract were frequent causes of cancer-related mortality in 1964 when the American Society of Clinical Oncology (ASCO) was founded and remain so in 2014, 50 years later. GI cancers accounted for 31% of the reported 305,000 cancer-related deaths in the United States in 1967 and 28% of the anticipated 585,720 such deaths in 2014. Worldwide, GI cancers led to more than 2.3 million deaths in 2008, far more than deaths resulting from any other group of malignant conditions.

7. Burki TK. Flexible sigmoidoscopy screening for colo-rectal cancer. Lancet Oncol 2014;15(10):e425.

■■ A new study has reaffirmed the value of flexible sigmoi-doscopy screening for colorectal cancer. The trial randomly assigned roughly 100 000 individuals aged 50–64 years taken directly from the Norwegian Population Register. 78 220 individuals were included in the control group, and the remaining 20 572 were assigned to screening; either to flexible sigmoidoscopy or, in equal numbers, to flexible sigmoidoscopy in combination with once-only immuno-logical faecal occult blood test (FIT).

8. Primrose JN, Perera R, Gray A, Rose P, Fuller A, Corkhill A, George S, Mant D; FACS Trial Investigators. Effect of 3 to 5 years of scheduled CEA and CT follow-up to detect recurrence of colorectal cancer: the FACS randomized clinical trial. JAMA 2014;311(3):263–70.

■■ IMPORTANCE: Intensive follow-up after surgery for colorectal cancer is common practice but is based on limited evidence. OBJECTIVE: To assess the effect of scheduled blood measurement of carcinoembryonic antigen (CEA) and computed tomography (CT) as follow-up to detect recurrent colorectal cancer treat-able with curative intent. DESIGN, SETTING, AND PARTICIPANTS: Randomized clinical trial in 39 National Health Service hospitals in the United Kingdom; 1202

ASCO-CRC-2015-V2-INT.indb 51 4/8/2015 4:41:42 PM

Copyright by MT

Page 56: › medical_mx › images › ... · 5 Samantha Hendren; Ellen McKeown; Arden M . Morris; Sandra L . Wong; Mary Oerline; Lyndia Poe; Darrell A . Campbell Jr, and Nancy J . Birkmeyer

52

Influential Papers

eligible participants were recruited between January 2003 and August 2009 who had undergone curative surgery for primary colorectal cancer, including adjuvant treatment if indicated, with no evidence of residual disease on investigation. INTERVENTIONS: Participants were randomly assigned to 1 of 4 groups: CEA only (n = 300), CT only (n = 299), CEA + CT (n = 302), or minimum follow-up (n = 301). Blood CEA was measured every 3 months for 2 years, then every 6 months for 3 years; CT scans of the chest, abdomen, and pelvis were performed every 6 months for 2 years, then annually for 3 years; and the minimum follow-up group received follow-up if symptoms occurred. MAIN OUTCOMES AND MEASURES: The primary outcome was surgical treatment of recurrence with curative intent; secondary outcomes were mortality (total and colorectal cancer), time to detection of recurrence, and survival after treat-ment of recurrence with curative intent. RESULTS: After a mean 4.4 (SD, 0.8) years of observation, cancer recur-rence was detected in 199 participants (16.6%; 95% CI, 14.5%–18.7%) overall; 71 of 1202 participants (5.9%; 95% CI, 4.6%–7.2%) were treated for recurrence with curative intent, with little difference according to Dukes staging (stage A, 5.1% [13/254]; stage B, 6.1% [34/553]; stage C, 6.2% [22/354]). Surgical treatment of recurrence with curative intent was 2.3% (7/301) in the minimum follow-up group, 6.7% (20/300) in the CEA group, 8% (24/299) in the CT group, and 6.6% (20/302) in the CEA + CT group. Compared with minimum follow-up, the absolute difference in the percentage of patients treated with curative intent in the CEA group was 4.4% (95% CI, 1.0%-7.9%; adjusted odds ratio [OR], 3.00; 95% CI, 1.23–7.33), in the CT group was 5.7% (95% CI, 2.2%–9.5%; adjusted OR, 3.63; 95% CI, 1.51–8.69), and in the CEA + CT group was 4.3% (95% CI, 1.0%–7.9%; adjusted OR, 3.10; 95% CI, 1.10–8.71). The number of deaths was not significantly different in the combined intensive monitoring groups (CEA, CT, and CEA + CT; 18.2% [164/901]) vs. the minimum follow-up group (15.9% [48/301]; difference, 2.3%; 95% CI, -2.6% to 7.1%). CONCLUSIONS AND RELEVANCE: Among patients who had undergone curative surgery for primary colorectal cancer, intensive imaging or CEA screening each provided an increased rate of surgical treatment of recurrence with curative intent compared with minimal follow-up; there was no advantage in com-bining CEA and CT. If there is a survival advantage to any strategy, it is likely to be small.

9. García-Albéniz X, Gallego R, Hofheinz RD, Fernández- Esparrach G, Ayuso-Colella JR, Bombí JA, Conill C, Cuatrecasas M, Delgado S, Ginés A, Miquel R, Pagés M, Pineda E, Pereira V, Sosa A, Reig O, Victoria I, Feliz L, María de Lacy A, Castells A, Burkholder I, Hochhaus A, Maurel J. Adjuvant therapy sparing in rectal cancer achieving complete response after chemoradiation. World J Gastroenterol 2014;20(42):15820–9.

■■ AIM: To evaluate the long-term results of conventional chemoradiotherapy and laparoscopic mesorectal excision in rectal adenocarcinoma patients without adjuvant therapy. METHODS: Patients with biopsy-proven adenocarcinoma of the rectum staged cT3-T4 by endoscopic ultrasound or magnetic resonance imaging received neoadjuvant con-tinuous infusion of 5-fluorouracil for five weeks and con-comitant radiotherapy. Laparoscopic surgery was planned after 5–8 wk. Patients diagnosed with ypT0N0 stage cancer were not treated with adjuvant therapy according to the protocol. Patients with ypT1–2N0 or ypT3–4 or N+ were offered 5-fluorouracil-based adjuvant treatment on an individual basis. An external cohort was used as a refer-ence for the findings. RESULTS: One hundred and seventy six patients were treated with induction chemoradiother-apy and 170 underwent total mesorectal excision. Can-cer staging of ypT0N0 was achieved in 26/170 (15.3%) patients. After a median follow-up of 58.3 mo, patients with ypT0N0 had five-year disease-free and overall survival rates of 96% (95%CI: 77–99) and 100%, respectively. We provide evidence about the natural history of patients with localized rectal cancer achieving a complete response after preoperative chemoradiation. The inherent good prognosis of these patients will have implications for clini-cal trial design and care of patients. CONCLUSION: With holding adjuvant chemotherapy after complete response following standard neoadjuvant chemoradiotherapy and laparoscopic mesorectal excision might be safe within an experienced multidisciplinary team.

10. Roxburgh CS, McMillan DC, Richards CH, Atwan M, Anderson JH, Harvey T, Horgan PG, Foulis AK. The clini-cal utility of the combination of T stage and venous invasion to predict survival in patients undergoing sur-gery for colorectal cancer. Ann Surg 2014;259(6): 1156–65.

■■ OBJECTIVE: To examine the clinical utility of improved detection of venous invasion (VI) in patients undergo-ing potentially curative resection of colorectal cancer. BACKGROUND: VI is a feature of colorectal cancer (CRC) progression. Elastica staining can be used to improve detection of VI and correspondingly its prediction of patient survival. METHODS: A single-center, observational study of pathology variables, including detection of VI by staining for elastica, using 631 stage I to III CRC specimens, collected from 1997 to 2009 (176 analyzed

ASCO-CRC-2015-V2-INT.indb 52 4/8/2015 4:41:42 PM

Copyright by MT

Page 57: › medical_mx › images › ... · 5 Samantha Hendren; Ellen McKeown; Arden M . Morris; Sandra L . Wong; Mary Oerline; Lyndia Poe; Darrell A . Campbell Jr, and Nancy J . Birkmeyer

53

Influential Papers

retrospectively and 455 analyzed prospectively), was per-formed. RESULTS: VI was detected in 56% of patients with CRC. Over a median follow-up period of 73 months, 238 patients died (134 from cancer). On multivariate analysis, VI by elastica staining was associated with a shorter sur-vival duration, independent of other pathology features, in all cases [hazard ratio (HR) = 3.94, 95% confidence interval (CI): 2.33–6.65, P < 0.001] and in node-negative cases (HR = 3.55, 95% CI: 1.81–6.97; P < 0.001). In the absence of elastica-detected VI, with the exception of T stage, no other pathology features were associated with survival time. Therefore, the combination of T stage and VI (TVI) on survival was examined. Five-year cancer mortality could be stratified between 100% and 54% for patients with node-negative tumors and between 100% and 33% for patients with node-positive tumors. In all cases, the TVI had similar predictive value as that of T stage and node status (TNM). In node-negative disease, TVI had superior predictive value.CONCLUSIONS: The results of the present study have prompted the development of a novel tumor staging system based on TVI. The TVI has clinical utility, especially in node-negative disease, in predicting outcome following curative resection for CRC.

11. Hoff PM, Saragiotto DF, Barrios CH, del Giglio A, Coutinho AK, Andrade AC, Dutra C, Forones NM, Correa M, Portella Mdo S, Passos VQ, Chinen RN, van Eyll B. Randomized phase III trial exploring the use of long-acting release octreotide in the preven-tion of chemotherapy-induced diarrhea in patients with colorectal cancer: the LARCID trial. J Clin Oncol 2014;32(10):1006–11.

■■ PURPOSE: Chemotherapy-induced diarrhea (CID) is a rela-tively common adverse event in the treatment of patients with colorectal cancer. The LAR for Chemotherapy-Induced Diarrhea (LARCID) trial evaluated the efficacy and safety of long-acting release octreotide (octreotide LAR) for the pre-vention of CID in this population. PATIENTS AND METHODS: Patients with colorectal cancer starting adjuvant or first-line treatment with a chemotherapy combination containing fluorouracil, capecitabine, and/or irinotecan were randomly assigned to receive octreotide LAR 30 mg intramuscularly every 4 weeks (experimental arm) or the physician’s treat-ment of choice in case of diarrhea (control arm). RESULTS: A total of 139 patients were randomly assigned, most of whom received fluorouracil- and oxaliplatin-containing chemotherapy regimens. The rate of diarrhea was 76.1% in the experimental group (n = 68) and 78.9% in the con-trol group (n = 71). Treatment with octreotide LAR did not prevent or reduce the severity of CID. Treatment choices for diarrhea management included loperamide in the majority of patients. No benefit from octreotide LAR was identified in terms of need for diarrhea treatment, opioids, or intrave-nous hydration or in the rate of hospitalization or quality of life. CONCLUSION: This study could not prove the efficacy of octreotide LAR in the prevention of CID.

12. Reames BN, Sheetz KH, Waits SA, Dimick JB, Regenbogen SE. Geographic variation in use of laparoscopic colectomy for colon cancer. J Clin Oncol 2014;32(32):3667–72.

■■ PURPOSE: Emerging evidence supporting the use of laparoscopic colectomy in patients with cancer has led to dramatic increases in utilization. Though certain patient and hospital characteristics may be associated with the use of laparoscopy, the influence of geography is poorly understood. METHODS: We used national Medicare claims data from 2009 and 2010 to examine geographic variation in utilization of laparoscopic colectomy for patients with colon cancer. Patients were assigned to hospital referral regions (HRRs) where they were treated. Multivariable logistic regression was used to generate age, sex, and race-adjusted rates of laparoscopic colec-tomy for each HRR. Patient quintiles of adjusted HRR utilization were used to evaluate differences in patient and hospital characteristics across low and high-utilizing HRRs. RESULTS: A total of 93,786 patients underwent colon resections at 3,476 hospitals during the study period, of which 30,502 (32.5%) were performed laparo-scopically. Differences in patient characteristics between the lowest and highest quintiles of HRR utilization were negligible, and there was no difference in the availability of laparoscopic technology. Yet adjusted rates of lapa-roscopic colectomy utilization varied from 0% to 66.8% across 306 HRRs in the United States. CONCLUSION: There is wide geographic variation in the utilization of laparoscopic colectomy for Medicare patients with colon cancer, suggesting treatment location may substantially influence a patient’s options for surgical approach. Future efforts to reduce variation will require increased dis-semination of training techniques, novel opportunities for learning among surgeons, and enhanced educational resources for patients.

13. Shin JS, Tut TG, Ho V, Lee CS. Predictive markers of radiotherapy-induced rectal cancer regression. J Clin Pathol 2014. pii: jclinpath-2014-202494.

■■ Patients with locally advanced rectal cancer receive preoperative radiotherapy to reduce the probability of recurrence and to possibly improve overall survival. However, this appears dependent on the extent of histological tumour regression seen in the resected bowel, which can be highly variable between individuals. No predictive marker that can stratify patient manage-ment in this regard is currently available. Experimental data implicates a variety of factors that are involved in the DNA damage response following radiation injury, tumour tissue oxygenation, autoimmune antitumour response triggered by radiotherapy and in the pathogen-esis of colorectal cancer, as potential indicators of radia-tion sensitivity. These details are presented in this review,

ASCO-CRC-2015-V2-INT.indb 53 4/8/2015 4:41:42 PM

Copyright by MT

Page 58: › medical_mx › images › ... · 5 Samantha Hendren; Ellen McKeown; Arden M . Morris; Sandra L . Wong; Mary Oerline; Lyndia Poe; Darrell A . Campbell Jr, and Nancy J . Birkmeyer

54

Influential Papers

which may serve as targets for clinical validation studies aiming to find predictors of radiotherapy response in rectal cancer.

14. Brouquet A, Nordlinger B. Minimally-invasive approach for rectal cancer surgery. Lancet Oncol 2014;15(7):680–1.

■■ Rectal surgery is the reference treatment for patients with non-metastatic rectal cancer. High-quality surgery, including total mesorectal excision, is particularly needed for patients with mid-rectal and low-rectal cancer to have potentially good oncological results and a low risk of complications.

15. Horne J, Bateman AC, Carr NJ, Ryder I. Lymph node revealing solutions in colorectal cancer: should they be used routinely? J Clin Pathol 2014;67(5):383–8.

■■ The Royal College of Pathologists (RCPath) and College of American Pathologists recommend that at least 12 lymph nodes should be harvested for adequate staging of colorectal carcinoma. Just one nodal tumour deposit upstages the malignancy from pN0 to pN1. This is criti-cally important as node-positive patients (pN1) are consid-ered for adjuvant chemotherapy whereas node-negative patients (pN0) may not be. It is not always easy to harvest the required number, especially in patients with rectal car-cinoma who may have received neoadjuvant therapy-an increasingly common treatment. The use of neoadjuvant therapy is known to further decrease the number and size of identifiable lymph nodes within specimens, meaning that the lymph node harvest often fails to reach RCPath guidelines. Lymph node revealing solutions consisting of either single chemicals such as alcohol or acetone or com-pounds have been investigated to help improve the lymph node harvest in difficult specimens, for example, those received following neoadjuvant therapy. Published research evidence reviewed here suggests that lymph node reveal-ing solutions significantly improve lymph node harvesting, and that glacial acetic acid, ethanol, water and formalin is advantageous in comparison with other revealing solutions in that it is safe, cheap, easy to use and relatively quick. However, the quantity of good evidence is limited and the clinical implications of improving lymph node harvesting require further research.

16. Kennedy RH, Francis EA, Wharton R, Blazeby JM, Quirke P, West NP, Dutton SJ. Multicenter rand-omized controlled trial of conventional versus laparoscopic surgery for colorectal cancer within an enhanced recovery programme: EnROL. J Clin Oncol 2014;32(17):1804–11.

■■ PURPOSE: Laparoscopic resection and a multimodal approach known as an enhanced recovery program (ERP) have been major changes in colorectal perioperative care that have improved clinical outcomes for colorectal can-cer resection. EnROL (Enhanced Recovery Open Versus Laparoscopic) is a multicenter randomized controlled trial examining whether the benefits of laparoscopy still exist when open surgery is optimized within an ERP. PATIENTS AND METHODS: Adults with colorectal cancer suitable for elective resection were randomly assigned at a ratio of 1:1 to laparoscopic or open surgery within an ERP, stratified by center, cancer site (colon vs. rectum), and age group (< 66 vs. 66–75 vs. >75 years) using minimization. The primary outcome was physical fatigue at 1 month postsurgery. Secondary outcomes included hospital stay, complications, other patient-reported outcomes (PROs), and physical function. Patients and outcome assessors were blinded until 7 days postsurgery or discharge if earlier. Central independent and blinded pathologic assessment of surgical quality was under-taken. RESULTS: A total of 204 patients (laparoscopy, n = 103; open surgery, n = 101) were recruited from 12 UK centers from July 2008 to April 2012. One-month physical fatigue scores were similar in both groups (mean: laparoscopy, 12.28; 95% CI, 11.37 to 13.19 vs. open surgery, 12.05; 95% CI, 11.14 to 12.96; adjusted mean difference, -0.23; 95% CI, -1.52 to 1.07). Median total hospital stay was significantly shorter after lapa-roscopic surgery (median: laparoscopy, 5; interquartile range [IQR], 4 to 9 vs. open surgery, 7; IQR, 5 to 11 days; P = .033). There were no differences in other secondary outcomes or in specimen quality after central pathologic review. CONCLUSION: In patients treated by experienced surgeons within an ERP, physical fatigue and other PROs were similar in both groups, but laparoscopic surgery significantly reduced length of hospital stay.

17. McCleary NJ, Dotan E, Browner I. Refining the chemotherapy approach for older patients with colon cancer. J Clin Oncol 2014;32(24):2570-80.

■■ Population studies support an increased incidence of most cancers among older adults. Colorectal cancer has

ASCO-CRC-2015-V2-INT.indb 54 4/8/2015 4:41:42 PM

Copyright by MT

Page 59: › medical_mx › images › ... · 5 Samantha Hendren; Ellen McKeown; Arden M . Morris; Sandra L . Wong; Mary Oerline; Lyndia Poe; Darrell A . Campbell Jr, and Nancy J . Birkmeyer

55

Influential Papers

high prevalence in the aging population, with a median age of 69 years at diagnosis and 74 years at death. The vast majority of patients with colon cancer (CC) will require chemotherapy treatments during their disease course, challenging oncologists with the task of tailoring therapy for older patients with CC in the face of limited evidence-based data to guide them. Factors such as comorbidity, performance status, cognitive function, and social support may affect decision making and complicate tolerance of

any recommended therapy. In recent years, attention to the specific needs of the aging population with cancer has given rise to the field of geriatric oncology in general, and has generated an increasing fund of knowledge on which to base chemotherapy delivery for this specific population of patients with CC. This article will review the available data specifically for chemotherapy management of older patients with CC in the postoperative and metastatic settings.

ASCO-CRC-2015-V2-INT.indb 55 4/8/2015 4:41:42 PM

Copyright by MT

Page 60: › medical_mx › images › ... · 5 Samantha Hendren; Ellen McKeown; Arden M . Morris; Sandra L . Wong; Mary Oerline; Lyndia Poe; Darrell A . Campbell Jr, and Nancy J . Birkmeyer

ASCO-CRC-2015-V2-INT.indb 56 4/8/2015 4:41:42 PM

Copyright by MT

Page 61: › medical_mx › images › ... · 5 Samantha Hendren; Ellen McKeown; Arden M . Morris; Sandra L . Wong; Mary Oerline; Lyndia Poe; Darrell A . Campbell Jr, and Nancy J . Birkmeyer

■■ Hot Topics

ASCO-CRC-2015-V2-INT.indb 57 4/8/2015 4:41:42 PM

Copyright by MT

Page 62: › medical_mx › images › ... · 5 Samantha Hendren; Ellen McKeown; Arden M . Morris; Sandra L . Wong; Mary Oerline; Lyndia Poe; Darrell A . Campbell Jr, and Nancy J . Birkmeyer

ASCO-CRC-2015-V2-INT.indb 58 4/8/2015 4:41:42 PM

Copyright by MT

Page 63: › medical_mx › images › ... · 5 Samantha Hendren; Ellen McKeown; Arden M . Morris; Sandra L . Wong; Mary Oerline; Lyndia Poe; Darrell A . Campbell Jr, and Nancy J . Birkmeyer

59

■■ Introduction

Several studies have identified an associa-tion between more-extensive lymph node evaluation and improved survival in surgi-cally treated patients with colon cancer.1,2 As a result, several practice organizations and consensus panels now recommend the multidisciplinary evaluation of ≥ 12 lymph nodes as a quality indicator for appropriate staging of patients with colon cancer.3–8 The first recommendation was issued by the Working Party Report to the World Congress of Gastroenterology in 1990.

■■ Helen M . Parsons; James W . Begun; Karen M . Kuntz; Todd M . Tuttle; Patricia M . McGovern, and Beth A . Virnig

Lymph Node Evaluation for Colon Cancer in an Era of Quality Guidelines: Who Improves?(J Clin Pract 2014;9(4):e164–e168.)

Introduction: In the 1990s, several organizations began recommending evaluation of > 12 lymph nodes during colon resection because of its association with improved survival . We examined practice implications of multispe-cialty quality guidelines over the past 20 years recommending evaluation of ≥ 12 lymph nodes during colon resec-tion for adequate staging .Materials and methods: We used the 1988 to 2009 Surveillance, Epidemiology, and End Results program to con-duct a retrospective observational cohort study of 90,203 surgically treated patients with colon cancer . We used Cochran-Armitage tests to examine trends in lymph node examination over time and multivariate logistic regres-sion to identify patient characteristics associated with guideline-recommended lymph node evaluation .Results: The introduction of practice guidelines was associated with gradual increases in guideline-recommended lymph node evaluation . From 1988 to 1990, 34% of patients had > 12 lymph nodes evaluated, increasing to 38% in 1994 to 1996 and to > 75% from 2006 to 2009 . Younger, white patients and those with more-extensive bowel pen-etration (T3/4 nonmetastatic) and high tumor grade saw more-rapid increases in lymph node evaluation (P <  .001) . Multivariate analyses demonstrated a significant interaction between year of diagnosis and both T stage and grade, indicating that those with higher T stage and higher grade were more likely to receive guideline-recommended care earlier .Conclusion: The implementation of lymph node evaluation guidelines was accepted gradually into practice but adopted more quickly among higher risk patients . By identifying patients who are least likely to receive guideline-recommended care, these findings present a starting point for promoting targeted improvements in cancer care and further understanding underlying contributors to these disparities .

Subsequently, several organizations, including the American Society for Clinical Oncology, the American College of Sur-geons, and a National Cancer Institute con-sensus panel, issued benchmarks, which call for no fewer than 12 nodes for qual-ity surveillance. However, although these guidelines published a consistent message over the past 20 years, their greatest varia-tion was the target audience.

Historically, innovation has spread through different groups at varying rates, reflected first in practitioners with access

Key points

•Several studies have identified an association between more-extensive lymph node evaluation and improved survival in surgically treated patients with colon cancer.

ASCO-CRC-2015-V2-INT.indb 59 4/8/2015 4:41:42 PM

Copyright by MT

Page 64: › medical_mx › images › ... · 5 Samantha Hendren; Ellen McKeown; Arden M . Morris; Sandra L . Wong; Mary Oerline; Lyndia Poe; Darrell A . Campbell Jr, and Nancy J . Birkmeyer

Hot Topics

60

to specialized knowledge and resources.9–11 When published, guidelines may initially alert clinicians about practice innovations, but questions may often arise about the necessity or appropriateness of applying these guidelines. This is particularly true when the published lymph node guidelines apply equally to all patients, regardless of their risk for lymph node positivity.3,4,7,12 In this respect, studies suggest that clini-cians often deal with uncertainty by seek-ing information and opinions from peers in their networks.13 However, with the pub-lication of the guidelines, opinion leaders themselves began questioning the validity of lymph node evaluation target because of uncertainty surrounding the mecha-nism behind lymph node evaluation and improved survival.1,2,14–16 To date, no study has systematically evaluated at a population level whether, after guideline release, rates of lymph node evaluation increased.

The purpose of our study was to evaluate how lymph node evaluation changed in an era of quality guidelines and identify which patients experienced guideline-recommended care first. We examined trends over time in the number of lymph nodes examined across patient tumor and demographic characteristics.

■■ Materials and methods

■■ DataFor this study, we used the 1988 to 2009 Surveillance, Epidemiology, and End Results (SEER 9 registries) cancer registry data. Sponsored by the National Cancer Institute, SEER currently collects and publishes can-cer incidence, treatment, and survival data from population-based cancer registries. The SEER 9 registries were used because they consistently collected data on cancer incidence in representative geographic areas throughout the United States during the study period, covering approximately 14% of the US population. Variables include patient and tumor characteristics, staging information, and first course of

treatment (including surgery and radia-tion therapy). SEER collects information on patient age, race, sex, year of diagnosis, tumor registry, and number of lymph nodes evaluated. Tumor characteristics include size, T stage (ie, depth of bowel penetra-tion), histologic grade, histologic type, and presence of metastasis.

■■ PatientsIncluded in our study were patients age > 18 years who were diagnosed with their first invasive adenocarcinoma of the colon from January 1, 1988, through December 31, 2009. We included only patients who underwent radical resection of their colon cancer as the first course of treatment according to SEER and, therefore, were eligible for nodal evaluation. Excluded from our study were patients whose cancer was diagnosed by autopsy or first cited on the death certificate (n = 124), patients who underwent preoperative irradiation (n = 224),17 and patients with an unknown number of nodes evaluated (n = 3,531). Our final study population included 90,203 surgically treated patients with colon cancer.

■■ Assessment of lymph node evaluationSEER has recorded the number of nodes pathologically examined for each patient since 1988. Using this information, we cat-egorized patients according to their receipt (yes or no) of guideline-recommended lymph node evaluation (at least 12 nodes evaluated).

■■ Statistical analysesFor our analyses, we evaluated differences in nodal evaluation across patient charac-teristics based on their year of diagnosis, which we categorized into seven groups: 1988 to 1990, 1991 to 1993, 1994 to 1996, 1997 to 1999, 2000 to 2002, 2003 to 2005, and 2006 to 2009. First, we used the Cochran-Armitage test to evaluate unadjusted trends over time in the pro-portion of patients receiving guideline-recommended lymph node evaluation. Analyses were stratified by patient demo-graphic, tumor, and stage characteristics

Key points

• In this respect, studies suggest that clinicians often deal with uncertainty by seeking information and opinions from peers in their networks.

•To date, no study has systematically evaluated at a population level whether, after guideline release, rates of lymph node evaluation increased.

•The purpose of our study was to evaluate how lymph node evaluation changed in an era of quality guidelines and identify which patients experienced guideline-recommended care first.

•We examined trends over time in the number of lymph nodes examined across patient tumor and demographic characteristics.

•We used the 1988 to 2009 Surveillance, Epidemiology, and End Results (SEER 9 registries) cancer registry data.

•We included only patients who underwent radical resection of their colon cancer as the first course of treatment according to SEER and, therefore, were eligible for nodal evaluation.

•Excluded from our study were patients whose cancer was diagnosed by autopsy or first cited on the death certificate (n = 124), patients who underwent preoperative irradiation (n = 224), and patients with an unknown number of nodes evaluated (n = 3,531).

•Our final study population included 90,203 surgically treated patients with colon cancer.

ASCO-CRC-2015-V2-INT.indb 60 4/8/2015 4:41:43 PM

Copyright by MT

Page 65: › medical_mx › images › ... · 5 Samantha Hendren; Ellen McKeown; Arden M . Morris; Sandra L . Wong; Mary Oerline; Lyndia Poe; Darrell A . Campbell Jr, and Nancy J . Birkmeyer

Lymph Node Evaluation for Colon Cancer in an Era of Quality Guidelines

61

to identify whether there were differential trends in lymph node evaluation across patient subgroups. After assessing this unadjusted relationship, we used logis-tic regression to examine the association between patient characteristics and receipt of guideline-recommended lymph node evaluation over time (yes or no). Each model was adjusted for patient age at diagnosis, race, sex, T stage, tumor grade, years of diagnosis and SEER registry, metastatic dis-ease, and type of surgery. We then tested for interactions between year of diagno-sis and patient characteristics to identify whether the impact of demographic or tumor factors on guideline-recommended lymph node evaluation changed over time. Because American Joint Committee on Cancer staging classifications have changed over time,18 we focused on T stage as our staging indicator to provide information on how lymph node evaluation changed among patients with similar a extent of bowel wall penetration.

In all models, we performed several sen-sitivity analyses (eg, alternative category groupings, removal of nonsignificant fac-tors, interaction analyses) to ensure that the observed effects were not an artifact of our modeling decisions. We used SAS soft-ware (version 9.2; SAS Institute, Cary, NC)

Key points

•We used logistic regression to examine the association between patient characteristics and receipt of guideline-recommended lymph node evaluation over time (yes or no).

• In all models, we performed several sensitivity analyses (eg, alternative category groupings, removal of nonsignificant factors, interaction analyses) to ensure that the observed effects were not an artifact of our modeling decisions.

•We identified 90,203 patients diagnosed with invasive colon cancer between 1988 and 2009 in the SEER 9 program. The majority of patients were women (53.0%), white (81.3%), and diagnosed after age 70 years (53.8%).

•Guideline-recommended lymph node evaluation for colon cancer increased significantly from 1990 to 2009, but it increased more rapidly among patients at higher risk for node positivity.

for all analyses. P values ≤ .05 were consid-ered statistically significant. This study was approved by the University of Minnesota Institutional Review Board.

■■ Results

We identified 90,203 patients diagnosed with invasive colon cancer between 1988 and 2009 in the SEER 9 program. The majority of patients were women (53.0%), white (81.3%), and diagnosed after age 70 years (53.8%). These patients were most predominately diagnosed with low-grade (grade 1 or 2, 75.6%), T-stage 3 (52.1%), and proximal (59.3%) tumors. Over the 20-year study period, the number of surgically treated patient cases of colon cancer included in our study remained rela-tively stable over time, ranging between approximately 3,000 and 4,000 per year.

■■ Changes in lymph node evaluation across patient and tumor characteristics

Guideline-recommended lymph node evalu-ation for colon cancer increased significantly from 1990 to 2009, but it increased more rapidly among patients at higher risk for node positivity (Figure 1). Directly after the first guideline publication in 1990,

FIGURE 1 ■■■Percentage of patients receiving guideline-recommended lymph node evaluation by year and T stage.

AJCC, American Joint Committee on Cancer; ASCO, American Society of Clinical Oncology; ASCRS, American Society of Colon and Rectal Surgeons; CAP, College of American Pathologists; COC, Commission on Cancer; NCI, National Cancer Institute; NCCN, National Comprehensive Cancer Network; NQF, National Quality Forum; UICC, Union for International Cancer Control.

ASCO-CRC-2015-V2-INT.indb 61 4/8/2015 4:41:43 PM

Copyright by MT

Page 66: › medical_mx › images › ... · 5 Samantha Hendren; Ellen McKeown; Arden M . Morris; Sandra L . Wong; Mary Oerline; Lyndia Poe; Darrell A . Campbell Jr, and Nancy J . Birkmeyer

Hot Topics

62

recommended evaluation remained relatively stable, with only a 5% increase in guideline-recommended care compared with the period before the first guideline release (36% vs. 34%, respectively). By 1994 to 1996, 38% of patients had > 12 lymph nodes eval-uated. Although the percentage of patient cases with guideline-recommended lymph node evaluation exceeded 75% in 2006 to 2009, it is important to note that the major-ity (> 50%) of surgically treated patients with colon cancer did not receive guideline-recommended evaluation until > 10 years after the first published recommendation. Furthermore, from 2006 to 2009, approxi-mately 25% of surgically treated patients were still not receiving guideline-recom-mended lymph node evaluation.

Overall, the increase in guideline-recommended lymph node evaluation was implemented differentially across patient groups. Younger patients, those with high-grade (3 or 4) disease, and those with T stage of 2 to 3 saw more rapid increases in guideline-recommended lymph node evaluation after the guidelines were published. Notably, a majority of patients with T-stage 3 disease and T4/nonmetastatic tumors were receiving guideline-recommended lymph node evalu-ation within 10 years of the first published guideline, whereas > 40% of patients with T1 tumors still had < 12 nodes evalu-ated in 2006 to 2007—almost 20 years after the first guidelines were released (Figure 1). Although less pronounced, 27% of patients with T4/metastatic tumors were not receiving recommended evaluation in 2006 to 2009. Similarly, a majority of patients with high-grade (3 or 4) tumors were receiving guideline-recommended evaluation by 2000 to 2002, whereas patients with low-grade tumors were not evaluated at this level until 2003 to 2005.

Although lymph node evaluation was similar between sexes, patients experienced differ-ential evaluation by age at diagnosis as well as race. Whereas the majority of patients diagnosed at age < 50 years experienced

guideline-recommended evaluation within 1 year of the first published guideline, those age ≥ 80 years did not experience this level of guideline-recommended evalu-ation until 2003. However, even in 2006, older patients were experiencing lower levels of guideline-recommended lymph node evaluation. Interestingly, trends by race did not reveal the typical white advan-tage. At the time the first guidelines were published, white patients received lower levels of guideline-recommended evaluation compared with blacks. By 2006 to 2009, however, this trend had reversed. Finally, patients with more-extensive resection were more likely to receive recommended evalu-ation. Overall, these patterns across demo-graphic and clinical characteristics indicate that increases in lymph node evaluation were not linked to the release of a specific lymph node evaluation guideline published over the past 20 years.

■■ Association between patient characteristics and guideline-recommended lymph node evaluation

The association between patient demo-graphic and treatment characteristics and receipt of guideline-recommended evaluation (yes vs. no) in each time period. Examining each individual multivariate analysis, we demonstrate that after adjust-ing for patient factors, younger age, higher T stage and grade, nonmetastatic disease, and more-extensive resection were all associated with earlier receipt of guideline-recommended lymph node evaluation (ie, these patients were more likely to have ≥ 12 nodes evaluated in the earliest time periods). Our analyses identified a signifi-cant interaction between year of diagnosis and both T stage and grade, indicating that those with higher T stage and grade—both signs of tumor aggressiveness—were more likely to have guideline-recommended staging earlier, yet remained more likely to receive this care at all points in time. As a result, we present patient demographic and tumor factors associated with guideline-recommended lymph node evaluation stratified by year of diagnosis.

Key points

•Overall, the increase in guideline-recommended lymph node evaluation was implemented differentially across patient groups.

•Younger patients, those with high-grade (3 or 4) disease, and those with T stage of 2 to 3 saw more rapid increases in guideline-recommended lymph node evaluation after the guidelines were published.

•A majority of patients with high-grade (3 or 4) tumors were receiving guideline-recommended evaluation by 2000 to 2002, whereas patients with low-grade tumors were not evaluated at this level until 2003 to 2005.

•Older patients were experiencing lower levels of guideline-recommended lymph node evaluation.

•White patients received lower levels of guideline-recommended evaluation compared with blacks.

•Overall, patterns across demographic and clinical characteristics indicate that increases in lymph node evaluation were not linked to the release of a specific lymph node evaluation guideline published over the past 20 years.

•Association between patient demographic and treatment characteristics and receipt of guideline-recommended evaluation (yes vs. no) in each time period.

ASCO-CRC-2015-V2-INT.indb 62 4/8/2015 4:41:43 PM

Copyright by MT

Page 67: › medical_mx › images › ... · 5 Samantha Hendren; Ellen McKeown; Arden M . Morris; Sandra L . Wong; Mary Oerline; Lyndia Poe; Darrell A . Campbell Jr, and Nancy J . Birkmeyer

Lymph Node Evaluation for Colon Cancer in an Era of Quality Guidelines

63

Across all years, increasing age at diagnosis was inversely associated with guideline-recommended lymph node eval-uation. Additionally, patients diagnosed with distal tumors were significantly less likely to receive guideline-compliant care. This pattern remained consistent over time, suggesting no lessening of the rela-tive pattern (distal vs. proximal colon: odds ratio [OR], 0.61; 95% CI, 0.55 to 0.68 in 1991 to 1993 and OR, 0.64; 95% CI, 0.58 to 0.69 in 2006 to 2009). How-ever, the association between T stage and receipt of guideline-consistent lymph node evaluation increased in magnitude directly after the first published guideline. After this publication, patients with higher T-stage tumors, particularly T3 tumors, were more likely to receive guideline- recommended care. By 2006 to 2009, how-ever, patients with T2, T3, and T4 tumors received similar levels of guideline-recommended care relative to those with T1 tumors. Additionally, although patients diagnosed with higher-grade tumors (3 or 4) were significantly more likely to have guideline-recommended lymph node evaluation by the end of the study period, this effect did not appear immediately after the first guideline was published; rather, it was observed beginning in 1997 to 1999 (grade 3 or 4 vs. 1 or 2: OR, 1.15; 95% CI, 1.06 to 1.27).

■■ Discussion

Among surgically treated patients with colon cancer from 1988 to 2009 in the SEER program, we found significant increases in guideline-recommended lymph node evaluation during the past 20 years, particularly among patients with high risk (eg, high grade and T-stage 3 or 4 non-metastatic disease) of being node positive. However, this improvement in guideline-recommended staging over the past two decades seemed to increase gradually over time, with adherence increasing in the later years of our study (Figure 1), poten-tially as a result of more-intense media

and peer-reviewed coverage of this topic beginning in the late 1990s. In addition, despite the release of > nine different recommendations, the rate of guideline-compliant lymph node evaluation is still low, with 25% of patients not receiving guideline-compliant care. This is impor-tant when considered in light of our study inclusion criteria, requiring participants to undergo radical resection, necessarily fall-ing under guideline recommendations for evaluation. By identifying those patients who are least likely to receive guideline-recommended care, these findings present a starting point for promoting targeted improvements in cancer care and further understanding underlying contributors to these disparities.

Overall, our results are consistent with those of prior studies indicating that younger age,19,20 more-advanced tumor depth20–22 and high tumor grade,20 and surgical technique are important predic-tors of more-extensive lymph node evalu-ation. We additionally found that factors associated with guideline-recommended lymph node evaluation at each time point were also associated with earlier increases in lymph node evaluation over time. This points toward the possibility that providers may more carefully stage patients who they believe are at higher risk for node-positive disease than those believed to be at lower risk of having node-positive disease. Fur-thermore, our study identifies that surgical technique plays a role in the number of lymph nodes that are subsequently evalu-ated. Of note, however, whereas T3 and T4 nonmetastatic tumors were both more likely to undergo guideline-recommended evaluation over time, > 27% of patients with T4 metastatic tumors were still not receiving recommended evaluation in 2006 to 2009, potentially because the presence of metastatic disease would not change treatment or staging criteria regardless of nodal status. However, because 12 lymph nodes is not an onerous threshold for evaluation, and relatively little would be lost in adequately staging all patients,

Key points

•Patients diagnosed with distal tumors were significantly less likely to receive guideline-compliant care.

•The association between T stage and receipt of guideline-consistent lymph node evaluation increased in magnitude directly after the first published guideline.

•Patients with T2, T3, and T4 tumors received similar levels of guideline-recommended care relative to those with T1 tumors.

•Among surgically treated patients with colon cancer from 1988 to 2009 in the SEER program, we found significant increases in guideline-recommended lymph node evaluation during the past 20 years.

•This is important when considered in light of our study inclusion criteria, requiring participants to undergo radical resection, necessarily falling under guideline recommendations for evaluation.

•Findings present a starting point for promoting targeted improvements in cancer care and further understanding underlying contributors to these disparities.

•We found that factors associated with guideline-recommended lymph node evaluation at each time point were also associated with earlier increases in lymph node evaluation over time.

•Our study identifies that surgical technique plays a role in the number of lymph nodes that are subsequently evaluated.

ASCO-CRC-2015-V2-INT.indb 63 4/8/2015 4:41:43 PM

Copyright by MT

Page 68: › medical_mx › images › ... · 5 Samantha Hendren; Ellen McKeown; Arden M . Morris; Sandra L . Wong; Mary Oerline; Lyndia Poe; Darrell A . Campbell Jr, and Nancy J . Birkmeyer

Hot Topics

64

it is surprising that more patients did not receive guideline-compliant staging. Because the guideline applies equally to all patients with surgically resected colon can-cer, this finding may provide better under-standing of how guidelines are used and point toward opportunities to improve the quality of staging.

Clinical practice guidelines for lymph node evaluation were developed based on the findings of several single- and multiinsti-tution studies identifying an association between increased lymph node evaluation and improved survival after surgical resec-tion for colon cancer.1,23–25 However, these studies varied widely in the minimum number of nodes they concluded would accurately determine nodal status, rang-ing from ≥ six to > 20 nodes. As these studies were published, several multidis-ciplinary practice groups and consensus panels convened to publish clinical practice guidelines. The resulting recommenda-tions varied in timing of publication and intended audience but were generally consistent in their threshold for adequate nodal evaluation. Because surgeons and pathologists vary in their roles for evaluat-ing nodal status, the publication of these guidelines in different sets of literature presents important implications for how the information was disseminated and incorporated into practice. Although some guidelines were published in the surgi-cal literature, the audience of which is responsible for removing tissue contain-ing lymph nodes, others were published in pathology journals, the membership of which focuses on identifying and deter-mining lymph node positivity. Holding the multidisciplinary team accountable for knowledge and use of these recommenda-tions, while highlighting the interweaving roles between the pathologists and sur-geons in achieving these guidelines, will be important as future updates are consid-ered. Specifically, future guidelines should capture the multidisciplinary approach to attaining this guideline, which can vary for a variety of factors ranging from surgical

technique to the rigor of gross dissection of the specimen.

Previous research has acknowledged that guidelines are released into complex social systems.10,26 Clinicians are busy profession-als who have social preferences and biases about what constitutes appropriate care. These individuals are often forced to bal-ance multiple and often conflicting pres-sures from administrators, colleagues, and patients to provide specific services and care. Social and financial pressures may be coupled with incentives, monetary or otherwise, that may not be aligned with the recommended practices. Specifically, Medicare and other payers have discussed or implemented lymph node evaluation standards as a quality measure for pay-for-performance initiatives.27,28 Additionally, these clinicians work within unique prac-tice settings, with varying levels of compe-tition for patients and access to resources. In that context, one might expect that clinicians would first apply new guide-lines to those patients perceived to be the highest-risk cases and, over time, apply them slowly and variably to others. For example, proximal tumors, advanced age, and more-extensive tumor penetration are all associated with worse prognosis, and such patients may be targeted by the multidisciplinary team for more-extensive evaluation. Additionally, the introduc-tion of these practice guidelines over the past 20 years has been coupled with vast changes in how individuals access information, including the growth of the Internet and online resources for clini-cians. In 1988, it would have been difficult for those not closely related to academic centers to have easy access to guideline information. However, online resources through the American Society for Clinical Oncology, the National Comprehensive Cancer Network, and other organizations make this information much more acces-sible. Additionally, today, many hospitals and cancer centers have colorectal cancer tumor boards, which serve as forums to implement and increase guideline

Key points

•Clinical practice guidelines for lymph node evaluation were developed based on the findings of several single- and multiinstitution studies identifying an association between increased lymph node evaluation and improved survival after surgical resection for colon cancer.

•Surgeons and pathologists vary in their roles for evaluating nodal status, the publication of these guidelines in different sets of literature presents important implications for how the information was disseminated and incorporated into practice.

•Holding the multidisciplinary team accountable for knowledge and use of these recommendations, while highlighting the interweaving roles between the pathologists and surgeons in achieving these guidelines, will be important as future updates are considered.

•Clinicians are busy professionals who have social preferences and biases about what constitutes appropriate care. These individuals are often forced to balance multiple and often conflicting pressures from administrators, colleagues, and patients to provide specific services and care.

•Social and financial pressures may be coupled with incentives, monetary or otherwise, that may not be aligned with the recommended practices.

ASCO-CRC-2015-V2-INT.indb 64 4/8/2015 4:41:43 PM

Copyright by MT

Page 69: › medical_mx › images › ... · 5 Samantha Hendren; Ellen McKeown; Arden M . Morris; Sandra L . Wong; Mary Oerline; Lyndia Poe; Darrell A . Campbell Jr, and Nancy J . Birkmeyer

Lymph Node Evaluation for Colon Cancer in an Era of Quality Guidelines

65

adherence for all involved in the treatment of these patients.

Although our study provides insight into the adoption of clinical practice guidelines, we acknowledge several data-related limi-tations. First, SEER does not collect infor-mation on comorbidities that may limit a surgeon’s ability to remove an adequate number of lymph nodes. However, these patients were seen as healthy enough to undergo surgical resection for treatment of their colon cancer. Additionally, we are unable to determine the reason behind the level of lymph node evaluation for an indi-vidual patient with cancer. Third, because our study population was predominately white, future studies should examine how the practice implications of guidelines vary in practice settings that include predomi-nately nonwhite populations. Finally, we were unable to identify the surgeon, pathol-ogist, or patient as the contributing factor associated with the level of lymph node evaluation. However, we do not believe that any of these limitations would challenge our population-based analysis, which under-scores the role of both demographic and clinical factors associated with the adoption of clinical guidelines in patients with colon cancer in the United States.

In conclusion, our population-based analy-sis of surgically treated patients with colon cancer identified differential adoption of clinical practice guidelines across patient tumor and demographic characteristics. Although we do not know the reasons for adherence, differences in physician beliefs about what constitutes good quality stag-ing may have influenced decisions about which patients would most benefit from comprehensive evaluation. Absent change in the guidelines, reminders that the guide-lines are intended to apply equally to all patients regardless of their risk for node positivity will be important as strategies for effective guideline development and dissemination are considered. In addition, future work should explore whether the experience with guideline adherence in colon cancer is replicated in other malig-nancies. Finally, accounting for the different roles surgeons, pathologists, and hospi-tals play in attaining this clinical practice guideline is crucial for setting priorities and aligning goals in future quality guide-lines. Overall, our findings identify those patients least likely to receive guideline-recommended care, promoting discussion about the further understanding of the underlying contributors to disparities in guideline-recommended care.

■■ References 1. Chang GJ, Bigas MA, Skibber JM, et al: Lymph node

evaluation and survival after curative resection of colon cancer: Systematic review. J Natl Cancer Inst 99:433–441, 2007.

2. Wong SL, Ji H, Hollenbeck BK, et al: Hospital and lymph node examination rates and survival after resection for colon cancer. JAMA 298:2149–2154, 2007.

3. Fielding LP, Arsenault PA, Chapuis PH, et al: Clinicopathological staging for colorectal cancer: An International Documentation System (IDS) and an International Comprehensive Anatomical Terminology (ICAT). J Gastroenterol Hepatol 6:325–344, 1991.

4. Fleming ID, Cooper JS, Henson DE, et al (eds): AJCC Cancer Staging Manual (ed 5). Philadelphia, PA, Lippincott, 1997.

5. Sobin LH, Wittekind C (eds): TNM Classification of Malignant Tumors (ed 5). New York, NY, Wiley, 1997.

6. Compton CC, Fielding LP, Burgart LJ, et al: Prognostic factors in colorectal cancer: College of American Pathologists consensus statement. Arch Pathol Lab Med 124:979–994, 2000.

7. Nelson H, Petrelli N, Carlin A, et al: Guidelines 2000 for colon and rectal cancer surgery. J Natl Cancer Inst 93:583–596, 2001.

8. Otchy D, Hyman NH, Simmang C, et al: Practice parameters for colon cancer. Dis Colon Rectum 47:1269–1284, 2004.

9. Rogers EM: Diffusion of Innovations. New York, NY, Free Press, 1983.

10. Rogers EM: Lessons for guidelines from the Diffusion of Innovations Joint Commission. J Qual Improvement 21: 324–328, 1995.

11. Jerome-D’Emilia B, Begun JW: Diffusion of breast conserving sur gery in medical communities. Soc Sci Med 60:143–151, 2005.

12. Desch CE, McNiff KK, Schneider EC, et al: American Society of Clinical Oncology/National Comprehensive Cancer Network quality measures. J Clin Oncol 26:3631–3637, 2008.

13. Mittman BS, Tonesk X, Jacobson PD: Implementing clinical practice guidelines: Social influence strategies and practitioner behavior change. Qual Rev Bull 18:413–422, 1992.

Key points

•Although our study provides insight into the adoption of clinical practice guidelines, we acknowledge several data-related limitations.

•SEER does not collect information on comorbidities that may limit a surgeon’s ability to remove an adequate number of lymph nodes.

•We are unable to determine the reason behind the level of lymph node evaluation for an individual patient with cancer.

•Our study population was predominately white, future studies should examine how the practice implications of guidelines vary in practice settings that include predominately nonwhite populations.

•We were unable to identify the surgeon, pathologist, or patient as the contributing factor associated with the level of lymph node evaluation.

ASCO-CRC-2015-V2-INT.indb 65 4/8/2015 4:41:43 PM

Copyright by MT

Page 70: › medical_mx › images › ... · 5 Samantha Hendren; Ellen McKeown; Arden M . Morris; Sandra L . Wong; Mary Oerline; Lyndia Poe; Darrell A . Campbell Jr, and Nancy J . Birkmeyer

Hot Topics

66

14. Baxter NA: Is lymph node count an ideal quality indicator for cancer care? J Surg Oncol 99:265–268, 2009.

15. Simunovic M, Baxter NB: Lymph node counts in colon cancer surgery: Lessons for users of quality indicators. JAMA 298:2194–2196, 2007.

16. Parsons HM, Tuttle TM, Kuntz KM, et al: Association between lymph node evaluation for colon cancer and node positivity over the past 20 years. JAMA 306:1089–1097, 2011.

17. Baxter NN, Morris AM, Rothenberger DA, et al: Impact of preoperative radiation for rectal cancer on subsequent lymph node evaluation: A population-based analysis. Int J Radiat Oncol Biol Phys 61:426–431, 2005.

18. Understanding the changes from the sixth to the seventh edition of the AJCC Cancer Staging Manual 2010. http://www.cancerstaging.org/staging/changes2010.pdf.

19. Ostadi MA, Harnish JL, Stegienko S, et al: Factors affecting the number of lymph nodes retrieved in colorectal cancer specimens. Surg Endosc 21:2142–2146, 2007.

20. Baxter NN, Virnig DJ, Rothenberger DA, et al: Lymph node evaluation in colorectal cancer patients: A population-based study. J Natl Cancer Inst 97:219–225, 2005.

21. Cserni G: Lymph node harvest reporting in patients with carcinoma of the large bowel: A French population-based study. Cancer 85:243–245, 1999.

22. Herrera-Ornelas L, Justiniano J, Castillo N, et al: Metastases in small lymph nodes from colon cancer. Arch Surg 122: 1253–1256, 1987.

23. Bui L, Rempel E, Reeson D,et al: Lymph node counts, rates of positive lymph nodes, and patient survival for colon cancer surgery in Ontario, Canada: A population-based study. J Surg Oncol 93:439–445, 2006.

24. Chen SL, Bilchik AJ: More extensive nodal dissection improves survival for stages I to III of colon cancer: A population-based study. Ann Surg 244:602–610, 2006.

25. Le Voyer TE, Sigurdson ER, Hanlon AL, et al: Colon cancer survival is associated with increasing number of lymph nodes analyzed:A secondary survey of intergroup trial INT-0089. J Clin Oncol 245:846–857, 2007.

26. Hay MC, Weisner TS, Subramanian S, et al: Harnessing expe-rience: Exploring the gap between evidence-based medicine and clinical practice. J Eval Clin Pract 14:707–713, 2008.

27. Health Care Financing Administration: Medicare Coverage of Colorectal Cancer Screening: Program Memoranda to Intermediaries/Carriers. Baltimore, MD, Health Care Financing Administration, 1997.

28. Bilimoria KY, Stewart AK, Edge SB: Lymph node evaluation as a colon cancer quality measure: A national hospital report card. J Natl Cancer Inst 100:1310–1317, 2008.

ASCO-CRC-2015-V2-INT.indb 66 4/8/2015 4:41:43 PM

Copyright by MT

Page 71: › medical_mx › images › ... · 5 Samantha Hendren; Ellen McKeown; Arden M . Morris; Sandra L . Wong; Mary Oerline; Lyndia Poe; Darrell A . Campbell Jr, and Nancy J . Birkmeyer

67

■■ Introduction

For the 1.2 million colorectal cancer (CRC) survivors in the United States, the chal-lenges of cancer extend beyond treatment completion.1 CRC survivors frequently do not undergo routine surveillance as recommended. 2–10 They also experience psychological distress (including depression and fear of recurrence)11–13 and issues with health-related quality of life.14–17 Further, CRC survivors are less likely to receive necessary general medical care, including management of chronic conditions and preventive care, than people who have never had CRC.18,19

■■ Talya Salz; Shrujal S . Baxi; Victoria S . Blinder; Elena B . Elkin; Margaret M . Kemeny; Mary S . McCabe; Chaya S . Moskowitz; Erin E . Onstad; Leonard B . Saltz; Larissa K .F . Temple, and Kevin C . Oeffinger

Colorectal Cancer Survivors’ Needs and Preferences for Survivorship Information(J Oncol Pract 2014;10(4):e277–e282.)

Purpose: Before developing a survivorship care plan (SCP) that colorectal cancer (CRC) survivors will value, under-standing the informational needs of CRC survivors is critical .Methods: We surveyed survivors treated for nonmetastatic CRC at two hospitals in New York about their needs and preferences for survivorship information . Participants completed treatment 6 to 24 months before the inter-view and had not received an SCP . We evaluated whether survivors knew their treatment history (10 topics), whether they understood ongoing risks (four topics), and their preferences for receiving 16 topics of survivorship information .Results: One hundred seventy-five survivors completed the survey . Most survivors remembered information about past treatment (98% to 99% for each treatment) . Fewer survivors knew their risks of local recurrence, distant recurrence, or developing a new CRC (69%, 77%, and 40%, respectively) . Most participants reported receiving information about their cancer history and ongoing oncology visits (77% to 86% across topics) . Across all topics, 93% to 99% of those who reported receiving information found the information useful . A minority of survivors reported they received information about symptoms to report to doctors, returning to work, or financial or legal issues (5% to 48% across topics), but those who did found the information useful (89% to 100% across topics) .Conclusions: In the absence of an SCP, CRC survivors still generally understood their cancer history . However, many lacked knowledge of ongoing risks and prevention . Most survivors stated that they found the survivorship infor-mation they received useful . SCPs for CRC survivors should focus less on past care and more on helping survivors understand their risks and plan for the future .

Visiting both a cancer specialist and pri-mary care physician is associated with better care for CRC survivors than visiting only one type of provider.20,21 To improve the coordination of specialty and primary care for cancer survivors, the Institute of Medicine (IOM) proposed the use of survi-vorship care plans (SCPs). An SCP contains a personalized cancer treatment summary and recommendations for ongoing care to be provided by the cancer specialist, the primary care provider, or other providers.22 Ideally, survivors share their SCPs with their primary care providers, in order to promote optimal post-treatment care. Although researchers have endorsed SCPs and

Key points

•For the 1.2 million colorectal cancer (CRC) survivors in the United States, the challenges of cancer extend beyond treatment completion. CRC survivors frequently do not undergo routine surveillance as recommended.

ASCO-CRC-2015-V2-INT.indb 67 4/8/2015 4:41:43 PM

Copyright by MT

Page 72: › medical_mx › images › ... · 5 Samantha Hendren; Ellen McKeown; Arden M . Morris; Sandra L . Wong; Mary Oerline; Lyndia Poe; Darrell A . Campbell Jr, and Nancy J . Birkmeyer

Hot Topics

68

suggested methods of implementation in clinical settings,23–31 many aspects of SCPs have not been evaluated, particularly for CRC survivors.32

SCPs are intended to help survivors seek out appropriate ongoing care, communi-cate their cancer history to a primary care provider, recognize important symptoms and late effects of their cancer, and find support resources. If CRC survivors are already aware of this information, SCPs may do little to improve the quality of their ongoing care. It is particularly important to understand whether survivors know enough about their cancer to seek out pri-mary care and communicate information about their medical history and follow-up care needs to a primary care provider.

Although CRC survivors have expressed enthusiasm for SCPs,33,34 their prefer-ences for the content and use of these plans have not been evaluated. Targeting SCPs to survivors’ needs and preferences can help maximize survivors’ benefit from SCPs. A plan that appeals to survivors has the potential to both enhance their understanding of survivorship issues and increase the likelihood that the survivor will share the information with a primary care provider.

Our study was designed to better under-stand whether CRC survivors who do not receive SCPs are equipped to communicate relevant information to primary care provid-ers and manage their own care. We also aimed to assess preferences for the con-tent, format, and delivery of SCPs.

■■ Methods

■■ Sample and data collectionWe conducted a telephone-administered survey of CRC survivors from two hospi-tals in the New York metropolitan area: Memorial Sloan-Kettering Cancer Center (MSKCC), a private tertiary not-for profit comprehensive cancer center, and Queens

Cancer Center (QCC), a comprehensive cancer center at Queens Hospital, a public hospital. We identified patients at both sites through hospital databases and medi-cal record review. Participants were eligible if they were 18 years or older at diagnosis of stages I–III CRC, had completed all non-surgical treatment for CRC at MSKCC or QCC between 6 and 24 months before the survey, had no evidence of disease at time of survey, and spoke English. We included survivors who received surgery at another hospital but received the remain-der of their treatment (chemotherapy, radiation therapy, or both) at MSKCC or QCC. Participants were ineligible if they were diagnosed with metachronous CRC, had metastatic disease, had a history of another cancer other than nonmelanoma skin cancer, received no treatment for their CRC, or received an SCP. At the time of the study, QCC patients and MSKCC patients who had completed treatment within the past 2 years were not offered SCPs. Each patient’s last-seen physician was provided an opportunity to exclude the patient from the study. Remaining eligible participants were mailed an invitation and called. Sur-veys were completed over the phone using a Computer-Assisted Telephone Interview system. All study participants gave verbal consent over the phone. We abstracted diagnosis and treatment data from the medical record. For patients with colon cancer, we used pathological staging. We used clinical stage for patients with rectal cancer when available; otherwise, we used pathological stage. The study was approved by the institutional review boards at both hospitals.

■■ QuestionnaireWe designed the survey to evaluate the information survivors may need and want from a survivorship care plan, as described in the IOM framework. Our goal in assess-ing knowledge of diagnosis and treatment was to understand whether patients with CRC had the minimum amount of informa-tion necessary to communicate their diag-nosis and treatment history to a primary

Key points

•SCPs are intended to help survivors seek out appropriate ongoing care, communi cate their cancer history to a primary care provider, recognize important symptoms and late effects of their cancer, and find support resources.

• It is particularly important to understand whether survivors know enough about their cancer to seek out primary care and communicate information about their medical history and follow-up care needs to a primary care provider.

•Our study was designed to better understand whether CRC survivors who do not receive SCPs are equipped to communicate relevant information to primary care providers and manage their own care.

•We conducted a telephone-administered survey of CRC survivors from two hospitals in the New York metropolitan area.

•We identified patients at both sites through hospital databases and medical record review.

•We included survivors who received surgery at another hospital but received the remainder of their treatment (chemotherapy, radiation therapy, or both) at MSKCC or QCC.

•Participants were ineligible if they were diagnosed with metachronous CRC, had metastatic disease, had a history of another cancer other than nonmelanoma skin cancer, received no treatment for their CRC, or received an SCP.

ASCO-CRC-2015-V2-INT.indb 68 4/8/2015 4:41:43 PM

Copyright by MT

Page 73: › medical_mx › images › ... · 5 Samantha Hendren; Ellen McKeown; Arden M . Morris; Sandra L . Wong; Mary Oerline; Lyndia Poe; Darrell A . Campbell Jr, and Nancy J . Birkmeyer

Colorectal Cancer Survivors’ Needs and Preferences

69

care provider. The questionnaire elicited 10 basic diagnosis and treatment details: when diagnosis occurred; the site and stage of disease; and whether and when the partici-pant had surgery, chemotherapy, or radia-tion therapy. We evaluated knowledge of ongoing risks and recommended testing by presenting four statements about risks of local recurrence, risk of distant recurrence, risk of second primary CRCs, and need for ongoing surveillance. For each state-ment, the response options were a 5-point Likert scale ranging from “strongly agree” to “strongly disagree.” We also assessed comparative perceived risk by asking how likely the participant was to have a new CRC compared with someone without a history of cancer, with response options on a 5-point Likert scale ranging from “much less likely” to “much more likely.” We assessed receipt of and preferences for survivorship information by asking whether the participant recalled receiving informa-tion (in a verbal or written manner) about each of 16 broad topics (adapted from the IOM framework) after treatment comple-tion. If participants reported receiving the information, we assessed whether the patient believed that it was useful. We also elicited preferences for timing of receipt of information and format of delivery.

■■ AnalysisSurvey responses were compared with medical record data to assess participant knowledge of their cancer history. Self-reported dates of diagnosis and treatment were considered accurate if they were within ± 30 days of the date in the medi-cal record. Participants’ knowledge was considered to be accurate if responses to true statements were “strongly agree” or “agree.” For the items that compared CRC risk to that of the general popula-tion, “much more likely” and “somewhat more likely” were considered to be accu-rate responses. All other responses were considered to be inaccurate. Receipt of information and preferences for informa-tion were described using means and per-centages. Binomial exact tests were used

Key points

•We evaluated knowledge of ongoing risks and recommended testing by presenting four statements about risks of local recurrence, risk of distant recurrence, risk of second primary CRCs, and need for ongoing surveillance.

•We assessed comparative perceived risk by asking how likely the participant was to have a new CRC compared with someone without a history of cancer, with response options on a 5-point Likert scale.

• If participants reported receiving the information, we assessed whether the patient believed that it was useful. We also elicited preferences for timing of receipt of information and format of delivery.

•There were no differences between those who refused and participants in terms of sex, tumor site, stage, receipt of surgery, and receipt of chemotherapy or radiation therapy.

•The mean age was 59 years, and the mean time since treatment completion was approximately 16 months (standard deviation = 5 months).

•Eighty-eight percent knew whether their disease was in their colon or rectum, and 99% to 100% accurately reported whether they had surgery, chemotherapy, or radiation therapy.

•Eighty-six percent of patients accurately reported their stage as nonmetastatic.

to determine 95% CIs. When there were small numbers and 100% in one group, one-sided 97.5% CIs were calculated. Stata/SE version 11.0 (StataCorp LP, College Station, TX) was used for all analyses.

■■ Results

■■ SampleOf the 333 eligible participants identified via medical record, 202 (61%) agreed to participate. After confirmation of eligi-bility by phone, 175 participants (87%) completed the survey. There were no dif-ferences between those who refused and participants in terms of sex, tumor site, stage, receipt of surgery, and receipt of chemotherapy or radiation therapy. Study participants were diagnosed between December 2007 and November 2010 and were interviewed between July 2010 and November 2011. As a result of the small number of patients with CRC seen at QCC, nearly all participants (97%) came from MSKCC. The mean age was 59 years, and the mean time since treatment completion was approximately 16 months (standard deviation = 5 months; Table 1).

■■ Knowledge of diagnosis and treatmentRespondents generally had strong basic knowledge of their diagnosis and treat-ment (Table 2). Eighty-eight percent knew whether their disease was in their colon or rectum, and 99% to 100% accurately reported whether they had surgery, chemo-therapy, or radiation therapy. Across the three treatment types, 77% to 94% of respondents reported the last date of treat-ment accurately. Seventy-nine percent of patients with colon cancer and 47% of patients with rectal cancer correctly reported their stage. Nineteen percent of patients with rectal cancer and 13% of patients with colon cancer stated that they did not remember their stage. Eighty-six percent of patients accurately reported their stage as nonmetastatic. We also inves-tigated whether patients who require more intensive monitoring (stages II and III) knew

ASCO-CRC-2015-V2-INT.indb 69 4/8/2015 4:41:44 PM

Copyright by MT

Page 74: › medical_mx › images › ... · 5 Samantha Hendren; Ellen McKeown; Arden M . Morris; Sandra L . Wong; Mary Oerline; Lyndia Poe; Darrell A . Campbell Jr, and Nancy J . Birkmeyer

Hot Topics

70

their stage; 79% of this higher risk group reported their stage as II or III.

■■ Knowledge of ongoing risks and recommended testing

Sixty-nine percent and 77%, respectively, correctly agreed that CRC could return at the original site of disease or in another part of the body (Table 2). Forty percent felt they were more likely to have a new CRC than a person who has never had CRC. Almost all participants (95%) agreed that they needed to continue with routine sur-veillance of their colon and rectum.

■■ Receipt of informationReported receipt of the 16 topics of infor-mation is shown in Table 3. Seventy-eight percent and 86% reported that they had received information about their diagnosis and treatment history, respectively. There was a wider range of reported receipt of information about future oncologic and nononcologic care. Most respondents reported receiving information about the importance of visiting an oncologist (77%), the need for tests to monitor for recur-rence (82%), and information about diet and exercise (66%). Only 38% reported being told of the importance of visiting a primary care provider after treatment com-pletion. Nonmedical information regarding such topics as support groups, return to work, and potential health insurance and legal issues was not commonly reported as having been received, with rates ranging from 5% to 51% of respondents across six categories.

■■ Preferences for informationFor each of the 16 topics, at least two thirds of respondents who received the information reported that they found it useful (Table 3). Summaries of diagnosis and treatment were found useful by 93% and 96%, respectively, of respondents who received that information. For information about the eight topics of ongoing medi-cal needs, between 94% and 100% of those who reported receiving information reported that it was useful. Among those

■■ TABLE 1 - Participant demographic and clinical characteristics (N = 175)

Characteristic No. %

Hospital

Memorial Sloan-Kettering Cancer Center

170 97

Queens Cancer Center 5 3

Sex

Male 90 51

Female 85 49

Race/ethnicity

White 150 86

Hispanic 5 3

Black 13 7

Asian 6 3

Other/unknown 1 1

Age at diagnosis, years

Mean 57

SD 12

Age at survey, years

Mean 59

SD 12

Time between diagnosis and survey, months

Mean 23

SD 6

Tumor site

Colon 101 58

Rectum 73 42

Both 1 1

Treatment received

Surgery 171 98

Chemotherapy 131 75

Radiation 52 30

Colorectal cancer stage

I 35 20

II 48 27

III 92 53

Clinical trial participation

Yes 10 6

No 165 94

SD, standard deviation.

Key points

•Sixty-nine percent and 77%, respectively, correctly agreed that CRC could return at the original site of disease or in another part of the body.

•Almost all participants (95%) agreed that they needed to continue with routine surveillance of their colon and rectum.

•Seventy-eight percent and 86% reported that they had received information about their diagnosis and treatment history.

•Only 38% reported being told of the importance of visiting a primary care provider after treatment completion.

•Nonmedical information regarding such topics as support groups, return to work, and potential health insurance and legal issues was not commonly reported as having been received, with rates ranging from 5% to 51% of respondents across six categories.

•At least two thirds of respondents who received the information reported that they found it useful.

ASCO-CRC-2015-V2-INT.indb 70 4/8/2015 4:41:44 PM

Copyright by MT

Page 75: › medical_mx › images › ... · 5 Samantha Hendren; Ellen McKeown; Arden M . Morris; Sandra L . Wong; Mary Oerline; Lyndia Poe; Darrell A . Campbell Jr, and Nancy J . Birkmeyer

Colorectal Cancer Survivors’ Needs and Preferences

71

who received information about nonmedi-cal needs, between 68% and 100% (across six topics) reported that the information was useful.

■■ Additional topics of importanceWhen asked whether there were addi-tional topics of information participants would like to have received after treat-ment completion, there were 62 com-ments about late effects and challenges to expect, such as the persistence of fatigue and bowel symptoms. Twenty-two com-ments addressed the need for information about general health, such as guidance about diet. There were 12 comments about knowing more about what follow-up is rec-ommended, including recommended tests and which physicians may be sensitive to the needs of cancer survivors. Twelve com-ments concerned the need for information

about cancer, such as recurrence risk and prognosis. Finally, there were seven com-ments that fell outside these categories, including finding local options for ongoing care, getting personalized information, and finding reliable sources of information.

■■ Preferences for timing and format of information delivery

Fifty-nine percent of respondents vol-unteered that they would have liked to have had survivorship information pro-vided during their treatment. Twenty-one percent preferred to receive information immediately after treatment completion, and 8% preferred to receive information several months after treatment comple-tion. Although not offered as a response option, 11% of respondents volunteered that they would have liked to have received survivorship information before treatment.

■■ TABLE 2 - Accuracy of self-reported cancer information and risk perception

Item n No. correct % correct 95% CI

Cancer information

Tumor site (colon vs. rectum) 175 154 88 82 to 92

Treatment received

Surgery 175 174 99 97 to 99

Chemotherapy 175 174 99 97 to 99

Radiation therapy 175 175 100 98 to 100*

Date of treatment (± 30 days)

Diagnosis 175 156 89 84 to 93

Surgery 171 160 94 89 to 97

Last chemotherapy treatment 131 101 77 69 to 84

Last radiation therapy treatment 52 46 88 77 to 96

Cancer stage 175 114 65 58 to 73

Clinical trial participation 161 151 94 89 to 97

Risk perception

Risk of recurrence or second CRC† 167 116 69 62 to 76

Risk of metastasis 175 134 77 70 to 83

Need for ongoing surveillance† 167 159 95 90 to 98

*One-sided 97.5% CI.†Participants who underwent a total colectomy were excluded.CRC, colorectal cancer.

Key points

•There were 12 comments about knowing more about what follow-up is recommended, including recommended tests and which physicians may be sensitive to the needs of cancer survivors.

•There were seven comments that fell outside these categories, including finding local options for ongoing care, getting personalized information, and finding reliable sources of information.

•Twenty-one percent preferred to receive information immediately after treatment completion.

ASCO-CRC-2015-V2-INT.indb 71 4/8/2015 4:41:44 PM

Copyright by MT

Page 76: › medical_mx › images › ... · 5 Samantha Hendren; Ellen McKeown; Arden M . Morris; Sandra L . Wong; Mary Oerline; Lyndia Poe; Darrell A . Campbell Jr, and Nancy J . Birkmeyer

Hot Topics

72

Respondents were able to select more than one preferred format for receiving survi-vorship information. Ninety-three percent stated they would like a conversation with their doctor, and 75% wanted a personal-ized printed document. Only 61% wanted to receive information from a Web site.

■■ Discussion

Our study found that among survivors who had not received an SCP, knowledge of the most salient aspects of diagnosis and treatment was generally accurate. This confirms prior evidence that CRC survivors know at least basic information about their illness. An Australian study evaluated CRC survivors’ recall of their own diag-nosis and found relatively good recall for

broad categories of information (eg, which symptoms prompted diagnosis).35 A more recent American study found that among CRC survivors in an integrated health care system (mean of 5.8 years from diagnosis), 89% knew the site of their disease; 88% knew the year of diagnosis; and 95% and 96% knew whether they had chemother-apy or radiation, respectively.36 Although our study included more recently diagnosed CRC survivors (mean 23 months from diag-nosis), participants in our study had simi-larly high accuracy in their recall of disease site, diagnosis year, and receipt of treat-ment. In both studies, only approximately two thirds of survivors accurately recalled their disease stage. The errors in staging were found primarily among rectal cancer survivors, who typically have a more com-plicated method of staging. Patients with

■■ TABLE 3 - Information given to survivors after completion of cancer treatment

Topic n Received information

Found information useful

No. % 95% CI No.* % 95% CI

Type of cancer 174 136 78 71 to 84 127 93 88 to 97

Type of treatment 174 149 86 80 to 90 143 96 91 to 99

Visiting a cancer doctor in the future 175 135 77 70 to 83 131 97 93 to 99

How often to visit a cancer doctor in the future 174 145 83 77 to 89 144 99 96 to 99

Visiting a PCP in the future 172 66 38 31 to 46 63 95 87 to 99

How often to visit a PCP in the future 173 31 31 13 to 24 29 94 79 to 99

Tests to make sure CRC has not come back 175 143 82 75 to 87 141 99 95 to 99

Test to check for other cancers 175 71 41 33 to 48 69 97 90 to 99

Symptoms to tell your doctor about 173 83 48 40 to 56 83 100 96 to 100*

Diet and exercise to keep you healthy 174 115 66 59 to 73 108 94 88 to 98

Support groups for people who have had CRC 175 90 51 44 to 59 61 68 57 to 77

Web sites for people who have had CRC 175 37 21 15 to 28 28 76 59 to 88

People to talk to if feeling sad or anxious 175 89 51 43 to 58 68 76 66 to 85

Going back to work after treatment 173 70 40 33 to 48 63 90 80 to 96

Health insurance or financial assistance issues 174 44 25 19 to 33 39 89 75 to 96

Legal issues related to employment or health insurance coverage 174 9 5 2 to 10 9 100 66 to 100*

*Denominator may not include all those who received information because of missing responses.†One-sided 97.5% CI.CRC, colorectal cancer; PCP, primary care provider.

Key points

•Our study found that among survivors who had not received an SCP, knowledge of the most salient aspects of diagnosis and treatment was generally accurate. This confirms prior evidence that CRC survivors know at least basic information about their illness.

•Approximately two thirds of survivors accurately recalled their disease stage. The errors in staging were found primarily among rectal cancer survivors, who typically have a more complicated method of staging.

ASCO-CRC-2015-V2-INT.indb 72 4/8/2015 4:41:44 PM

Copyright by MT

Page 77: › medical_mx › images › ... · 5 Samantha Hendren; Ellen McKeown; Arden M . Morris; Sandra L . Wong; Mary Oerline; Lyndia Poe; Darrell A . Campbell Jr, and Nancy J . Birkmeyer

Colorectal Cancer Survivors’ Needs and Preferences

73

rectal cancer are staged before treatment (clinical staging) and again after surgery (pathological staging), which may confuse patients. In our study, 79% of higher risk survivors (who need more intensive surveil-lance) accurately reported having either stage II or stage III disease. If primary care providers rely solely on survivors to report their stage and guide follow-up care, incor-rect reporting of stage may result in under-use of surveillance.

Some respondents in our study (23% to 31%) reported that they were not at risk for recurrence and second primary CRCs, sug-gesting that they were either unaware of their risks or were expressing optimism about remaining cancer free. Although risk per-ception is often seen as predictive of health behaviors, in our population, 95% of CRC survivors agreed that ongoing surveillance was important. Ideally, a future study would see how closely the intention to receive sur-veillance is related to actual surveillance.

Taken together, survivors in our study gen-erally recalled basic information about their diagnosis and treatment, although they recalled less information about their stage and ongoing risks. Also, respondents more frequently reported receiving medical infor-mation about their cancer or its follow-up than about nonmedical issues, such as sup-port groups, counseling, and financial and insurance issues. The lack of communica-tion about nonmedical issues, ranging from psychosocial support to practical problems, highlights the need for multidisciplinary support for cancer survivors. Many of these topics fall beyond the reasonable purview of oncology providers and suggest that oncology care must at least be sup-plemented with nonclinical support. SCPs document available supportive resources for CRC survivors and may facilitate the use of these important services.

A troubling finding in our study was that only 38% of respondents recalled being told about the importance of primary care. If oncology providers are not providing

this information to survivors, it may, in part, indicate a confidence on the part of the oncologist that survivors are already being seen by a primary care provider. Nevertheless, coordinated post-treatment care depends on active participation of both oncology and primary care providers to ensure comprehensive, nonduplica-tive care.22 The SCP has two roles in the coordination of care. First, it informs or reminds the survivor of the importance of visiting a primary care provider. Second, if the primary care provider receives a copy of the SCP (from either the survivor or directly from the oncology provider), the primary care provider gets a concise summary of the survivor’s treatment history, information about late effects and ongoing risks that may help guide ongoing primary care, and information about how the oncology pro-vider intends to share post-treatment care.

SCPs serve not only as communication between providers, but also as a way for patients to know what to expect and how to manage their own health. Although not explicitly asked whether they would like to have received an SCP, survivors in our study broadly found the information they received to be useful, and many reported wishing they had received even more information, such as their specific risks for recurrence, changes to their body, recovery time, and symptoms to expect. Survivors also favored receiving this information early: the majority wanted to receive information during treat-ment. Creating a document that includes a plan of care at the start of treatment could be used as the basis for a summary of treatment to be delivered after treatment completion. Whether patients and survi-vors would appreciate having an ongoing dynamic document remains untested.

This study was limited in that almost all patients were from one cancer center that is likely not representative of all CRC survivors. However, the similarity of our findings to those in a CRC survivor cohort from an integrated health care system in the Midwest suggests that our findings

Key points

•Some respondents in our study (23% to 31%) reported that they were not at risk for recurrence and second primary CRCs, suggesting that they were either unaware of their risks or were expressing optimism about remaining cancer free.

•Survivors in our study generally recalled basic information about their diagnosis and treatment, although they recalled less information about their stage and ongoing risks.

•The lack of communication about nonmedical issues, ranging from psychosocial support to practical problems, highlights the need for multidisciplinary support for cancer survivors.

•A troubling finding in our study was that only 38% of respondents recalled being told about the importance of primary care.

•SCPs serve not only as communication between providers, but also as a way for patients to know what to expect and how to manage their own health.

•Creating a document that includes a plan of care at the start of treatment could be used as the basis for a summary of treatment to be delivered after treatment completion. Whether patients and survivors would appreciate having an ongoing dynamic document remains untested.

ASCO-CRC-2015-V2-INT.indb 73 4/8/2015 4:41:44 PM

Copyright by MT

Page 78: › medical_mx › images › ... · 5 Samantha Hendren; Ellen McKeown; Arden M . Morris; Sandra L . Wong; Mary Oerline; Lyndia Poe; Darrell A . Campbell Jr, and Nancy J . Birkmeyer

Hot Topics

74

are externally valid.36 The study was not sufficiently powered to determine differ-ences by cancer center or other patient characteristics. Future studies should focus on CRC survivors in other hospital set-tings (including public hospitals), in order to explore such variation. By design, we only asked basic information about cancer diagnosis and treatment, reasoning that this was the minimum amount a primary care provider would need to provide optimal post-treatment care. However, it is possible that CRC survivors would be unable to recall more detailed treatment information, such as specific chemotherapy agents and toxicity information that could be useful to a primary care provider. Also, patients may not have been able to recall accurately whether they received survivor-ship information when completing treat-ment. Although receipt of information is likely underreported in this study, limited recall underscores the need for informa-tion to be presented in a format that can be reviewed or remembered more than

6 months after treatment completion, when survivorship needs are still salient. Finally, respondents with no evidence of disease may, with the benefit of hindsight, prefer knowing more information than they may have wanted when they initially com-pleted treatment. Survivorship information, particularly about ongoing risks, may be more overwhelming to receive at the time of treatment completion, when the future may feel uncertain. Preference assessment may be useful at various times during the survivorship period.

In summary, although CRC survivors may not need an SCP to communicate basic treatment history with their primary care providers, they prefer to be given this infor-mation. CRC survivors reported that the receipt of survivorship information was use-ful, even in the first 2 years after treatment completion. The primary benefit of SCPs for CRC survivors may be less in recording past care and more in helping them understand their risks and plan for the future.

■■ References 1. American Cancer Society: Cancer Treatment and Survivorship

Facts & Figures 2012–2013. Atlanta, GA, American Cancer Society, 2013.

2. Mysliwiec P, Cronin K, Schatzkin A: New malignancies following cancer of the colon, rectum, and anus, in Curtis RE, Freedman DM, Ron E, et al (eds): New Malignancies Among Cancer Survivors: SEER Cancer Registries, 1973–2000, Bethesda, MD, National Cancer Institute, 2006, pp 111–144.

3. Elston Lafata J, Cole Johnson C, Ben-Menachem T, et al: Sociodemographic differences in the receipt of colorectal cancer surveillance care following treatment with curative intent. Med Care 39:361–372, 2001.

4. Ellison GL, Warren JL, Knopf KB, et al: Racial differences in the receipt of bowel surveillance following potentially curative colorectal cancer surgery. Health Serv Res 38: 1885–1903, 2003.

5. Rulyak SJ, Mandelson MT, Brentnall TA, et al: Clinical and sociodemographic factors associated with colon surveillance among patients with a history of colorectal cancer. Gastrointest Endosc 59:239–247, 2004.

6. Elston Lafata J, Simpkins J, Schultz L, et al: Routine surveillance care after cancer treatment with curative intent. Med Care 43:592–599, 2005.

7. Cooper GS, Payes JD: Temporal trends in colorectal procedure use after colorectal cancer resection. Gastrointest Endosc 64:933–940, 2006.

8. Rolnick S, Hensley Alford S, Kucera GP, et al: Racial and age differences in colon examination surveillance following a diagnosis of colorectal cancer. J Natl Cancer Inst Monogr 96–101, 2005.

9. Cooper GS, Kou TD, Reynolds HL Jr: Receipt of guideline-recommended follow-up in older colorectal cancer survivors: A population-based analysis. Cancer 116:2029–2037, 2008.

10. Hu CY, Delclos GL, Chan W, et al: Post-treatment surveillance in a large cohort of patients with colon cancer. Am J Manag Care 17:329–336, 2011.

11. Deimling GT, Bowman KF, Sterns S, et al: Cancer-related health worries and psychological distress among older adult, long-term cancer survivors. Psychooncology 15:306–320, 2006.

12. Ramsey SD, Berry K, Moinpour C, et al: Quality of life in long term survivors of colorectal cancer. Am J Gastroenterol 97:1228–1234, 2002.

13. Stein KD, Syrjala KL, Andrykowski MA: Physical and psychological long-term and late effects of cancer. Cancer 112:2577–2592, 2008 (suppl).

14. Grant M, McMullen CK, Altschuler A, et al: Gender differences in quality of life among long-term colorectal cancer survivors with ostomies. Oncol Nurs Forum 38: 587–596, 2011.

15. Sprangers MA, Taal BG, Aaronson NK, et al: Quality of life in colorectal cancer: Stoma vs. nonstoma patients. Dis Colon Rectum 38:361–369, 1995.

Key points

•The study was not sufficiently powered to determine differences by cancer center or other patient characteristics.

•Future studies should focus on CRC survivors in other hospital settings (including public hospitals), in order to explore such variation.

• In summary, although CRC survivors may not need an SCP to communicate basic treatment history with their primary care providers, they prefer to be given this information.

•CRC survivors reported that the receipt of survivorship information was useful, even in the first 2 years after treatment completion.

ASCO-CRC-2015-V2-INT.indb 74 4/8/2015 4:41:44 PM

Copyright by MT

Page 79: › medical_mx › images › ... · 5 Samantha Hendren; Ellen McKeown; Arden M . Morris; Sandra L . Wong; Mary Oerline; Lyndia Poe; Darrell A . Campbell Jr, and Nancy J . Birkmeyer

Colorectal Cancer Survivors’ Needs and Preferences

75

16. Jansen L, Herrmann A, Stegmaier C, et al: Health-related quality of life during the 10 years after diagnosis of colorectal cancer: A population-based study. J Clin Oncol 29: 3263–3269, 2011.

17. Caravati-Jouvenceaux A, Launoy G, Klein D, et al: Health-related quality of life among long-term survivors of colorectal cancer: A population-based study. Oncologist 16: 1626–1636, 2011.

18. Earle CC, Neville BA: Under use of necessary care among cancer survivors. Cancer 101:1712–1719, 2004.

19. Snyder CF, Frick KD, Herbert RJ, et al: Quality of care for comorbid conditions during the transition to survivorship: Differences between cancer survivors and noncancer controls. J Clin Oncol 31:1140–1148, 2013.

20. Snyder CF, Earle CC, Herbert RJ, et al: Preventive care for colorectal cancer survivors: A 5-year longitudinal study. J Clin Oncol 26:1073–1079, 2008.

21. Snyder CF, Earle CC, Herbert RJ, et al: Trends in follow-up and preventive care for colorectal cancer survivors. J Gen Intern Med 23:254–259, 2008.

22. Hewitt M, Greenfield S, Stovall E (eds): From Cancer Patient to Cancer Survivor: Lost in Transition. Washington, DC, National Academies Press, 2005.

23. Schrag D: The Cancer Treatment Summary: Changing the Culture of Documentation to Facilitate High-Quality Cancer Care. Alexandria, VA, American Society of Clinical Oncology, 2008. www.asco.org/sites/default/files/background_ on_treatment_summary_schrag.pdf

24. Earle CC: Failing to plan is planning to fail: Improving the quality of care with survivorship care plans. J Clin Oncol 24:5112–5116, 2006.

25. Ganz PA, Casillas J, Hahn EE: Ensuring quality care for cancer survivors: Implementing the survivorship care plan. Semin Oncol Nurs 24:208–217, 2008.

26. Ganz PA, Hahn EE: Implementing a survivorship care plan for patients with breast cancer. J Clin Oncol 26:759–767, 2008.

27. Miller R: Implementing a survivorship care plan for patients with breast cancer. Clin J Oncol Nurs 12:479–487, 2008.

28. McCabe MS, Jacobs L: Survivorship care: Models and programs. Semin Oncol Nurs 24:202–207, 2008.

29. Feuerstein M: The cancer survivorship care plan: Health care in the context of cancer. J Oncol Pract 5:113–115, 2009.

30. Hoffman B, Stovall E: Survivorship perspectives and advocacy. J Clin Oncol 24:5154–5159, 2006.

31. Horning SJ: Follow-up of adult cancer survivors: New paradigms for survivorship care planning. Hematol Oncol Clin North Am 22:201–210, vs. 2008.

32. Salz T, Oeffinger KC, McCabe MS, et al: Survivorship care plans in research and practice. CA Cancer J Clin 12:20142, 2012.

33. Baravelli C, Krishnasamy M, Pezaro C, et al: The views of bowel cancer survivors and health care professionals regarding survivorship care plans and post treatment follow up. J Cancer Surviv 3:99–108, 2009.

34. Hewitt ME, Bamundo A, Day R, et al: Perspectives on post-treatment cancer care: Qualitative research with survivors, nurses, and physicians. J Clin Oncol 25:2270–2273, 2007.

35. Lynch BM, Youlden D, Fritschi L, et al: Self-reported information on the diagnosis of colorectal cancer was reliable but not necessarily valid. J Clin Epidemiol 61:498–504, 2008.

36. Nissen MJ, Beran MS, Lee MW, et al: Views of primary care providers on follow-up care of cancer patients. Family Med 39:477–482, 2007.

ASCO-CRC-2015-V2-INT.indb 75 4/8/2015 4:41:44 PM

Copyright by MT

Page 80: › medical_mx › images › ... · 5 Samantha Hendren; Ellen McKeown; Arden M . Morris; Sandra L . Wong; Mary Oerline; Lyndia Poe; Darrell A . Campbell Jr, and Nancy J . Birkmeyer

ASCO-CRC-2015-V2-INT.indb 76 4/8/2015 4:41:44 PM

Copyright by MT

Page 81: › medical_mx › images › ... · 5 Samantha Hendren; Ellen McKeown; Arden M . Morris; Sandra L . Wong; Mary Oerline; Lyndia Poe; Darrell A . Campbell Jr, and Nancy J . Birkmeyer

■■ Landmark Clinical Trials

ASCO-CRC-2015-V2-INT.indb 77 4/8/2015 4:41:44 PM

Copyright by MT

Page 82: › medical_mx › images › ... · 5 Samantha Hendren; Ellen McKeown; Arden M . Morris; Sandra L . Wong; Mary Oerline; Lyndia Poe; Darrell A . Campbell Jr, and Nancy J . Birkmeyer

ASCO-CRC-2015-V2-INT.indb 78 4/8/2015 4:41:44 PM

Copyright by MT

Page 83: › medical_mx › images › ... · 5 Samantha Hendren; Ellen McKeown; Arden M . Morris; Sandra L . Wong; Mary Oerline; Lyndia Poe; Darrell A . Campbell Jr, and Nancy J . Birkmeyer

79

Landmark Clinical Trials

■■ Effect of flexible sigmoidoscopy screening on colorectal cancer incidence and mortality: a randomized clinical trial

AUTHORSHolme Ø, Løberg M, Kalager M, Bretthauer M, Hernán MA, Aas E, Eide TJ, Skovlund E, Schneede J, Tveit KM, Hoff G.

ERRATUM INJAMA 2014;312(9):964.

IMPORTANCEColorectal cancer is a major health burden. Screening is recommended in many countries.

OBJECTIVETo estimate the effectiveness of flexible sigmoidoscopy screening on colorectal cancer incidence and mortality in a population-based trial.

DESIGN, SETTING, AND PARTICIPANTSRandomized clinical trial of 100,210 individuals aged 50 to 64 years, identified from the population of Oslo city and Telemark County, Norway. Screening was performed in 1999–2000 (55–64-year age group) and in 2001 (50–54-year age group), with follow-up ending December 31, 2011. Of those selected, 1415 were excluded due to prior colorectal cancer, emigration, or death, and 3 could not be traced in the population registry.

INTERVENTIONSParticipants randomized to the screening group were invited to undergo screening. Within the screening group, participants were randomized 1:1 to receive once-only flexible sigmoidoscopy or combination of once-only flex-ible sigmoidoscopy and fecal occult blood testing (FOBT). Participants with positive screening test results (cancer, adenoma, polyp ≥ 10 mm, or positive FOBT) were offered colonoscopy. The control group received no intervention.

MAIN OUTCOMES AND MEASURESColorectal cancer incidence and mortality.

RESULTSA total of 98,792 participants were included in the intention-to-screen analyses, of whom 78,220

comprised the control group and 20,572 comprised the screening group (10,283 randomized to receive a flexible sigmoidoscopy and 10,289 to receive flexible sigmoidoscopy and FOBT). Adherence with screening was 63%. After a median of 10.9 years, 71 participants died of colorectal cancer in the screening group vs. 330 in the control group (31.4 vs. 43.1 deaths per 100,000 person-years; absolute rate difference, 11.7 [95% CI, 3.0–20.4]; hazard ratio [HR], 0.73 [95% CI, 0.56–0.94]). Colorectal cancer was diagnosed in 253 participants in the screening group vs. 1086 in the control group (112.6 vs. 141.0 cases per 100,000 person-years; abso-lute rate difference, 28.4 [95% CI, 12.1–44.7]; HR, 0.80 [95% CI, 0.70–0.92]). Colorectal cancer incidence was reduced in both the 50- to 54-year age group (HR, 0.68; 95% CI, 0.49–0.94) and the 55- to 64-year age group (HR, 0.83; 95% CI, 0.71–0.96). There was no dif-ference between the flexible sigmoidoscopy only vs. the flexible sigmoidoscopy and FOBT screening groups.

CONCLUSIONS AND RELEVANCEIn Norway, once-only flexible sigmoidoscopy screening or flexible sigmoidoscopy and FOBT reduced colorectal can-cer incidence and mortality on a population level com-pared with no screening. Screening was effective both in the 50- to 54-year and the 55- to 64-year age groups.

REFERENCEJAMA 2014;312(6):606–15.

■■ Preoperative magnetic resonance imaging assessment of circumferential resection margin predicts disease-free survival and local recurrence: 5-year follow-up results of the MERCURY study

AUTHORSTaylor FG, Quirke P, Heald RJ, Moran BJ, Blomqvist L, Swift IR, Sebag-Montefiore D, Tekkis P, Brown G; Mag-netic Resonance Imaging in Rectal Cancer European Equivalence Study Study Group.

PURPOSEThe prognostic relevance of preoperative high-resolution magnetic resonance imaging (MRI) assessment of cir-cumferential resection margin (CRM) involvement is unknown. This follow-up study of 374 patients with

ASCO-CRC-2015-V2-INT.indb 79 4/8/2015 4:41:44 PM

Copyright by MT

Page 84: › medical_mx › images › ... · 5 Samantha Hendren; Ellen McKeown; Arden M . Morris; Sandra L . Wong; Mary Oerline; Lyndia Poe; Darrell A . Campbell Jr, and Nancy J . Birkmeyer

rectal cancer reports the relationship between preopera-tive MRI assessment of CRM staging, American Joint Committee on Cancer (AJCC) TNM stage, and clinical variables with overall survival (OS), disease-free survival (DFS), and time to local recurrence (LR).

PATIENTS AND METHODSPatients underwent protocol high-resolution pelvic MRI. Tumor distance to the mesorectal fascia of ≤ 1 mm was recorded as an MRI-involved CRM. A Cox proportional hazards model was used in multivariate analysis to determine the relationship of MRI assessment of CRM to survivorship after adjusting for preoperative covariates.

RESULTSSurviving patients were followed for a median of 62 months. The 5-year OS was 62.2% in patients with MRI-clear CRM compared with 42.2% in patients with MRI-involved CRM with a hazard ratio (HR) of 1.97 (95% CI, 1.27 to 3.04; P < .01). The 5-year DFS was 67.2% (95% CI, 61.4% to 73%) for MRI-clear CRM compared with 47.3% (95% CI, 33.7% to 60.9%) for MRI-involved CRM with an HR of 1.65 (95% CI, 1.01 to 2.69; P < .05). Local recurrence HR for MRI-involved CRM was 3.50 (95% CI, 1.53 to 8.00; P < .05). MRI-involved CRM was the only preoperative staging parameter that remained significant for OS, DFS, and LR on multivariate analysis.

CONCLUSIONHigh-resolution MRI preoperative assessment of CRM status is superior to AJCC TNM-based criteria for assessing risk of LR, DFS, and OS. Furthermore, MRI CRM involvement is significantly associated with distant metastatic disease; therefore, colorectal cancer teams could intensify treatment and follow-up accordingly to improve survival outcomes.

REFERENCEJ Clin Oncol 2014;32(1):34–43.

■■ Tumor regression grading after preoperative chemoradiotherapy for locally advanced rectal carcinoma revisited: updated results of the CAO/ARO/AIO-94 trial

AUTHORSFokas E, Liersch T, Fietkau R, Hohenberger W, Beissbarth T, Hess C, Becker H, Ghadimi M, Mrak K, Merkel S, Raab HR, Sauer R, Wittekind C, Rödel C.

PURPOSEWe previously described the prognostic impact of tumor regression grading (TRG) on the outcome of patients with rectal carcinoma treated with preoperative chemo-radiotherapy (CRT) in the CAO/ARO/AIO-94 trial. Here we report long-term results after a median follow-up of 132 months.

PATIENTS AND METHODSTRG after preoperative CRT was determined in 386 surgical specimens by the amount of viable tumor cells versus fibrosis, ranging from TRG 4 (no viable tumor cells) to TRG 0 (no signs of regression). Clinicopatho-logic parameters and TRG were correlated to the cumu-lative incidence of local recurrence, distant metastasis, and disease-free survival (DFS).

RESULTSTen-year cumulative incidence of distant metastasis and DFS were 10.5% and 89.5% for patients with TRG 4 (complete regression), 29.3% and 73.6% for TRG 2 and 3 (intermediate regression), and 39.6% and 63% for TRG 0 and 1 (poor regression), respectively (P = .005 and P = .008, respectively). On multivariable analysis, residual lymph node metastasis (ypN+) and TRG were the only independent prognostic factors for cumulative incidence of distant metastasis (P < .001 and P = .035, respec-tively) and DFS (P < .001 and P = .039, respectively), whereas local recurrence was significantly affected by ypN status (P < .001) and lymphatic invasion (P = .026).

CONCLUSIONComplete and intermediate tumor regressions were associated with improved long-term outcome in patients with rectal carcinoma after preoperative CRT independ-ent of clinicopathologic parameters. This classification system needs to be prospectively tested in multiple data sets to validate its reproducibility in a wider setting.

ASCO-CRC-2015-V2-INT.indb 80 4/8/2015 4:41:44 PM

Copyright by MT

Page 85: › medical_mx › images › ... · 5 Samantha Hendren; Ellen McKeown; Arden M . Morris; Sandra L . Wong; Mary Oerline; Lyndia Poe; Darrell A . Campbell Jr, and Nancy J . Birkmeyer

REFERENCEJ Clin Oncol 2014;32(15):1554–62.

■■ Neoadjuvant chemotherapy without routine use of radiation therapy for patients with locally advanced rectal cancer: a pilot trial

AUTHORSSchrag D, Weiser MR, Goodman KA, Gonen M, Hollywood E, Cercek A, Reidy-Lagunes DL, Gollub MJ, Shia J, Guillem JG, Temple LK, Paty PB, Saltz LB.

PURPOSEAlthough neoadjuvant chemoradiotherapy achieves low local recurrence rates in clinical stages II to III rectal can-cer, it delays administration of optimal chemotherapy. We evaluated preoperative infusional fluorouracil, leucovorin, and oxaliplatin (FOLFOX)/bevacizumab with selective rather than consistent use of chemoradiotherapy.

PATIENTS AND METHODSThirty-two patients with clinical stages II to III rectal cancer participated in this single-center phase II trial. All were candidates for low anterior resection with total mesorectal excision (TME). Patients were to receive six cycles of FOL-FOX, with bevacizumab included for cycles 1 to 4. Patients with stable/progressive disease were to have radiation

before TME, whereas responders were to have immediate TME. Postoperative radiation was planned if R0 resection was not achieved. Postoperative FOLFOX × 6 was recom-mended, but adjuvant regimens were left to clinician dis-cretion. The primary outcome was R0 resection rate.

RESULTSBetween April 2007 and December 2008, 32 (100%) of 32 study participants had R0 resections. Two did not complete preoperative chemotherapy secondary to cardi-ovascular toxicity. Both had preoperative chemoradiother-apy and then R0 resections. Of 30 patients completing preoperative chemotherapy, all had tumor regression and TME without preoperative chemoradiotherapy. The pathologic complete response rate to chemother-apy alone was 8 of 32 (25%; 95% CI, 11% to 43%). The 4-year local recurrence rate was 0% (95% CI, 0% to 11%); the 4-year disease-free survival was 84% (95% CI, 67% to 94%).

CONCLUSIONFor selected patients with clinical stages II to III rectal can-cer, neoadjuvant chemotherapy and selective radiation does not seem to compromise outcomes. Preoperative Radiation or Selective Preoperative Radiation and Evalua-tion Before Chemotherapy and TME (PROSPECT), a ran-domized phase III trial to validate this experience, is now open in the US cooperative group network.

REFERENCEJ Clin Oncol 2014;32(6):513–8.

ASCO-CRC-2015-V2-INT.indb 81 4/8/2015 4:41:44 PM

Copyright by MT

Page 86: › medical_mx › images › ... · 5 Samantha Hendren; Ellen McKeown; Arden M . Morris; Sandra L . Wong; Mary Oerline; Lyndia Poe; Darrell A . Campbell Jr, and Nancy J . Birkmeyer

ASCO-CRC-2015-V2-INT.indb 82 4/8/2015 4:41:44 PM

Copyright by MT

Page 87: › medical_mx › images › ... · 5 Samantha Hendren; Ellen McKeown; Arden M . Morris; Sandra L . Wong; Mary Oerline; Lyndia Poe; Darrell A . Campbell Jr, and Nancy J . Birkmeyer

■■ Self-Assessment

ASCO-CRC-2015-V2-INT.indb 83 4/8/2015 4:41:44 PM

Copyright by MT

Page 88: › medical_mx › images › ... · 5 Samantha Hendren; Ellen McKeown; Arden M . Morris; Sandra L . Wong; Mary Oerline; Lyndia Poe; Darrell A . Campbell Jr, and Nancy J . Birkmeyer

ASCO-CRC-2015-V2-INT.indb 84 4/8/2015 4:41:45 PM

Copyright by MT

Page 89: › medical_mx › images › ... · 5 Samantha Hendren; Ellen McKeown; Arden M . Morris; Sandra L . Wong; Mary Oerline; Lyndia Poe; Darrell A . Campbell Jr, and Nancy J . Birkmeyer

85

1 Rectal cancer care has been the subject of intensive quality measurement and improvement programs in Europe. Which of the following statements is true regarding the results reported from the surgical qual-ity improvement program in Norway?

A . Local recurrence rates dropped from 12% to 6% .

B . 4-year survival increased from 60% to 73% .C . Both A and B are correct .D . The surgical quality improvement programs

have been developed mainly in the US . Results from European countries are lacking .

2 Adjuvant therapy plays a major role in treating colo-rectal cancer, and physicians’ views of its effective-ness influence treatment decisions. Which statement is NOT true regarding the survey on around 1,300 medical oncologists, radiation oncologists and sur-geons in the United States about the benefits and risks of adjuvant therapies for colorectal cancer?

A . Over 90% of responders believe the benefits of adjuvant therapies for stage III colorectal cancer outweigh the risks .

B . Only 22% believed in the net benefit of chemotherapy for stage II colon cancer .

C . Only 50% believed in the net benefit of radi-otherapy for stage II rectal cancer .

D . Only 15% believed in the net benefit of chemotherapy for stage II rectal cancer .

3 Which statement is true regarding the findings of the phase II ADORE trial, an open-label, multicentre, phase 2, randomized controlled trial of oxaliplatin, FU and leucovorin versus FU and leucovorin as adjuvant chemotherapy for locally advanced rectal cancer after preoperative chemoradiotherapy?

A . The primary endpoint was 5-year overall survival .

B . The trial included 150 patients .C . Any grade neutropenia, thrombocytopenia,

fatigue, nausea and sensory neuropathy were

significantly more common in the FOLFOX group than in the FU-leucovorin group .

D . There were no differences in the 3-year disease-free survival in the FOLFOX vs. FU-leucovorin treated patients .

4 Which of the following statement is NOT true regard-ing the results of the PETACC-8 trial, an open-label, randomized phase 3 trial of oxaliplatin, FU and leu-covorin with or without cetuximab in patients with resected stage III colon cancer?

A . Patients from nine European countries par-ticipated in the trial .

B . Patients were stratified by N-status, T-status and obstruction or perforation status .

C . The primary endpoint was 3-year overall survival .

D . The addition of cetuximab to FOLFOX4 chemotherapy did not improve disease free survival compared with FOLFOX4 alone in patients with KRAS exon 2 wild-type resected stage III colon cancer .

5 Which statement is true regarding the prognos-tic impact of deficient mismatch repair (dMMR) in stage II and III colon cancer patients?

A . dMMR tumors are associated with higher T stage .

B . dMMR tumors are associated with right-sided tumor location .

C . dMMR tumors are associated with improved overall survival .

D . All of the above are correct .

6 The results of the FACS randomized clinical trial on the effect of 3 to 5 years of scheduled CEA and CT follow-up to detect recurrence of colorectal cancer show that:

A . Among patients who had undergone cura-tive surgery for primary colorectal cancer,

■■ Questions

ASCO-CRC-2015-V2-INT.indb 85 4/8/2015 4:41:45 PM

Copyright by MT

Page 90: › medical_mx › images › ... · 5 Samantha Hendren; Ellen McKeown; Arden M . Morris; Sandra L . Wong; Mary Oerline; Lyndia Poe; Darrell A . Campbell Jr, and Nancy J . Birkmeyer

86

Questions

intensive imaging with CT or CEA screening, each provided an increased rate of surgical treatment of recurrence with curative intent compared with minimal follow-up .

B . There was no advantage in combining CEA and CT .

C . A and B are correct .D . None of the above is correct .

7 The LARCID trial explored the use of long-acting release octreotide in the prevention of chemotherapy-induced diarrhea in patients with colorectal cancer. The results of this trial show that:

A . The rate of diarrhea was similar in the experi-mental group receiving octreotide compared to the control group .

B . No benefit from octreotide was identified in terms of need for diarrhea treatment, opioids, or intravenous hydration or in the rate of hospitalization .

C . Octreotide treatment was associated with improvement in the quality of life in patients during adjuvant chemotherapy .

D . A and B are correct .

8 Which statement is true regarding the effect of flex-ible sigmoidoscopy screening on colorectal cancer incidence and mortality in Norway reported by Holme et al, in JAMA 2014?

A . Colorectal cancer incidence was reduced only in the 50 to 54 year age group .

B . Colorectal cancer incidence was reduced only in the 54 to 64 year age group .

C . There was no difference between the flexible sigmoidoscopy only versus the flexible sigmoi-doscopy and fecal occult blood testing groups .

D . A and C are correct .

9 Which of the following statements is NOT true regarding the prognostic impact of tumor regression grading on the outcome of patients with rectal carci-noma treated with preoperative chemoradiotherapy for locally advanced rectal carcinoma?

A . A score of 4 for tumor regression grading means that there are no viable tumor cells in the specimen examined .

B . A score of 0 for tumor regression grading means that there are no signs of regression .

C . On multivariate analysis, residual lymph node metastasis and tumor regression grading were independent prognostic factors for cumu-lative incidence of distant metastasis and disease-free survival .

D . On multivariate analysis, residual lymph node metastasis and tumor regression grading were independent prognostic factors for local recurrence .

10 The results of the MERCURY study at 5-years of follow-up show that high-resolution MRI preoperative assess-ment of circumferential resection margin (CRM) status is superior to AJCC TNM-based criteria for assessing risk of:

A . Local recurrence .B . Disease-free survival .C . Overall survival .D . All of the above .

ASCO-CRC-2015-V2-INT.indb 86 4/8/2015 4:41:45 PM

Copyright by MT

Page 91: › medical_mx › images › ... · 5 Samantha Hendren; Ellen McKeown; Arden M . Morris; Sandra L . Wong; Mary Oerline; Lyndia Poe; Darrell A . Campbell Jr, and Nancy J . Birkmeyer

■■ Self-Assessment

ASCO-CRC-2015-V2-INT.indb 87 4/8/2015 4:41:45 PM

Copyright by MT

Page 92: › medical_mx › images › ... · 5 Samantha Hendren; Ellen McKeown; Arden M . Morris; Sandra L . Wong; Mary Oerline; Lyndia Poe; Darrell A . Campbell Jr, and Nancy J . Birkmeyer

ASCO-CRC-2015-V2-INT.indb 88 4/8/2015 4:41:45 PM

Copyright by MT

Page 93: › medical_mx › images › ... · 5 Samantha Hendren; Ellen McKeown; Arden M . Morris; Sandra L . Wong; Mary Oerline; Lyndia Poe; Darrell A . Campbell Jr, and Nancy J . Birkmeyer

89

■■ Answers

1 The correct answer is C.

The surgical quality improvement program in Norway resulted in local recurrence rates dropping from 12% to 6% and 4-year survival increasing from 60% to 73%. (Hendren S et al, Journal of Oncology Practice 2014).

2 The correct answer is D.

Over 90% of responders believe the benefits of adju-vant therapies for stage III colorectal cancer outweigh the risks. Only 22% and 50% believed in the net benefit of chemotherapy for stage II colon cancer and stage II rectal cancer, respectively. Only 50% believed in the net benefit of radiotherapy for stage II rectal cancer. (Wong AC et al, Journal of Oncology Practice 2014).

3 The correct answer is C.

The primary endpoint was 3-year disease-free survival. The trial included 321 patients. Any grade neutropenia, thrombocytopenia, fatigue, nausea and sensory neu-ropathy were significantly more common in the FOL-FOX group than in the FU-leucovorin group; however, there were no differences in the frequency of grade 3 or 4 adverse events. 3-year disease-free survival was 71.6% in the FOLFOX group compared with 62.9% in the FU plus leucovorin group (HR 0.657, 95% CI 0.434–0.994; p = 0.047) (Hong YS et al, Lancet Oncol 2014).

4 The correct answer is C.

2559 patients from nine European countries participated in the trial. Patients were stratified by N-status, T-status and obstruction or perforation status. The primary end-point was disease-free survival. The addition of cetuxi-mab to FOLFOX4 chemotherapy did not improve disease free survival compared with FOLFOX4 alone in patients with KRAS exon 2 wild-type resected stage III colon can-cer. (Taieb J et al, Lancet Oncol 2014).

5 The correct answer is D.

dMMR tumors were associated with higher T stage (dMMR, 11% T1/2 vs. 18% T3 vs. 21% T4, p < 0.001), and right-sided tumor location (dMMR, left 9% vs. right 27%, p < 0.001). Compared to MMR proficient tumors, dMMR was strongly associated with improved overall survival (HR 0.27, p = 0.01) (Sargent DJ et al, J Clin Oncol 32:5s, 2014, abstr 3507).

6 The correct answer is C.

Among patients who had undergone curative surgery for primary colorectal cancer, intensive imaging with CT or CEA screening, each provided an increased rate of surgical treatment of recurrence with curative intent compared with minimal follow-up. There was no advantage in combining CEA and CT. If there is a survival advantage to any strategy, it is likely to be small. (Primrose JN et al, JAMA 2014).

ASCO-CRC-2015-V2-INT.indb 89 4/8/2015 4:41:45 PM

Copyright by MT

Page 94: › medical_mx › images › ... · 5 Samantha Hendren; Ellen McKeown; Arden M . Morris; Sandra L . Wong; Mary Oerline; Lyndia Poe; Darrell A . Campbell Jr, and Nancy J . Birkmeyer

Answers

90

7 The correct answer is D.

The rate of diarrhea was 76.1% in the experimental group and 78.9% in the control group. Treatment with octreotide did not prevent or reduce the severity of chemotherapy-induced diarrhea. No benefit from octreotide therapy was identified in terms of need for diarrhea treatment, opioids, or intravenous hydration or in the rate of hospitalization or quality of life. (Hoff PM et al, J Clin Oncol 2014).

8 The correct answer is C.

Colorectal cancer incidence was reduced in both the 50 to 54 year age group (HR 0.68, 95% CI 0.49–0.94) and the 55 to 64 year age group (HR 0.83, 95% CI 0.70–0.96). There was no difference between the flex-ible sigmoidoscopy only versus the flexible sigmoidos-copy and fecal occult blood testing groups. (Holme et al, JAMA 2014).

9 The correct answer is D.

A score of 4 for tumor regression grading means that there are no viable tumor cells in the specimen examined, whereas 0 means that there are no signs of regression. On multivariate analysis, residual lymph node metastasis and tumor regression grading were independent prognostic factors for cumulative incidence of distant metastasis (p < 0.001 and p = 0.0035, respectively) and disease-free survival (p < 0.001 and p = 0.039, respectively), whereas local recurrence was significantly affected by residual lymph node metastasis (p < 0.001) and lymphatic invasion (p = 0.026). (Fokas E et al, J Clin Oncol 2014).

10 The correct answer is D.

High-resolution MRI preoperative assessment of circum-ferential resection margin status is superior to AJCC TNM-based criteria for assessing risk of local recurrence, disease-free survival and overall survival. The 5-year over-all survival was 62.2% in patients with MRI-clear CRM compared with 42.2% in patients with MRI-involved CRM with a HR of 1.97 (95% CI 1.27 to 3.04, p < 0.01). The 5-year disease-free survival was 67.2% (95% CI 61.4% to 73%) for MRI-clear CRM compared with 47.3% (95% CI 33.7% to 60.9%) for MRI-involved CRM with a HR of 1.65 (95% CI 1.01 to 2.69, p < 0.05). Local recurrence HR for MRI-involved CRM was 3.5 (95% CI 1.53 to 8, p < 0.05) (Taylor FG et al, J Clin Oncol 2014).

ASCO-CRC-2015-V2-INT.indb 90 4/8/2015 4:41:45 PM

Copyright by MT

Page 95: › medical_mx › images › ... · 5 Samantha Hendren; Ellen McKeown; Arden M . Morris; Sandra L . Wong; Mary Oerline; Lyndia Poe; Darrell A . Campbell Jr, and Nancy J . Birkmeyer

ASCO-CRC-2015-V2-INT.indb 91 4/8/2015 4:41:45 PM

Copyright by MT

Page 96: › medical_mx › images › ... · 5 Samantha Hendren; Ellen McKeown; Arden M . Morris; Sandra L . Wong; Mary Oerline; Lyndia Poe; Darrell A . Campbell Jr, and Nancy J . Birkmeyer

ASCO-CRC-2015-V2-INT.indb 92 4/8/2015 4:41:45 PM

Copyright by MT